You are on page 1of 137

HEPATITIS

1. The liver receives blood from two sources. The _____________ is responsible for
pumping blood rich in nutrients to the liver.
A. hepatic artery
B. hepatic portal vein
C. mesenteric artery
D. hepatic iliac vein
The answer is B. The liver receives blood from two sources. The hepatic portal vein
is responsible for pumping blood rich in nutrients to the liver.

2. Which statements are INCORRECT regarding the anatomy and physiology of


the liver? Select all that apply:
A. The liver has 3 lobes and 8 segments.
B. The liver produces bile which is released into the small intestine to help digest
fats.
C. The liver turns urea, a by-product of protein breakdown, into ammonia.
D. The liver plays an important role in the coagulation process.
The answers are A and C. The liver has 2 lobes (note 3), and the liver turns
ammonia (NOT urea), which is a by-product of protein breakdown, into ammonia. All
the other statements are true about liver’s anatomy and physiology.

3. You’re providing an in-service on viral hepatitis to a group of healthcare


workers. You are teaching them about the types of viral hepatitis that can turn into
chronic infections. Which types are known to cause ACUTE infections ONLY? Select
all that apply:
A. Hepatitis A
B. Hepatitis B
C. Hepatitis C
D. Hepatitis D
E. Hepatitis E
The answers are A and E. Only Hepatitis A and E cause ACUTE infections…not
chronic. Hepatitis B, C, and D can cause both acute and chronic infections.

4. Which patients below are at risk for developing complications related to a


chronic hepatitis infection, such as cirrhosis, liver cancer, and liver failure? Select all
that apply:
A. A 55-year-old male with Hepatitis A.
B. An infant who contracted Hepatitis B at birth.
C. A 32-year-old female with Hepatitis C who reports using IV drugs.
D. A 50-year-old male with alcoholism and Hepatitis D.
E. A 30-year-old who contracted Hepatitis E.
The answers are B, C, and D. Infants or young children who contract Hepatitis B are
at a very high risk of developing chronic Hepatitis B (which is why option B is
correct). Option C is correct because most cases of Hepatitis C turn into chronic
cases and IV drug use increases this risk even more. Option D is correct because
Hepatitis D occurs when Hepatitis B is present and constant usage of alcohol
damages the liver. Therefore, the patient is at high risk of developing chronic
hepatitis. Hepatitis A and E tend to only cause acute infections….not chronic.

5. A patient is diagnosed with Hepatitis A. The patient asks how a person can
become infected with this condition. You know the most common route of
transmission is?
A. Blood
B. Percutaneous
C. Mucosal
D. Fecal-oral
The answer is D. Hepatitis A is most commonly transmitted via the fecal-oral route.

6. Which of the following is NOT a common source of transmission for Hepatitis


A? Select all that apply:
A. Water
B. Food
C. Semen
D. Blood
The answers are C and D. The most common source for transmission of Hepatitis A
is water and food.

7. A 36-year-old patient’s lab work show anti-HAV and IgG present in the blood.
As the nurse you would interpret this blood work as?
A. The patient has an active infection of Hepatitis A.
B. The patient has recovered from a previous Hepatitis A infection and is now
immune to it.
C. The patient is in the preicetric phase of viral Hepatitis.
D. The patient is in the icteric phase of viral Hepatitis.
The answer is B. When a patient has anti-HAV (antibodies of the Hepatitis A virus)
and IgG, this means the patient HAD a past infection of Hepatitis A but it is now
gone, and the patient is immune to Hepatitis A now. If the patient had anti-HAV and
IgM, this means the patient has an active infection of Hepatitis A.

8. TRUE or FALSE: A patient with Hepatitis A is contagious about 2 weeks


before signs and symptoms appear and 1-3 weeks after the symptoms appear.
The answer is TRUE.
9. A 25-year-old patient was exposed to the Hepatitis A virus at a local
restaurant one week ago. What education is important to provide to this patient?
A. Inform the patient to notify the physician when signs and symptoms of viral
Hepatitis start to appear.
B. Reassure the patient the chance of acquiring the virus is very low.
C. Inform the patient it is very important to obtain the Hepatitis A vaccine
immediately to prevent infection.
D. Inform the patient to promptly go to the local health department to receive
immune globulin.
The answer is D. Since the patient was exposed to Hepatitis A, the patient would
need to take preventive measures to prevent infection because infection is possible.
The patient should not wait until signs and symptoms appear because the patient
can be contagious 2 weeks BEFORE signs and symptoms appear. The vaccine
would not prevent Hepatitis A from this exposure, but from possible future exposures
because it takes the vaccine 30 days to start working. The best answer is option D.
The patient would need to receive immune globulin to provide temporary immunity
within 2 weeks of exposure.

10. Select all the ways a person can become infected with Hepatitis B:
A. Contaminated food/water
B. During the birth process
C. IV drug use
D. Undercooked pork or wild game
E. Hemodialysis
F. Sexual intercourse
The answers are B, C, E, and F. Hepatitis B is spread via blood and body fluids. It
could be transmitted via the birthing process, IV drug use, hemodialysis, or sexual
intercourse etc.

11. A patient has completed the Hepatitis B vaccine series. What blood result below
would demonstrate the vaccine series was successful at providing immunity to
Hepatitis B?
A. Positive IgG
B. Positive HBsAg
C. Positive IgM
D. Positive anti-HBs
The answer is D. A positive anti-HBs (Hepatitis B surface antibody) indicates either a
past infection of Hepatitis B that is now cleared and the patient is immune, OR that
the vaccine has been successful at providing immunity. A positive HBsAg (Hepatitis
B surface antigen) indicates an active infection.
12. A patient has lab work drawn and it shows a positive HBsAg. What education
will you provide to the patient?
A. Avoid sexual intercourse or intimacy such as kissing until blood work is
negative.
B. The patient is now recovered from a previous Hepatitis B infection and is now
immune.
C. The patient is not a candidate from antiviral or interferon medications.
D. The patient is less likely to develop a chronic infection.
The answer is A. A positive HBsAg (hepatitis B surface antigen) indicates an active
Hepatitis B infection. Therefore, the patient should avoid sexual intercourse and
other forms of intimacy until their HBsAg is negative.

13. A patient with Hepatitis A asks you about the treatment options for this
condition. Your response is?
A. Antiviral medications
B. Interferon
C. Supportive care
D. Hepatitis A vaccine
The answer is C. There is no current treatment for Hepatitis A but supportive care
and rest. Treatments for the other types of Hepatitis such as B, C, and D include
antiviral or interferon (mainly the chronic cases) along with rest.

14. A patient was exposed to Hepatitis B recently. Postexposure precautions


include vaccination and administration of HBIg (Hepatitis B Immune globulin). HBIg
needs to be given as soon as possible, preferably ___________ after exposure to be
effective.
A. 2 weeks
B. 24 hours
C. 1 month
D. 7 days
The answer is B. HBIg should be given 24 hours after exposure to maximum
effectiveness of temporary immunity against Hepatitis B. It would be given within 12
hours after birth to an infant born to a mother who has Hepatitis B.

15. You’re providing education to a patient with an active Hepatitis B infection. What
will you include in their discharge instructions? Select all that apply:
A. “Take acetaminophen as needed for pain.”
B. “Eat large meals that are spread out through the day.”
C. “Follow a diet low in fat and high in carbs.”
D. “Do not share toothbrushes, razors, utensils, drinking cups, or any other type
of personal hygiene product.”
E. “Perform aerobic exercises daily to maintain strength.”
The answers are C and D. The patient should NOT take acetaminophen (Tylenol)
due to its effective on the liver. The patient should eat small (NOT large), but
frequent meals…this may help with the nausea. The patient should rest (not perform
aerobic exercises daily) because this will help with liver regeneration.

16. What is the MOST common transmission route of Hepatitis C?


A. Blood transfusion
B. Sharps injury
C. Long-term dialysis
D. IV drug use
The answer is D. IV drug use is the MOST common transmission route of Hepatitis
C.

17. A patient is diagnosed with Hepatitis D. What statement is true about this type
of viral Hepatitis? Select all that apply:
A. The patient will also have the Hepatitis B virus.
B. Hepatitis D is most common in Southern and Eastern Europe, Mediterranean,
and Middle East.
C. Prevention of Hepatitis D includes handwashing and the Hepatitis D vaccine.
D. Hepatitis D is most commonly transmitted via the fecal-oral route.
The answers are A and B. These are true statements about Hepatitis D. Prevention
for Hepatitis D includes handwashing and the Hepatitis B vaccine (since it occurs
only with the Hepatitis B virus). It is transmitted via blood.

18. Select all the signs and symptoms associated with Hepatitis?
A. Arthralgia
B. Bilirubin 1 mg/dL
C. Ammonia 15 mcg/dL
D. Dark urine
E. Vision changes
F. Yellowing of the sclera
G. Fever
H. Loss of appetite
The answers are A, D, F, G, and H. The bilirubin and ammonia levels are normal in
these options, but they would be abnormal in Hepatitis. A normal bilirubin is 1 or
less, and a normal ammonia is 15-45 mcg/dL.

19. A patient with Hepatitis has a bilirubin of 6 mg/dL. What findings would correlate
with this lab result? Select all that apply:
A. None because this bilirubin level is normal
B. Yellowing of the skin and sclera
C. Clay-colored stools
D. Bluish discoloration on the flanks of the abdomen
E. Dark urine
F. Mental status changes
The answers are B, C, and E. This is associated with a high bilirubin level. A normal
bilirubin level is 1 or less.

20. A patient with Hepatitis is extremely confused. The patient is diagnosed with
Hepatic Encephalopathy. What lab result would correlate with this mental status
change?
A. Ammonia 100 mcg/dL
B. Bilirubin 7 mg/dL
C. ALT 56 U/L
D. AST 10 U/L
The answer is A. When ammonia levels become high (normal 15-45 mcg/dL) it
affects brain function. Therefore, the nurse would see mental status changes in a
patient with this ammonia level.

21. The physician writes an order for the administration of Lactulose. What lab
result indicates this medication was successful?
A. Bilirubin <1 mg/dL
B. ALT 8 U/L
C. Ammonia 16 mcg/dL
D. AST 10 U/L
The answer is C. Lactulose is ordered to decrease a high ammonia level. It will
cause excretion of ammonia via the stool. A normal ammonia level would indicate
the medication was successful (normal ammonia level 15-45 mcg/dL).

22. How is Hepatitis E transmitted?


A. Fecal-oral
B. Percutaneous
C. Mucosal
D. Body fluids
The answer is A.

23. Which patient below is at MOST risk for developing a complication related to a
Hepatitis E infection?
A. A 45-year-old male with diabetes.
B. A 26-year-old female in the 3rd trimester of pregnancy.
C. A 12-year-old female with a ventricle septal defect.
D. A 63-year-old male with cardiovascular disease.
The answer is B. Patients who are in the 3rd trimester of pregnancy are at a HIGH
risk of developing a complication related to a Hepatitis E infection.

24. What is the BEST preventive measure to take to help prevent ALL types of viral
Hepatitis?
A. Vaccination
B. Proper disposal of needles
C. Hand hygiene
D. Blood and organ donation screening
The answer is C. Hand hygiene can help prevent all types of viral hepatitis. However,
not all types of viral Hepatitis have a vaccine available or are spread through needle
sticks or blood/organs donations. Remember Hepatitis A and E are spread only via
fecal-oral routes.

25. Select all the types of viral Hepatitis that have preventive vaccines available in
the United States?
A. Hepatitis A
B. Hepatitis B
C. Hepatitis C
D. Hepatitis D
E. Hepatitis E
The answers are A and B. Currently there is only a vaccine for Hepatitis A and B in
the U.S.

26. A patient is prescribed Peginterferon alfa-2a. The nurse will prepare to


administer this medication. What route?
A. Oral
B. Intramuscular
C. Subcutaneous
D. Intravenous
The answer is C. This medication is administered subq.

27. A patient with viral Hepatitis states their flu-like symptoms have subsided.
However, they now have yellowing of the skin and sclera along with dark urine.
Based on this finding, what is the phase of Hepatitis?
A. Icteric
B. Posticteric
C. Preicteric
D. Convalescent
The answer is A. The Preicteric (prodromal) Phase: flulike symptoms…joint pain,
fatigue, nausea vomiting, abdominal pain change in taste, liver enzymes and bilirubin
increasing….Icteric Phase: decrease in the flu-like symptoms but will have jaundice
and dark urine (buildup of bilirubin) yellowing of skin and white part of the eyeball,
clay-colored stool (bilirubin not going to stool to give it’s normal brown color)
enlarged liver and pain in this area….Posticteric (convalescent) Phase: jaundice and
dark urine start to subside and stool returns to normal brown color, liver enzymes
and bilirubin decrease to normal

28. During the posticteric phase of Hepatitis the nurse would expect to find? Select
all that apply:
A. Increased ALT and AST levels along with an increased bilirubin level
B. Decreased liver enzymes and bilirubin level
C. Flu-like symptoms
D. Resolved jaundice and dark urine
The answers are B and D. Posticteric (convalescent) Phase: jaundice and dark urine
start to subside and stool returns to normal brown color, liver enzymes and bilirubin
decrease to normal

HIV/AIDS
1. The Human Immunodeficiency Virus (HIV) mainly attacks what type of cells in the
human body?
A. Red Blood Cells
B. CD4 positive cells
C. Stem Cells
D. Platelets
The answer is B. The HIV virus attacks the human body’s immune system,
specifically the CD4 positive cells…mainly the helper t cells. These cells are white
blood cells that help the immune system fight infection.

2. The Human Immunodeficiency Virus (HIV) can NOT be spread in what type of
fluid below? Select all that apply:
A. Breastmilk
B. Blood
C. Tears
D. Semen
E. Vaginal Fluid
F. Sweat
The answers are C and F. HIV can NOT be spread in tears or sweat (unless blood is
present which rarely occurs). HIV is spread in the following fluids: breastmilk, blood,
semen, and vaginal fluid.

3. Which statement below is not true regarding the role of the helper t cell?
A. The helper T cell releases cytokines to help activate other immune system cells.
B. The helper T cell is part of the adaptive immune system.
C. The helper T cell is cytotoxic and kills invaders.
D. The helper T cell has CD4 receptors found on its surface.
The answer is C. All the other options are true statements about the helper T cell.
Option C is NOT true. Helper T cells are not cytotoxic and kill invaders (this is
another type of t cells called cytotoxic t cell). Helper T cells are “helpers” in that they
HELP the immune system by releasing cytokines, which help activate other immune
system cells.
4. Identify the correct statements about the anatomy of the Human
Immunodeficiency Virus (HIV). Select all that apply:
A. HIV is a retrovirus.
B. Inside the virus is packaged DNA.
C. The protein projections found on the virus’ surface play a key role in attaching to
the receptors on the helper t-cell.
D. The glycoproteins (specifically GP140) are vital for engaging the receptors on the
targeted cell.
The answers are A and C. Options B is an incorrect statement, it should say “Inside
the virus is packaged RNA (not DNA)”, and Option C is incorrect because it should
say “The glycoproteins (specifically GP120…NOT GP140) are vital for engaging the
CD4 receptor on the targeted cell.

5. The first step in the “Life Cycle of HIV” is the attachment step. During this step
what co-receptors are also targeted so entry can be gained inside the cell?
A. CCR5 or CXCR4
B. CD4 or CD2
C. CCR4 or CXCR5
D. CD8 or CD5
The answer is A: CCR5 or CXCR4. Attachment occurs when the glycoproteins
(GP120) projections make contact and bind with a CD4 receptor. In addition, there is
also binding with certain co-receptors called CCR5 or CXCR4 to gain entry into the
cell.

6. What step in the ““Life Cycle of HIV” does the Human Immunodeficiency Virus
(HIV) become united with the targeted cell and dumps its contents into that cell?
A. Integration
B. Fusion
C. Budding
D. Assembly
The answer is B: Fusion.

7. Which statement below best describes the role of reverse transcriptase?


A. It’s an enzyme that helps cut up the long protein chains of HIV.
B. It plays a key role in the maturity of the virus.
C. Reverse transcriptase is an enzyme that turns viral RNA into viral DNA.
D. Reverse transcriptase is an enzyme that allows the viral DNA to become part of
the cell’s DNA.
The answer is C: Reverse transcriptase is an enzyme that turns viral RNA into viral
DNA.

8. Fill in the blank: Once inside the nucleus of the cell, the Human Immunodeficiency
Virus (HIV) needs to become part of the cell’s DNA. To do this, the virus releases an
enzyme called __________________.
A. Protease
B. Reverse transcriptase
C. Kinase
D. Integrase

9. What enzyme is responsible for cutting long chains of virus to help prep the HIV
for maturity?
A. Integrase
B. Protease
C. Reverse transcriptase
D. Kinase

10. A 30-year-old patient is in the Acute Stage of HIV. What findings below correlate
with this stage of HIV? Select all that apply:
A. CD4 level <500 cells/mm3
B. No present of Opportunistic Infections
C. High viral load
D. Patient reports flu-like symptoms
E. Patient is asymptomatic
The answers are B, C, and D. The Acute Stage of HIV is the first stage and tends to
occur a couple of weeks to a month after becoming infected. The patient’s viral load
is very HIGH during this time, but the CD4 count should be greater than 500
cells/mm3. Therefore, NO opportunistic infections are present during this time (the
CD4 count is high enough to fight off these types of infections/diseases). In addition,
the patient may report flu-like symptoms (aches, joint pain, headache, fever, fatigue,
sore throat, swollen lymph nodes, GI upset, and rash). The patient is usually
asymptomatic in the 2nd stage (Chronic Stage of HIV).

11. A patient arrives at the clinic and requests an HIV test. The patient had
unprotected sexual intercourse 2 days ago with a person who may have HIV. As the
nurse you know there is a window period for detecting an infection of HIV. What
statements should you provide to the patient about this window period and testing for
HIV? Select all that apply:
A. No test is available at this time to show immediate infection.
B. The window period is the time when you become infected with HIV to when a test
can deliver positive results.
C. Window periods vary depending on the type of HIV test administered.
D. The absolute earliest an HIV test can detect HIV is about 3 months.
The answers are A, B, C. These are correct statement. Option D is incorrect
because the NAT (nucleic acid test) can detect HIV the earliest of all the test types. It
can detect around 10 days after exposure for some patients. It assesses for the
virus’ genetic material and measures the amount of virus present in the blood. It is
not commonly ordered and is used only for high risk patients.
12. The physician orders a combination HIV antigen/antibody test on a patient. The
patient was potentially exposed to HIV 3 weeks ago. What HIV antigen does this test
assess for?
A. GP120
B. p24
C. GP41
D. P35

13. Which HIV test can give the earliest test results?
A. Nucleic Acid Test (NAT)
B. Antibody HIV Test
C. Combination HIV antigen/antibody test
D. CD4 count

14. A patient, who is in the Chronic Stage of HIV, has a CD4 count ordered. What
does this test measure?
A. Red blood cells
B. B cells
C. Cytotoxic T cells
D. Helper T cells

15. What is a normal CD4 count?


A. 200-500 cells/mm3
B. 1500-3500 cells/mm3
C. 500-1500 cells/mm3
D. <200 cells/mm3

16. A 48-year-old patient is HIV positive. The patient has no signs and symptoms
and has a CD4 count of 400 cells/mm3. In addition, no opportunistic infections or
diseases are present. These findings correlate with what stage of HIV?
A. Acute
B. Chronic
C. AIDS
The answer is B: Chronic. These findings correlate with the Chronic Stage (also
called the Asymptomatic Stage) of HIV. Signs and symptoms may not be
experienced, the viral load is lower than the Acute Stage, but the virus is still
replicating and destroying the cells. The patient can still transmit the virus to others.
In addition, the CD4 count should be more than 200 cells/mm3 to about 500
cells/mm3. In addition, no opportunistic infections or diseases should be present.

17. Select the criteria below that is used to help diagnosed a patient with Acquired
Immunodeficiency Syndrome (AIDS):
A. CD4 count <200 cells/mm3
B. Presence of opportunistic infection
C. CD4 count >1500 cells/mm3
D. WBC 9500
E. Absence of opportunistic infection
The answers are A and B. A patient is diagnosed with AIDS if: CD4 count drops to
less than 200 cells/mm3 or an opportunistic infection is present

18. A patient with AIDS has dark purplish brown lesions on the mucous membranes
of the mouth. As the nurse you know these lesions correlate with what type of
opportunistic disease?
A. Epstein-Barr Virus
B. Herpes Simplex Virus
C. Cytomegalovirus
D. Kaposi’s Sarcoma

19. What sign and symptoms in your patient with HIV indicates the disease is
worsening and the immune system is severely compromised?
A. Open, oozing lesions around the mouth
B. White hair like spots on the side of the tongue
C. Cheesy white film on the tonsils and inside cheeks
D. Vision changes
The answer is B. This is known as oral hairy leukoplakia. It occurs when the immune
system is extremely compromised like with HIV and the Epstein-Barr virus. It is a
signal that HIV is getting worse.

20. A patient with AIDS has developed CMV (cytomegalovirus). The nurse makes it
a priority to educate the patient about which of the following regarding CMV?
A. Avoiding exposure to cat feces.
B. Drinking water from clean sources only.
C. Scheduling an eye appointment.
D. Isolating for 14 days to prevent transmission of CMV to others.
The answer is C. CMV is an opportunistic infection that can develop in patients with
AIDS. This virus can cause damage to the retina (hence retinitis) and lead to
blindness. The nurse should educate the patient about scheduling an eye exam to
monitor for this condition.

21. Your patient is in the last stage of HIV. The patient CD4 count is 100 cells/mm3.
Which of the following FUNGAL infections is your patient at risk for? Select all that
apply:
A. Mycobacterium tuberculosis
B. Candidiasis
C. Coccidioidomycosis
D. Toxoplasmosis
E. Histoplasmosis
The answers are B, C, and E. Candidiasis, Coccidioidomycosis, Histoplasmosis are
all FUNGAL infections. Mycobacterium tuberculosis is a bacterial infection, and
Toxoplasmosis is a protozoal (parasitic type) infection.

22. Which type of opportunistic infection occurs from inhaling a parasitic organism
that can be found in cat and bird feces?
A. Histoplasmosis
B. Crytococcosis
C. Mycobacterium Avium complex
D. Toxoplasmosis
The answer is D. Toxoplasmosis is a protozoal parasitic infection that is found in cat
and bird feces along with undercooked meat, such as pork and red meat.

23. You’re providing education to a patient with AIDS on how to prevent opportunistic
infections. Which statement below requires the nurse to re-educate the patient about
this topic?
A. “I’m traveling to Puerto Rico next week and will be sure to pack bottled water.”
B. “I’ve switched to buying raw organic milk.”
C. “Last month I received the Pneumovax.”
D. “My neighbor bought a cat last week.”
The answer is B. Patients with AIDS should AVOID raw, undercooked, and
unpasteurized foods and beverages. These items could contain organisms that could
cause serious infections to a person with a compromised immune system.

24. The nurse plays a vital role in screening patients for a possible HIV infection.
What questions below could the nurse ask to help identify a patient who is at risk for
HIV? Select all that apply:
A. “How often do you use alcohol or drugs?”
B. “Have you recently experienced an abusive relationship?”
C. “If you are sexually active, do you or your partner use protection?”
D. “In the past month, have you felt sad or unable to get out of bed?”
E. “Have you ever been treated for a sexually transmitted infection?”
F. “Do you ever experience intrusive or unwanted thoughts?”
The answers are A, C, E. The nurse should screen patients for HIV. Questions about
sexual behavior (if the patient is sexually active, how often, treatment for STI, barrier
devices used, and number of partners), drug/alcohol usage, and if they’ve had a
blood transfusion before 1985, can all help the nurse identify patients who may need
to undergo HIV testing.

25. TRUE OR FALSE: The Centre for Disease Control and Prevention (CDC)
recommends that all people between the ages of 13-64 be tested at least once for
HIV during a routine health visit, regardless of risk factors.
The answer is TRUE.
26. A 25-year-old patient reports that they engage in high risk activities that could
lead to an HIV infection. The patient’s test results show the patient is HIV-negative.
The nurse should provide the patient with education about?
A. PEP
B. PrEP
C. Opportunistic Infections
D. Nucleic Acid Test (NAT)
The answer is B. The nurse should provide the patient with education about PrEP
(Pre-Exposure Prophylaxis). These medications are taken BEFORE a possible
encounter with HIV, which helps prevent a possible HIV infection. The patient must
be HIV-negative before taking these medications.

27. Which patient below is a candidate for PEP (Post-exposure Prophylaxis)?


A. A 32-year-old patient who reports sharing IV drug injection devices with a person
who is HIV-positive 5 days ago.
B. A 28-year-old patient who engages in high risk activities on a regular basis that
could lead to an HIV infection.
C. A 55-year-old who is HIV-positive.
D. A 30-year-old who was sexually assaulted two days ago.
The answer is D. PEP (post-exposure prophylaxis) is medication that is taken
AFTER an encounter with an HIV infected person. These medications can help
prevent becoming infected with HIV, if started within 72 HOURS of the exposure
(option A is not a candidate). These medications are NOT for routine usage but for
emergencies (sexual assault, needle stick etc.). If a patient is at high risk (as with the
patient in option B), they should consider PrEP (pre-exposure prophylaxis). PEP is
taken for 28 days.

28. A 25-year-old female is about to deliver a baby. The patient is HIV-positive and
has been taking antiretroviral therapy during the pregnancy. What steps can be
taken to help prevent transmitting the virus to the baby after birth?
A. Substitute formula for breastfeeding.
B. Administer antiretroviral treatment to the newborn for 2 weeks after birth.
C. Avoid kissing and hugging the newborn.
D. Stop taking antiretroviral therapy for 2 months postpartum.
The answer is A. Breast Milk can transmit the HIV virus. The patient should avoid
breastfeeding but use formula instead. Option B is wrong because the newborn
should receive antiretroviral treatment for 4-6 weeks after birth (NOT 2 weeks). The
mother can kiss and hug her newborn (this does NOT transmit the virus), and the
patient should not quit taking antiretroviral therapy during the postpartum period.

29. Which statement below is not a true statement about Antiretroviral Treatment?
A. “The patient starts out taking 3 medications from at least 2 drug classes.”
B. “ART decreases the amount of virus in the blood within about 6 months.”
C. “ART helps decrease the risk of developing an opportunistic infection.”
D. “Antiretroviral medications are taken when signs and symptoms appear and then
tapered off.”
The answer is D: This option is false and all the others are true regarding ART. ART
must be taken EXACTLY as prescribed (everyday, at the same time, at the right
dosage etc.). It is not taken when signs and symptoms appear and tapered off. If
ART is not taken as prescribed or doses are missed, drug resistance can develop. In
other words, the medications will stop being effective against HIV.

30. Which class of antiretroviral medications bind with the CD4 receptors and inhibit
the HIV’s glycoprotein from being able to activate and engage the co-receptors?
A. Post-attachment Inhibitors
B. Fusion Inhibitors
C. Integrase Inhibitors
D. Non-nucleoside Reverse Transcriptase Inhibitors
The answer is A: Post-attachment Inhibitors. Trogarzo (ibalizumab) is a type of
post-attachment inhibitor. It is given IV every 2 weeks.

31. Fill-in-the-blank: Chemokine Receptor Antagonists block the co-receptor


__________ on the cell so HIV cannot engage the receptor and enter the cell.
A. CXCR4
B. CD4
C. CCR5
D. GP120

32. The patient is prescribed Enfuvirtide (Fuzeon). The nurse prepares to administer
this medication via?
A. Intravenous route
B. Oral route
C. Topical Route
D. Subcutaneous Route

33. Which antiretroviral medication treatment stops the enzyme reverse transcriptase
from working by binding to it?
A. Non-nucleoside Reverse Transcriptase Inhibitors (NNRTIs)
B. Integrase Inhibitors
C. Protease Inhibitors
D. Fusion Inhibitors

34. A patient with Stage 2 HIV (Chronic Stage) is prescribed to take Raltegravir. The
nurse knows that this medication is part of what class of ART?
A. Protease Inhibitors
B. Integrase Inhibitors
C. Nucleoside Reverse Transcriptase Inhibitors
D. Attachment Inhibitors
The answer is B. Raltegravir is an Integrase Inhibitor. Remember from the lecture:
“tegra” in Integrase….the meds in this class have “tegra” in the middle…like
RalTEGRAvir.

35. What group of ART for the treatment of HIV stops an enzyme from cutting the
long chains of virus so the immature virus can’t be assembled and mature?
A. Protease Inhibitors
B. Integrase Inhibitors
C. Nucleoside Reverse Transcriptase Inhibitors
D. Attachment Inhibitors

36. A patient with HIV is prescribed to start antiretroviral therapy. The nurse is
providing education about these medications. Which statement below by the patient
indicates they need re-education on these medications?
A. “If I take these medications as prescribed, the viral load will become undetectable,
and I have a low risk of transmitting the virus to others.”
B. “Drug resistance is likely to develop if I’m non-compliant with my medications.”
C. “I currently take a medication called St. John’s Wort to treat depression.”
D. “This therapy does not cure me from HIV but helps me live a healthier and longer
life.”
The answer is C. Patients who take ART should be educated about how these
medications can interact with over-the-counter medications, especially herbal
supplements like St. John’s Wort. This medication is used to treat depression. The
patient should be re-educated about this topic.

APPENDICITIS
A client telephones the health clinic with complaints of generalized abdominal pain
which is aggravated by moving or walking. The client has not been able to eat for a
day and is nauseated. Which advice should the nurse provide to this​client?
A. "Take a warm shower and apply a heating pad to the​abdomen."
B. "Rest in bed and drink warm​fluids."
C. "Seek immediate medical​attention."
D. "Take an​over-the-counter laxative."

Which clinical manifestation does the nurse expect with acute​appendicitis?


A. High fever
B. Nausea and vomiting
C. Rebound tenderness
D. Pain relieved with ambulation

Which condition may occur if the client does not seek medication attention for acute
appendicitis within 24-36 h
​ ours? (Select all that​apply.)
A. Seizure
B. Constipation
C. Nausea
D. Peritonitis
E. Perforation
​Rationale: If treatment is not​initiated, tissue necrosis and gangrene result within
24-36 ​hours, leading to perforation​(rupture). Perforation allows the contents of the
gastrointestinal​(GI) tract to flow into the peritoneal space of the​abdomen, resulting
in peritonitis. Appendicitis does not cause​seizures, nausea, or constipation.

A teenage boy presents with suspected appendicitis. The caregiver​asks, "Why did
my son get​this?" Which response by the nurse is the most​appropriate?
A. "Your son has been eating too much​fiber."
B. "Your son is eating too many fruits and​vegetables."
C. "Your son has not been getting enough​exercise."
D. "Your adolescent son is in a risk​group."
​Rationale: Adolescent boys are at greatest risk for appendicitis. Appendicitis cannot
be​prevented, but certain dietary habits may reduce the risk of developing this
condition. Eating foods that contain high fiber​content, such as fresh fruits and​
vegetables, decreases the incidence of appendicitis.

Appendicitis almost always results from an obstruction in the appendiceal lumen.


Which problem should the nurse identify as the cause of this​obstruction?
A. Monolith
B. Fecalith
C. Tonsillolith
D. Ptyalith

Which statement by a client diagnosed with acute appendicitis leads the nurse to
believe the client needs teaching about dietary​interventions?
A. ​"I eat raw vegetables for a snack several days per​week."
B. ​"I don't like fruits and​vegetables."
C. ​"I prefer to have meat with each​meal."
D. "I eat fruit with breakfast every​day."

Which assessment finding leads the nurse to suspect that an older client may have​
appendicitis? (Select all that​apply.)
A. Pain migrating from the lower left to the upper right quadrant
B. Tenderness when pressing McBurney point
C. Confusion
D. No abdominal pain
E. Internal rotation of the left hip increases pain
Answer: B, C, E
Rationale: Fewer than​30% of older adults who have appendicitis present with
classic symptoms. Classic signs of acute appendicitis are pain that is aggravated by
moving or​walking, rebound tenderness of McBurney​point, and extension or internal
rotation of the right hip that increases pain and confusion.

A teenage girl is being assessed for the possibility of appendicitis. Which other
condition should the nurse​consider? (Select all that​apply.)
A. Pelvic inflammatory disease
B. Ovulation
C. Menstruation
D. Urinary tract infection
E. Ruptured ectopic pregnancy

Appendicitis in a pregnant woman is a complex problem. Which statement is true


based on the given​premise?
A. Appendicitis is the most common surgical presentation in pregnant women.
B. Appendicitis does not occur in pregnant women.
C. Appendicitis will cause fetal death.
D. A pregnant woman will have surgery postpartum.

A client presents with suspected appendicitis. The nurse should prepare the client for
which collaborative​intervention?
A. Chest​x-ray
B. Abdominal ultrasound
C. Electrolytes
D. Complete blood count​(CBC)

The nurse would increase the comfort of the patient with appendicitis by:
"a. Having the patient lie prone
b. Flexing the patient's right knee
c. Sitting the patient upright in a chair
d. Turning the patient onto his or her left side
The patient with appendicitis usually prefers to lie still, often with the right leg flexed
to decrease pain.

"The nurse is caring for a patient in the emergency department with complaints of
acute abdominal pain, nausea, and vomiting. When the nurse palpates the patient's
left lower abdominal quadrant, the patient complains of pain in the right lower
quadrant. The nurse will document this as which of the following diagnostic signs of
appendicitis?
a. Rovsing sign
b. referred pain
c. Chvostek's sign
d. rebound tenderness
In patients with suspected appendicitis, Rovsing sign may be elicited by palpation of
the left lower quadrant, causing pain to be felt in the right lower quadrant.
Which of the following positions should the client with appendicitis assume to relieve
pain ?
A. Prone B. Sitting C. Supine D. Lying with legs drawn up
Lying still with legs drawn up towards the chest helps relieve tension on the
abdominal muscle, which helps to reduce the amount of discomfort felt. Lying flat or
sitting may increase the amount of pain experienced

"When evaluating a male client for complications of acute pancreatitis, the nurse
would observe for:
a. increased intracranial pressure.
b. decreased urine output.
c. bradycardia.
d. Hypertension

"When preparing a male client, age 51, for surgery to treat appendicitis, the nurse
formulates a nursing diagnosis of Risk for infection related to inflammation,
perforation, and surgery. What is the rationale for choosing this nursing diagnosis?
a. Obstruction of the appendix may increase venous drainage and cause the
appendix to rupture.
b. Obstruction of the appendix reduces arterial flow, leading to ischemia,
inflammation, and rupture of the appendix.
c. The appendix may develop gangrene and rupture, especially in a middle-aged
client.
d. Infection of the appendix diminishes necrotic arterial blood flow and increases
venous drainage.

A client is admitted with a diagnosis of acute appendicitis. When assessing the


abdomen, the nurse would expect to find rebound tenderness at which location?
a) Left lower quadrant
b) Left upper quadrant
c) Right upper quadrant
d) Right lower quadrant

The nurse is monitoring a client diagnosed with appendicitis who is scheduled for
surgery in 2 hours. The client begins to complain of increased abdominal pain and
begins to vomit. On assessment, the nurse notes that the abdomen is distended and
bowel sounds are diminished. Which is the appropriate nursing intervention?
1. Notify the Physician
2. Administer the prescribed pain medication
3. Call and ask the operating room team to perform the surgery as soon as possible
4. Reposition the client and apply a heating pad on warm setting to the client's
abdomen
A client is admitted with right lower quadrant pain, anorexia, nausea, low-grade
fever, and elevated white blood cell count. Which complication is most likely the
cause? 1. fecalith 2. Bowel Kinking 3. Internal bowel occlusion 4. Abdominal wall
swelling

A client with acute appendicitis develops a fever, tachycardia, and hypotension.


Based on these assessment findings, the nurse should further assess the client for
which of the following complications?
1. Deficient fluid volume.
2. Intestinal obstruction.
3. Bowel ischemia.
4. Peritonitis
"Complications of acute appendicitis are perforation, peritonitis, and abscess
development. Signs of the development of peritonitis include abdominal pain and
distention, tachycardia, tachypnea, nausea, vomiting, and fever. Because peritonitis
can cause hypovolemic shock, hypotension can develop.

The client diagnosed with appendicitis has undergone an appendectomy. At two


hours postoperative, the nurse takes the vital signs and notes T 102.6 F, P 132, R
26, and BP 92/46. Which interventions should the nurse implement? List in order of
priority.
1. Increase the IV rate.
2. Notify the health care provider.
3. Elevate the foot of the bed.
4. Check the abdominal dressing.
5. Determine if the IV antibiotics have been administered.
Order of priority: 1, 3, 4, 5, 2.

Select all the following options that are NOT causes of appendicitis:
A. Fecalith
B. Routine usage of NSAIDs
C. Infection due to Helicobacter pylori
D. Lymph node enlargement due to viral or bacterial infection
E. Diet low in fiber

A 23 year old patient is admitted with suspected appendicitis. The patient states he
is having pain around the umbilicus that extends into the lower part of his abdomen.
In addition, he says that the pain is worse on the right lower quadrant. The patient
points to his abdomen at a location which is about a one-third distance between the
anterior superior iliac spine and umbilicus. This area is known as what?
A. Rovsing’s Point
B. Hamman’s Point
C. McBurney’s Point
D. Murphy’s Point
Thinking back to the scenario in question 3, what other signs and symptoms are
associated with appendicitis. SELECT-ALL-THAT-APPLY:
A. Increased red blood Cells
B. Patient has the desire to be positioned in the prone position to relieve pain
C. Umbilical pain that extends in the right lower quadrant
D. Abdominal rebound tenderness
E. Abdominal Flaccidity

An 18 year old patient is admitted with appendicitis. Which statement by the patient
requires immediate nursing intervention?
A. “The pain hurts so much it is making me nauseous.”
B. “I have no appetite.”
C. “The pain seems to be gone now.”
D. “If I position myself on my right side, it makes the pain less intense.”

6. You’re providing education to a group of nursing students about the care of a


patient with appendicitis. Which statement by a nursing student requires
re-education about your teaching?
A. “After an appendectomy the patient may have a nasogastric tube to remove
stomach fluids and swallowed air.”
B. “Non-pharmacological techniques for a patient with appendicitis include
application of heat to the abdomen and the side-lying position.”
C. “The nurse should monitor the patient for signs and symptoms of peritonitis which
includes increased heart rate, respirations, temperature, abdominal distention, and
intense abdominal pain.”
D. “It is normal for some patients to have shoulder pain after a laparoscopic
appendectomy.”

Your patient is 4 days post-opt from an appendectomy. Which assessment finding


requires further evaluation?
A. The patient reports their last bowel movement was the day before surgery.
B. The patient reports incisional pain.
C. The patient coughs and deep breaths while splinting the abdominal incision.
D. Options A and C

A patient is recovering after having an appendectomy. The patient is 48 hours


post-opt from surgery and is tolerating full liquids. The physician orders for the
patient to try solid foods. What types of foods should the patient incorporate in their
diet?
A. Foods high in fiber
B. Foods low in fiber
C. Foods high in carbohydrates
D. Foods low in protein
A patient is scheduled for an appendectomy at noon. While performing your morning
assessment, you note that the patient has a fever of 103.8 ‘F and rates abdominal
pain 9 on 1-10. In addition, the abdomen is distended and the patient states, “I was
feeling better last night but it seems the pain has become worse.” The patient is
having tachycardia and tachypnea. Based on the scenario, what do you suspect the
patient is experiencing?
A. Pulmonary embolism
B. Colon Fistulae
C. Peritonitis
D. Hemorrhage

PANCREATITIS
1. Inside the pancreas are special cells that secrete digestive enzymes and
hormones. The cells that secrete digestive enzymes are known as ______________
cells.
A. Islet of Langerhans
B. Protease
C. Acinar
D. Amylase
Acinar cells secrete digestive enzymes such as amylase, protease, and lipase.

2. From the pancreas and gallbladder, the common bile duct and pancreatic duct
open into the ____________ where digestive enzymes and bile flow through the
duodenum via the major duodenal papilla which is surrounded by a muscular valve
that controls the release of digestive enzymes known as the ______________.
A. ampulla of vater, sphincter of Oddi
B. papilla of vater, sphincter of Oddi
C. minor duodenal papilla, ampulla of vater
D. jejunum, sphincter of pylori
Digestive enzymes from the pancreas duct and bile from the common bile duct flow
through the ampulla of vater (this is where the pancreatic duct and common bile duct
form together) into the duodenum via the major duodenal papilla (also called the
papilla of vater) which is surrounded by the sphincter of Oddi. This sphincter is a
muscular valve that controls the release of digestive enzymes/bile and prevents
reflux of stomach contents into the pancreas and bile duct.

3. Select-ALL-that-apply: In the pancreas, the acinar cells release:


A. Amylase
B. Somatostatin
C. Lipase
D. Protease
4. You’re caring for a 45 year old patient who is admitted with suspected acute
pancreatitis. The patient reports having extreme mid-epigastric pain that radiates to
the back. The patient states the pain started last night after eating fast food. As the
nurse, you know the two most common causes of acute pancreatitis are:
A. High cholesterol and alcohol abuse
B. History of diabetes and smoking
C. Pancreatic cancer and obesity
D. Gallstones and alcohol abuse

5. Which patient below is at MOST risk for CHRONIC pancreatitis?


A. A 25 year old female with a family history of gallstones.
B. A 35 year old male who reports social drinking of alcohol.
C. A 15 year old female with cystic fibrosis.
D. A 66 year old female with stomach cancer.

6. Your patient with acute pancreatitis is scheduled for a test that will use a scope to
assess the pancreas, bile ducts, and gallbladder. The patient asks you, “What is the
name of the test I’m going for later today?” You tell the patient it is called:
A. MRCP
B. ERCP
C. CT scan of the abdomen
D. EGD

7. A patient is admitted to the ER with the following signs and symptoms: very painful
mid-epigastric pain felt in the back, elevated glucose, fever, and vomiting. During the
head-to-toe assessment, you notice bluish discoloration around the belly button. As
the nurse, you know this is called?
A. Grey-Turner’s Sign
B. McBurney’s Sign
C. Homan’s Sign
D. Cullen’s Sign

8. While assisting a patient with chronic pancreatitis to the bathroom, you note the
patient’s stool to be oily/greasy in appearance. In your documentation you note this
as:
A. Steatorrhea
B. Melena
C. Currant
D. Hematochezia
Steatorrhea is an oily/greasy appearance of the stool which can occur in chronic
pancreatitis. This occurs due to the inability of the pancreas to produce digestive
enzymes which help break down fats. Melena is used to describe tarry/black stool,
hematochezia is used to describe red stools, and currants are jelly type stools.
9. A patient with acute pancreatitis is reporting excessive thirst, excessive voiding,
and blurred vision. As the nurse, is it your priority?
A. Reassure the patient this is normal with pancreatitis
B. Check the patient’s blood glucose
C. Assist the patient with drinking a simple sugar drink like orange juice
D. Provide a dark and calm environment
Patients with acute pancreatitis are at risk for hyperglycemia (the signs and
symptoms the patient are reporting are classic symptoms of hyperglycemia).
Remember the endocrine function of the pancreas (which is to release
insulin/glucagon etc. is insufficient) so the nurse must monitor the patient's blood
glucose levels even if the patient is not diabetic.

10. A patient who received treatment for pancreatitis is being discharged home.
You’re providing diet teaching to the patient. Which statement by the patient requires
immediate re-education about the diet restrictions?
A. “It will be hard but I will eat a diet low in fat and avoid greasy foods.”
B. “It is very important I limit my alcohol intake to no more than 2-3 glasses of wine a
week.”
C. “I will concentrate on eating complex carbohydrates rather than refined
carbohydrates.”
D. “I will purchase foods that are high in protein.”

11. The physician orders a patient with pancreatitis to take a pancreatic enzyme.
What assessment finding demonstrates the pancreatic enzymes are working
properly?
A. Abdominal girth is decreased
B. Skin turgor is less than 2 seconds
C. Blood glucose is 250
D. Stools appear formed and solid
Pancreatic enzymes help the body break down carbs, proteins, and fats because the
body is not sufficiently producing digestive enzymes anymore. Hence, the stool will
not appear as oily or greasy (decrease in steatorrhea) but appear solid and formed.

12. During a home health visit, you are assessing how a patient takes the prescribed
pancreatic enzyme. The patient is unable to swallow the capsule whole, so they
open the capsule and mix the beads inside the capsule with food/drink. Which food
or drink is safe for the patient to mix the beads with?
A. Pudding
B. Ice cream
C. Milk
D. Applesauce
The patient should mix the medications with acidic foods like applesauce. It is very
important the patient does NOT use alkaline foods for mixing (like dairy products,
pudding etc.) because they can damage the enzyme. Remember these enzymes
thrive in acidic conditions similar to your stomach.

A nurse is completing the admission assessment of a client who has acute


pancreatitis. Which finding is the first priority?
A) History of cholelithiasis
B) Elevated serum amylase levels
C) Decrease in bowel sounds upon auscultation
D) Hand spasms present when blood pressure is checked
Rationale: the greatest risk to the client is ECG changes and hypotension from
hypocalcemia. Hand spasms when taking blood pressure is a manifestation of
hypocalcemia

A nurse is completing an admission assessment of a client who has pancreatitis.


Which of the following findings should the nurse expect?
A. Pain in the UQ rating to the shoulder
B. Report of pain being worse when sitting upright
C. Pain relieved with defecation
D. Epigastric pain radiating to the left shoulder
Rationale: A client with pancreatitis will report pain being worse when lying down in
the fetal position, and pain that radiates to the back. left flank, or left, shoulder.

A nurse is assessing a client who has pancreatitis. Which of the following actions
should the nurse take to assess the presence of Cullen's sign.
A. Tap lightly at the costovertebral margin on the client's back
B. Palpate the RLQ
C. Inspect the skin around the umbilicus
D. Auscultate the area below the scapula
Rationale: Cullen's sign is indicated by a bluish-gray discoloration in the
periumbillical area

A nurse is completing nutritional teaching for a client who has pancreatitis. Which of
the following statements by the client indicates an understanding of the teaching?
(SATA)
A. I plan to eat small, frequent meals.
B. I will eat easy-to-digest foods with limited spice
C. I will use skim milk when cooking
D. I plan to drink regular cola
E. I will limit alcohol intake to two drinkers per day

The nurse is caring for a 55-yr-old man patient with acute pancreatitis resulting from
gallstones. Which clinical manifestation would the nurse expect?

A) Hematochezia
B) Left upper abdominal pain
C) Ascites and peripheral edema
D) Temperature over 102 F
Rationale: Abdominal pain (usually in the left upper quadrant) is the predominant
manifestation of acute pancreatitis.

Nursing management of the patient with acute pancreatitis includes: (SATA)


A. Check for signs of hypocalcemia
B. Provide a diet low in carbohydrates
C. Giving insulin based on sliding scale
D. Observing stools for signs of steatorrhea
E. Monitoring for infection, particularly respiratory tract infection

A patient with sudden pain in the left upper quadrant radiating to the back and
vomiting was diagnosed with acute pancreatitis. Which intervention should the nurse
include in the patient's plan of care?
A) Immediately start enteral feeding to prevent malnutrition.
B) Insert an NG and maintain NPO status to allow pancreas to rest.
C) Initiate early prophylactic antibiotic therapy to prevent infection.
D) Administer acetaminophen (Tylenol) every 4 hours for pain relief.

The client is admitted to the medical department with a diagnosis of R/O acute
pancreatitis. What laboratory values should the nurse monitor to confirm this
diagnosis?
A. Creatinine and BUN
B. Troponin and CK-MB
C. Serum amylase and lipase
D. Serum bilirubin and calcium

What client problem has a priority for the client diagnosed with acute pancreatitis?
A. Risk for fluid volume deficit
B. Alteration in comfort
C. Imbalanced nutrition: less than the body requires
D. Knowledge deficient
Rationale: Autodigestion of the pancreas results in severe pain, accompanied by
nausea, vomiting,abdominal tenderness, and muscle guarding. Maslow's hierarchy
should be applied. After ABC's pain is the next priority

The client is diagnosed with acute pancreatitis. What health-care provider's admitting
order should the nurse question?
A. Bedrest with bathroom privileges
B. Initiate IV therapy of D5W at 125 mL/hr
C. Weight client daily
D. Low fat, low carb diet
The nurse is preparing to administer A.M. medications to the following clients. Which
medication should the nurse question before administering?
1. Pancreatic enzymes to the client who has finished breakfast.
2. The pain medication, morphine, to the client who has a respiratory rate of 20.
3. The loop diuretic to the client who has a serum potassium level of 3.9 mEq/L.
4. The beta blocker to the client who has an apical pulse of 68 bpm.
1. Pancreatic enzymes must be administered with meals to enhance the digestion of
starches and fats in the gastrointestinal tract.

The nurse is completing discharge teaching to the client diagnosed with acute
pancreatitis. Which instruction should the nurse discuss with the client?
1. Instruct the client to decrease alcohol intake.
2. Explain the need to avoid all stress.
3. Discuss the importance of stopping smoking.
4. Teach the correct way to take pancreatic enzymes.
3. Smoking stimulates the pancreas to release pancreatic enzymes and should be
stopped.

The male client diagnosed with chronic pancreatitis calls and reports to the clinic
nurse that he has been having a lot of "gas," along with frothy and very foul-smelling
stools. Which action should the nurse take?
1. Explain that this is common for chronic pancreatitis.
2. Ask the client to bring in a stool specimen to the clinic.
3. Arrange an appointment with the HCP for today.
4. Discuss the need to decrease fat in the diet so that this won't happen.
Steatorrhea (fatty, frothy, foul-smelling stool) is caused by a decrease in pancreatic
enzyme secretion and indicates impaired digestion and possibly an increase in the
severity of the pancreatitis. The client should see the HCP.

The nurse is discussing complications of chronic pancreatitis with a client diagnosed


with the disease. Which complication should the nurse discuss with the client?
1. Diabetes insipidus.
2. Crohn's disease.
3. Narcotic addiction.
4. Peritonitis.
Narcotic addiction is related to the frequent, severe pain episodes that often occur
with chronic pancreatitis and its complications and that require narcotics for relief.

The client has just had an endoscopic retrograde cholangiopancreatogram (ERCP).


Which post-procedure intervention should the nurse implement?
1. Assess for rectal bleeding.
2. Increase fluid intake.
3. Assess gag reflex.
4. Keep in a supine position.
The gag reflex will be suppressed as a result of the local anesthesia applied to the
throat to insert the endoscope into the esophagus; therefore, the gag reflex must be
assessed prior to allowing the client to resume eating or drinking.

The client diagnosed with acute pancreatitis is in pain. Which position should the
nurse assist the client to assume to help decrease the pain?
1. Recommend lying in the prone position with legs extended.
2. Maintain a tripod position over the bedside table.
3. Place in a side-lying position with knees flexed.
4. Encourage a supine position
This fetal position decreases pain caused by stretching of the peritoneum as a result
of edema.

The client with an acute exacerbation of chronic pancreatitis has a nasogastric tube
and is NPO. Which interventions should the nurse implement? Select all that apply.
1. Monitor the bowel sounds.
2. Weigh the client daily.
3. Assess the intravenous site.
4. Provide oral and nasal care.
5. Monitor the blood glucose.
The return of bowel sounds indicates the return of peristalsis, and the nasogastric
suction is usually discontinued within 24 to 48 hours thereafter. Daily weight gain
reflects fluid gain. The nurse should assess for signs of infection or infiltration.
Fasting and the N/G tube increase the client's risk for mucous membrane irritation
and breakdown. Blood glucose levels are monitored because clients with chronic
pancreatitis can develop diabetes mellitus.with a pillow under the knees.

INFLAMMATORY BOWEL DISEASE


1. True or False: A patient with Crohn’s Disease can experience inflammation in the
large intestine that affects mainly the mucosa (inner layer) of the bowel.

2. A patient with Crohn’s Disease is most likely to have the disease. What part of the
GI tract?
A. Rectum
B. Duodenum of the small intestine
C. Terminal Ileum
D. Descending colon

3. You’re providing teaching to a patient who has been newly diagnosed with Crohn’s
Disease. Which statement by the patient’s spouse requires re-education?
A. “Crohn’s Disease can be scattered throughout the GI tract in patches with some
areas appearing healthy while others are diseased.”
B. “There is no cure for Crohn’s Disease.”
C. “Strictures are a common complication with Crohn’s Disease.”
D. “Crohn’s Disease can cause the haustra of the large intestine to lose its form.”

4. A physician is explaining to a patient that the patient has a type of Crohn’s


Disease that is found in both the ileum and colon. As the nurse, you know this type
of Crohn’s Disease is called?
A. Gastroduodenal Crohn’s Disease
B. Granulomatous Colitis
C. Ileitis
D. Ileocolitis

5. Select ALL of the following that are complications associated with Crohn’s
Disease:
A. Cobble-stone appearance of GI lining
B. Lead-pipe sign
C. Toxic megacolon
D. Fistula
E. Abscess
F. Anal Fissure

6. Your patient with Crohn’s Disease is admitted with an opening that has formed
between the bowel and bladder. As the nurse, you know what type of complication
associated with this disease?
A. Enterovesical Fistula
B. Rectovaginal Stricture
C. Enteroenteric Fistula
D. Perianal Fissure

7. A patient experiencing a flare-up with Crohn’s Disease is ordered complete bowel


rest by the physician. You are administering TPN (total parenteral nutrition) per
physician order. When developing the patient’s nursing plan of care, which nursing
diagnosis is MOST important to include in the care plan?
A. Risk for allergy response
B. Risk for unstable blood glucose level
C. Risk for imbalanced nutrition: more than body requirements
D. Risk for imbalanced nutrition: less than body requirements

8. A patient is receiving treatment for Crohn’s Disease. Which food found on the
patient’s food tray should the patient avoid?
A. Fresh Salad
B. White rice
C. Baked chicken
D. Cooked skinless apples
9. A physician has prescribed a patient with a severe case of Crohn’s Disease to
take a drug that works by suppressing the immune system. This medication achieves
this by blocking a protein that plays a role in the inflammatory process. Which drug
does this describe?
A. Azathioprine
B. Sulfasalazine
C. Infliximab
D. Prednisone

10. A patient with Crohn’s Disease is taking corticosteroids. The patient is


complaining of extreme thirst, polyuria, and blurred vision. What is your next nursing
action?
A. Check the patient’s blood glucose
B. Give the patient a food containing sugar (ex: orange juice)
C. Administer oxygen via nasal cannula
D. Assess bowel sounds

11. The nurse is questioning a client with Crohn disease about the presence of
current symptoms. Which symptom should the nurse expect to find in the​client?
(Select all that​apply.)
Left lower quadrant abdominal pain relieved by defecation
Blood in the stool
​Fissures, ulcers,​fistulas, and abscesses of the anorectal area
Persistent diarrhea
Malaise

12. The nurse is planning care for a female client with ulcerative colitis​(UC). Which
characteristic should the nurse​recognize? (Select all that​apply.)
Vomiting is the predominant manifestation.
It affects more women than men.
It may include arthritis involving one or several joints.
Mild to moderate symptoms includes six or fewer stools per day.
Left lower quadrant cramping is relieved by defecation.
Rationale: Clients with severe disease may have systemic manifestations such as
arthritis involving one or several​joints, skin and mucous membrane​lesions, or
uveitis​(inflammation of the​uvea, the vascular layer of the​eye, which may involve
the sclera and cornea as​well). Left lower quadrant cramping relieved by defecation
is common. Mild to moderate UC is characterised by six or fewer stools per​day,
intermittent rectal bleeding and​mucus, and few systemic manifestations. It is more
common in men than in​women, with an average age of diagnosis of 40 years.​
Diarrhea, not​vomiting, is the predominant manifestation of UC.
The nurse is planning care for a client in the early stages of ulcerative colitis. Which
part of the colon should the nurse understand to be initially affected by ulcerative​
colitis?
The rectosigmoid area
The transverse colon
The ileocecal junction
The duodenum
Ulcerative colitis begins with inflammation at the base of the crypts of​Lieberkühn in
the distal large intestine and rectal mucosa.​Microscopic, pinpoint mucosal
hemorrhages​occur, and crypt abscesses develop. These abscesses penetrate the
superficial submucosa and spread​laterally, leading to necrosis and sloughing of
bowel mucosa. Ulcerative colitis can progress to the entire​colon, stopping at the
ileocecal junction. The duodenum is part of the small intestine and is not affected by
ulcerative colitis. The transverse colon is not initially affected by ulcerative colitis.

The nurse is caring for a client in the early stages of Crohn disease. Which type of
lesion should the nurse recall that occurs at the beginning of this disease​process?
Aphthoid
Crypt abscess
Canker sore
Fistula
Aphthoid lesions are​small, inflammatory ulcers with a white base and elevated
margin. They have a similar appearance to a canker​sore, but they are not actually
cankers.

The nurse is preparing a community initiative to help reduce the rate of inflammatory
bowel disease​(IBD). On which behavior should the nurse​focus?
Increasing daily exercise habits
Eliminating alcohol use
Making lifestyle changes to a​low-fat diet
Smoking cessation
Rationale: Smoking cigarettes is a major risk factor for the development of IBD and
should be the behavior on which the nurse focuses.

A nurse is working at a​gastroenterologist's office. Which distinction between


inflammatory bowel diseases​(IBD) in the pediatric population should the nurse
understand as​important?
Children often present with fistulizing or structuring Crohn disease.
IBD is more common in boys than in girls.
Most pediatric clients have little colonic involvement with Crohn disease.
Surgery is not usually required in the pediatric population.
The nurse notes that a pediatric client with inflammatory bowel disease​(IBD) has
had poor growth since the last examination. Which suggestion should the nurse
make to the​client's parents?
​"Add daily probiotic​supplements."
​"Administer liquid dietary​supplements."
​"Increase fiber intake to add bulk to​stools."
​"Decrease fluid intake to reduce​diarrhea."
A child with IBD is at higher risk for poor growth and malabsorption. The nurse
should advise the parents to increase calorie and protein intake. Liquid dietary
supplements can be very effective to promote growth. Adding fiber or probiotics and
decreasing fluid intake will not promote growth and nutrition status in this child.

The nurse prepares teaching for the parents of a child with inflammatory bowel
disease​(IBD). Which mineral should the nurse encourage the parents to provide to
the child to prevent future​complications?
Magnesium
Sodium
Calcium
Potassium
Children with IBD are at higher risk for poor bone density as they get older. It is
important that they take in an adequate amount of calcium to promote bone health
and strength.​Magnesium, sodium, and potassium do not help to maintain strong
bones.

A newly pregnant client who has a history of inflammatory bowel disease​(IBD) asks
how the disease will impact their pregnancy. Which response by the nurse is
accurate​?
​"You will need to be on bedrest throughout the entire​pregnancy."
​"Your disease may increase the risk for some pregnancy​complications."
​"The medications used to treat your disease are safe to use during your​pregnancy."
​"IBD should have no effect on pregnancy or the baby.​We'll watch you​closely."
Rationale: Pregnant women with IBD are at higher risk for​preeclampsia, medically
indicated preterm​delivery, preterm premature rupture of​membranes, and delivering
infants with low APGAR scores. There is also a risk of some major congenital​
malformations, which are not completely explained by the medications used. It is not
necessary for a woman to be on complete bed rest throughout her entire pregnancy.
Not all drugs are safe for the pregnant woman.

The parent of a child with inflammatory bowel disease​(IBD) reports that the child is
refusing to​eat, causing the parent to become frustrated. Which advice should the
nurse provide to this​parent?
​"It is important for the child to eat. Make foods that they​like."
​"Be firm. The child will eat when​hungry."
​"I'll let the healthcare provider know about your​concerns."
​"I'll arrange for a consultation with a nutritionist to get more ideas for meal​time."

Which sign/symptoms should the nurse expect to find in a client diagnosed with
ulcerative colitis?
1. Twenty bloody stools a day
2. Oral temperature of 102 degrees Fahrenheit
3. Hard, rigid abdomen
4. Urinary stress incontinence

The client diagnosed with inflammatory bowel disease has a serum potassium level
of 3.4 mEq/L. Which action should the nurse implement first?
1. Notify the health-care provider
2. Assess the client for muscle weakness
3. Request telemetry for the client
4. Prepare to administer potassium IV

The client is diagnosed with an acute exacerbation of ulcerative colitis. Which


intervention should the nurse implement?
1. Provide a low-residue diet
2. Rest the client's bowels
3. Assess vital signs daily
4. Administer antacids orally

The client diagnosed with IBD is prescribed total parenteral nutrition (TPN). Which
intervention should the nurse implement?
1. Check the client's glucose level
2. Administer an oral hypoglycemic
3. Assess the peripheral intravenous site
4. Monitor the client's oral food intake

The client is diagnosed with an acute exacerbation of IBD. Which priority intervention
should the nurse implement first?
1. Weigh the client daily and document in the client's chart
2. Teach coping strategies such as dietary modifications
3. Record the frequency, amount, and color of stools
4. Monitor the client's oral fluid intake every shift

The client diagnosed with Crohn's disease is crying and tells the nurse "I can't take it
anymore. I never know when I will get sick and end up here in the hospital." Which
statement is the nurse's best response?
1. "I understand how frustrating this must be for you."
2. "You must keep thinking about the good things in your life."
3. "I can see you are very upset. I'll sit down and we can talk."
4. "Are you thinking about doing anything like committing suicide?"
The client diagnosed with ulcerative colitis has an ileostomy. Which statement
indicates the client needs more teaching concerning the ileostomy?
1. "My stoma should be pink and moist."
2. "I will irrigate my ileostomy every morning."
3. "If I get a red, bumpy, itchy rash I will call my HCP."
4. "I will change my pouch if it starts leaking."

The client diagnosed with IBD is prescribed sulfasalazine (Asulfidine), a sulfonamide


antibiotic. Which statement best describes the rationale for administering this
medication?
1. It is administered rectally to help decrease colon inflammation.
2. This medication slows gastrointestinal motility and reduces diarrhea.
3. This medication kills the bacteria causing the exacerbation.
4. It acts topically on the colon mucosa to decrease inflammation.

The client was diagnosed with Crohn's disease, also known as regional enteritis.
Which statement by the client supports the diagnosis?
1. "My pain goes away when I have a bowel movement."
2. "I have bright red blood in my stool all the time."
3. "I have episodes of diarrhea and constipation."
4. "My abdomen is hard and rigid and I have a fever."

CHOLECYSTITIS
1. The gallbladder is found on the __________ side of the body and is located under
the ____________. It stores __________.
A. right; pancreas; bilirubin
B. left; liver; bile
C. right; thymus’ bilirubin
D. right; liver; bile
The answer is D. The gallbladder is found in the RIGHT side of the body and is
located under the LIVER. It stores BILE.

2. Which statements below are CORRECT regarding the role of bile? Select all that
apply:
A. Bile is created and stored in the gallbladder.
B. Bile aids in digestion of fat soluble vitamins, such as A, D, E, and K.
C. Bile is released from the gallbladder into the duodenum.
D. Bile contains bilirubin.
The answers are B, C, and D. Option A is INCORRECT because bile is created in
the LIVER (not gallbladder), but bile is stored in the gallbladder.
3. You’re providing a community in-service about gastrointestinal disorders. During
your teaching about cholecystitis, you discuss how cholelithiasis can lead to this
condition. What are the risk factors for cholelithiasis that you will include in your
teaching to the participants? Select all that apply:
A. Being male
B. Underweight
C. Being female
D. Older age
E. Native American
F. Caucasian
G. Pregnant
H. Family History
I. Obesity
The answers are C, D, E, G, H and I. Cholelithiasis is the formation of gallstones.
Risk factors include: being female, older age (over 40), Native American or Mexican
American descent, pregnancy, obesity, and family history.

4. A patient is being transferred to your unit with acute cholecystitis. In the report the
transferring nurse tells you that the patient has a positive Murphy’s Sign. You know
that this means:
A. The patient stops breathing in when the examiner palpates under the ribs on the
right upper side of the abdomen at the midclavicular line.
B. The patient stops breathing out when the examiner palpates under the ribs on the
right upper side of the abdomen at the midclavicular line.
C. The patient verbalizes pain when the lower right quadrant is palpated.
D. The patient reports pain when pressure is applied to the right lower quadrant but
then reports an increase in pain intensity when the pressure is released.
The answer is A. Murphy’s Sign can occur with cholecystitis. This occurs when the
patient is placed in the supine position and the examiner palpates under the ribs on
the right upper side of the abdomen. The examiner will have the patient breathe out
and then take a deep breath in. The examiner will simultaneously (while the patient
is breathing in) palpate on this area under the ribs at the midclavicular line (hence
the location of the gallbladder). It is a POSITIVE Murphy’s Sign when the patient
stops breathing in during palpation due to pain.

5. Your patient is post-op day 3 from a cholecystectomy due to cholecystitis and has
a T-Tube. Which finding during your assessment of the T-Tube requires immediate
nursing intervention?
A. The drainage from the T-Tube is yellowish/green in color.
B. There is approximately 750 cc of drainage within the past 24 hours.
C. The drainage bag and tubing is at the patient’s waist.
D. The patient is in the Semi-Fowler’s position.
The answer is B. A T-Tube should not drain more than about 500 cc of drainage per
day (within 24 hours). A T-Tube’s drainage will go from blood tinged (fresh post-op)
to yellowish/green within 2-3 days. The drainage bag and tubing should be below the
site of insertion (at or below the patient’s waist so gravity can help drainage the bile),
and the patient should be in Semi-Fowler’s to Fowler’s position to help with draining
the bile.

6. The physician orders a patient’s T-Tube to be clamped 1 hour before and 1 hour
after meals. You clamp the T-Tube as prescribed. While the tube is clamped which
finding requires you to notify the physician?
A. The T-Tube is not draining.
B. The T-Tube tubing is below the patient’s waist.
C. The patient reports nausea and abdominal pain.
D. The patient’s stool is brown and formed.
The answer is C. A nurse should ONLY clamp a T-Tube with a physician’s order.
Most physicians will prescribe to clamp the T-tube 1 hour before and 1 hour after
meals. WHY? So, bile will flow down into the small intestine (instead out of the body)
during times when food is in the small intestine to help with the digestion of fats. This
is to help the small intestine adjust to the flow of bile (remember normally it received
bile when the gallbladder contracted but now it will flow from the liver to the small
intestine continuously). Option C is an abnormal finding. The patient should not
report nausea or abdominal pain when the tube is blocked. This could indicate a
serious problem. Option A is correct because the T-tube should not be draining
because it’s clamped. Option B is correct because the T-tube tubing should be below
or at the patient’s waist level. Option D is correct because this shows the body is
digesting fats and bilirubin is exiting the body through the stool (remember bilirubin is
found in the bile and gives stool its brown color…it would be light colored if the
bilirubin was not present). You would NOT want to see steatorrhea (fat/greasy liquid
stools) because this shows the bile isn’t being delivered to help digest the fats.

7. Your recent admission has acute cholecystitis. The patient is awaiting a


cholecystostomy. What signs and symptoms are associated with this condition?
Select all that apply:
A. Right lower quadrant pain with rebound tenderness
B. Negative Murphy’s Sign
C. Epigastric pain that radiates to the right scapula
D. Pain and fullness that increases after a greasy or spicy meal
E. Fever
F. Tachycardia
G. Nausea
The answers are C, D, E, F, and G. Option A and B are not associated with
cholecystitis, but a POSITIVE Murphy’s Sign is.

8. A patient in the emergency room has signs and symptoms associated with
cholecystitis. What testing do you anticipate the physician will order to help diagnose
cholecystitis? Select all that apply:
A. Lower GI series
B. Abdominal ultrasound
C. HIDA Scan (Hepatobiliary Iminodiacetic AciD scan)
D. Colonoscopy
The answers are B and C. These two tests can assess for cholecystitis. A lower GI
series would not assess the gallbladder but the lower portions of the GI system like
the rectum and large intestine. Option D is wrong because it would also assess the
lower portions of the GI system.

9. You’re precepting a nursing student who is helping you provide T-Tube drain care.
You explain to the nursing student that the t-shaped part of the drain is located in
what part of the biliary tract?
A. Cystic duct
B. Common hepatic duct
C. Common bile duct
D. Pancreatic duct
The answer is C. The “T-shaped” part of the drain is located in the common bile duct
and helps deliver bile to the duodenum (small intestine).

10. Your patient is unable to have a cholecystectomy for the treatment of


cholecystitis. Therefore, a cholecystostomy tube is placed to help treat the condition.
Which statement about a cholecystostomy (C-Tube) is TRUE?
A. The C-Tube is placed in the cystic duct of the gallbladder and helps drain infected
bile from the gallbladder.
B. Gallstones regularly drain out of the C-Tube, therefore, the nurse should flush the
tube regularly to ensure patency.
C. The C-Tube is placed through the abdominal wall and directly into the gallbladder
where it will drain infected bile from the gallbladder.
D. The tubing and drainage bag of the C-Tube should always be level with the
insertion site to ensure the tube is draining properly.
The answer is C. This is the only correct statement about a cholecystostomy. A
cholecystostomy, also sometimes called a C-Tube, is placed when a patient can’t
immediately have the gallbladder removed (cholecystectomy) due to cholecystitis. It
is placed through the abdominal wall and into the gallbladder. It will drain infected
bile (NOT gallstones). The tubing and drainage bag should be at or below waist level
so it drains properly.

11. A patient, who has recovered from cholecystitis, is being discharged home. What
meal options below are best for this patient?
A. Baked chicken with steamed carrots and rice
B. Broccoli and cheese casserole with gravy and mashed potatoes
C. Cheeseburger with fries
D. Fried chicken with a baked potato
The answer is A. The patient should eat a low-fat diet and avoid greasy/fatty/gassy
foods. Option B is wrong because this contains dairy/animal fat like the cheese and
gravy, and broccoli is known to cause gas. Option C and D are greasy food options.

12. Your patient is diagnosed with acute cholecystitis. The patient is extremely
nauseous. A nasogastric tube is inserted with GI decompression. The patient reports
a pain rating of 9 on 1-10 scale and states the pain radiates to the shoulder blade.
Select all the appropriate nursing interventions for the patient:
A. Encourage the patient to consume clear liquids.
B. Administered IV fluids per MD order.
C. Provide mouth care routinely.
D. Keep the patient NPO.
E. Administer analgesic as ordered.
F. Maintain low intermittent suction to the NG tube.
The answers are B, C, D, E, and F. The treatment for cholecystitis includes
managing pain, managing nausea/vomiting (a NG tube with GI decompression
(removal of stomach contents) to low intermittent suction may be ordered to help
severe cases), keep patient NPO until signs and symptoms subside, mouth care
from vomiting and nasogastric tube, and administer IV fluids to keep the patient
hydrated.

CYSTITIS
1. You’re assessing your patients during morning rounding. Which patient below is at
MOST risk for developing a urinary tract infection?
A. A 25 year old patient who finished a regime of antibiotics for strep throat 10 weeks
ago
B. A 55 year old female who is post-opt day 7 from hip surgery.
C. A 68 year old male who is experiencing nausea and vomiting.
D. A 87 year old female with Alzheimer’s disease who is experiencing bowel
incontinence.

2. A 36 year old female, who is 29 weeks pregnant, reports she is experiencing


burning when voiding. The physician orders a urinalysis. Which statement by the
patient demonstrates she understands how to collect the specimen?
A. “I’ll hold the cup firmly against the urethra while collecting the sample.”
B. “I will cleanse back to front with the antiseptic wipe before peeing in the cup.”
C. “First, I will pee a small amount of urine in the toilet and then collect the rest in the
cup.”
D. “I will be sure to drink a lot of fluids to keep the urine diluted before peeing into the
cup.”
3. During a head-to-toe assessment on a patient with a possible urinary tract
infection, you perform costovertebral angle percussion. The costovertebral angle is
found?
A. between the bottom of the 12th rib and spine
B. between the right upper quadrant and umbilicus
C. between the sternal notch and angle of Louis
D. between the ischial spine and umbilicus

4. A 76 year old female is admitted due to a recent fall. The patient is confused and
agitated. The family members report that this is not normal behavior for the patient.
They explain that the patient is very active in the community and cares for herself.
Based on the information you have gathered about the patient, which physician’s
order takes priority?
A. “Collect a urinalysis”
B. “Collect a T3 and T4 level”
C. “Insert a Foley Catheter”
D. “Keep patient NPO”

5. The physician orders a urine culture on your patient in room 5505 with a urinary
tract infection. In addition, the patient is ordered to start IV Bactrim
(Sulfamethoxazole/Trimethoprim). How will you proceed with following this order?
A. First, hang the antibiotic, and then collect the urine culture.
B. First, hang the antibiotic and when the antibiotic is finished infusing, collect the
urine culture.
C. First, collect the urine culture, and then hang the antibiotic.
D. First, collect the urine culture and then hold the dose of the antibiotic until the
urine culture is back from the lab.

6. A patient with a urinary tract infection is taking Bactrim


(Sulfamethoxazole/Trimethoprim). As the nurse you know it is important that the
patient consumes 2.5 to 3 L of fluid per day to prevent any of the following
complications?
A. Brown urine
B. Crystalluria
C. Renal Stenosis
D. Renal Calculi

7. You’re providing discharge teaching to a female patient on how to prevent urinary


tract infections. Which statement is INCORRECT?
A. “Void immediately after sexual intercourse.”
B. “Avoid wearing tight fitting underwear.”
C. “Try to void every 2-3 hours”
D. “Use scented sanitary napkins or tampons during menstruation.”
8. A patient, who is having spasms and burning while urinating due to a UTI, is
prescribed “Pyridium” (Phenazopyridine). Which option below is a normal side effect
of this drug?
A. Hematuria
B. Crystalluria
C. Urethra mucous
D. Orange colored urine

9. You’re caring for a patient with an indwelling catheter. The patient complains of
spasm like pain at the catheter insertion site. Which of the following options below
are other signs and symptoms the patient could experience or the nurse could
observe if a urinary tract infection was present? SELECT-ALL-THAT-APPLY:
A. Increased WBC
B. Crystalluria
C. Positive McBurney’s Sign
D. Feeling the need to void even though a catheter is present
E. Dark and cloudy urine
F. Cramping

10. On your nursing care plan for a patient with a urinary tract infection, which of the
following would be appropriate nursing interventions? SELECT-ALL-THAT-APPLY:
A. Encourage voiding every 2-3 hours while awake.
B. Restrict fluid intake to 1-2 liters per day.
C. Monitor intake and output daily.
D. The patient verbalizes the importance of using vaginal sprays to decrease
reoccurrence of urinary tract infections prior to discharge home.

A client is admitted to the emergency department for possible acute pyelonephritis.


Which manifestation should the nurse consider to be consistent with this​disorder?
(Select all that​apply.)
A. Flank tenderness
B. Diarrhea
C. Nocturia
D. Urinary frequency
E. Vomiting

The nurse is caring for a client diagnosed with a urinary tract infection​(UTI). Which
assessment finding supports this​diagnosis?
(Select all that​apply.)
A. Abdominal pain
B. Burning sensation on urination
C. Clear urine
D. Hypothermia
E. Flank pain
The nurse is caring for a client with a urinary tract infection​(UTI). Which condition
should the nurse determine as a possible​cause? (Select all that​apply.)
A. Excessive oral fluid intake
B. Vesicoureteral reflux
C. Renal scarring
D. Structural deviations
E. Use of antibiotics

The nurse is caring for a client with pyelonephritis. Which clinical manifestation
should the nurse assess in the​client?
(Select all that​apply.)
A. Fever
B. Vomiting
C. Enuresis
D. Flank pain
E. Dysuria

The nurse is teaching the parents of an​18-month-old female toddler with a urinary
tract infection​(UTI). Which should be included in the teaching to prevent the future
risk of a​UTI?
A. Increase the​child's fluid intake.
B. Increase the​child's intake of vitamin C.
C. Provide the child with a daily cup of​low-sugar cranberry juice.
D. Cleanse the perineal area front to back.

A client asks which fluids to avoid in light of repeated urinary tract infections​(UTIs).
Which food should the nurse teach the client to​avoid? (Select all that​apply.)
A. Coffee
B. Citrus juices
C. Alcoholic beverages
D. Milk
E. Cranberry juice

Which topic is important to include in the home care teaching for a client with a
urinary tract infection​(UTI)? (Select all that​apply.)
A. Adequate fluid consumption
B. Good hygiene methods
C. Proper nutrition
D. Wearing polyester underwear
E. Voiding every 5 to 6 hours

The nurse is teaching a female client about the prevention of urinary tract infections​
(UTIs). Which information should the nurse​include?
A.​"Wash the perineum after​intercourse."
B.​"Avoid bubble​baths."
C.​"Void after​intercourse."
D.​"Empty the bladder every 2​hours."

The nurse is caring for a postpartum client. Which intervention is the most important
for the nurse to integrate into the plan of care to prevent a urinary tract infection​
(UTI)?
A.​"Change peri pads every 4​hours."
B.​"Use an antiseptic preparation after​voiding."
C.​"Increase fluid​intake."
D.​"Empty the bladder​completely."

The nurse is teaching parents of​school-age children practices that should decrease
the risk of urinary tract infections​(UTIs). Which information should the nurse​
include?
A. ​"Avoid large amounts of dairy in the​child's diet."
B.​"Provide drinks with sugar substitutes when​possible."
C. ​"Encourage the child to void five to six times a​day."
D.​"Encourage juices to increase the acidity of the​child's urine."

The client is reporting chills, fever, and left costovertebral pain. Which diagnostic test
should the nurse expect the HCP to prescribe first?
1.A midstream urine for culture.
2.A sonogram of the kidney.
3.An intravenous pyelogram for renal calculi.
4.A CT scan of the kidneys.
Fever, chills, and costovertebral pain are symptoms of a urinary tract infection (acute
pyelonephritis), which requires a urine culture first to confirm the diagnosis.

The female client in an outpatient clinic is being sent home with a diagnosis of
urinary tract infection (UTI). Which instruction should the nurse teach to prevent a
recurrence of a UTI?
1.Clean the perineum from back to front after a bowel movement.
2.Take warm tub baths instead of hot showers daily.
3.Void immediately preceding sexual intercourse.
4.Avoid coffee, tea, colas, and alcoholic beverages.

The nurse is discharging a client with a health-care facility acquired urinary tract
infection. Which information should the nurse include in the discharge teaching?
1.Limit fluid intake so the urinary tract can heal.
2.Collect a routine urine specimen for culture.
3.Take all the antibiotics as prescribed.
4.Tell the client to void every five (5) to six (6) hours.
The nurse is teaching an 80-year-old client with a urinary tract infection about the
importance of increasing fluids in the diet. Which of the following puts this client at a
risk for not obtaining sufficient fluids?
1. Diminished liver function.
2. Increased production of antidiuretic hormone.
3. Decreased production of aldosterone.
4. Decreased ability to detect thirst.
The sensation of thirst diminishes in those greater than 60 years of age; hence, fluid
intake is decreased and dissolved particles in the extracellular fluid compartment
become more concentrated. There is no change in liver function in older adults, nor
is there a reduction of ADH and aldosterone as a normal part of aging.

A client with a urinary tract infection is to take nitrofurantoin (Macrodantin) four times
each day. The client asks the nurse, "What should I do if I forget a dose?" What
should the nurse tell the client?
1. "You can wait and take the next dose when it is due."
2. "Double the amount prescribed with your next dose."
3. "Take the prescribed dose as soon as you remember it, and if it is very close to the
time for the next dose, delay that next dose."
4. "Take a lot of water with a double amount of your prescribed dose."

A nurse is assessing a client with a urinary tract infection who takes an


antihypertensive drug. The nurse reviews the client's urinalysis results pH 6.8, RBC
3 per high power field, color-yellow, specific gravity-1.030 . The nurse should:
1. Encourage the client to increase fluid intake.
2. Withhold the next dose of antihypertensive medication.
3. Restrict the client's sodium intake.
4. Encourage the client to eat at least half of a banana per day.
The client's urine specific gravity is elevated. Specific gravity is a reflection of the
concentrating ability of the kidneys. This level indicates that the urine is
concentrated. By increasing fluid intake, the urine will become more dilute.
Antihypertensives do not make urine more concentrated unless there is a diuretic
component within them. The nurse should not hold a dose of antihypertensive
medication. Sodium tends to pull water with it; by restricting sodium, less water, not
more, will be present. Bananas do not aid in the dilution of urine.

A client has nephropathy. The physician orders that a 24-hour urine collection be
done for creatinine clearance. Which of the following actions is necessary to ensure
proper collection of the specimen?
1. Collect the urine in a preservative-free container and keep it on ice.
2. Inform the client to discard the last voided specimen at the conclusion of urine
collection.
3. Ask the client what his weight is before beginning the collection of urine.
4. Request an order for insertion of an indwelling urinary catheter.
All urine for creatinine clearance determination must be saved in a container with no
preservatives and refrigerated or kept on ice. The first urine voided at the beginning
of the collection is discarded, not the last. A self-report of weight may not be
accurate. It is not necessary to have an indwelling urinary catheter inserted for urine
collection.

A 24-year-old female client comes to an ambulatory care clinic in moderate distress


with a probable diagnosis of acute cystitis. When obtaining the client's history, the
nurse should ask the client if she has had:
1. Fever and chills.
2. Frequency and burning on urination.
3. Flank pain and nausea.
4. Hematuria.
The classic symptoms of cystitis are severe burning on urination, urgency, and
frequent urination. Systemic symptoms, such as fever and nausea and vomiting, are
more likely to accompany pyelonephritis than cystitis. Hematuria may occur, but it is
not as common as frequency and burning.

The client asks the nurse, "How did I get this urinary tract infection?" The nurse
should explain that in most instances, cystitis is caused by:
1. Congenital strictures in the urethra.
2. An infection elsewhere in the body.
3. Urinary stasis in the urinary bladder.
4. An ascending infection from the urethra.
Although various conditions may result in cystitis, the most common cause is an
ascending infection from the urethra. Strictures and urine retention can lead to
infections, but these are not the most common cause. Systemic infections are rarely
causes of cystitis.

The client, who is a newlywed, is afraid to discuss her diagnosis of cystitis with her
husband. Which would be the nurse's best approach?
1. Arrange a meeting with the client, her husband, the physician, and the nurse.
2. Insist that the client talk with her husband because good communication is
necessary for a successful marriage.
3. Talk first with the husband alone and then with both of them together to share the
husband's reactions.
4. Spend time with the client addressing her concerns and then stay with her while
she talks with her husband.

The nurse teaches a female client who has cystitis methods to relieve her discomfort
until the antibiotic takes effect. Which of the following responses by the client would
indicate that she understands the nurse's instructions?
1. "I will place ice packs on my perineum."
2. "I will take hot tub baths."
3. "I will drink a cup of warm tea every hour."
4. "I will void every 5 to 6 hours."
Hot tub baths promote relaxation and help relieve urgency, discomfort, and spasm.
Applying heat to the perineum is more helpful than cold because heat reduces
inflammation. Although liberal fluid intake should be encouraged, caffeinated
beverages, such as tea, coffee, and cola, can be irritating to the bladder and should
be avoided. Voiding at least every 2 to 3 hours should be encouraged because it
reduces urinary stasis.

The client with cystitis is given a prescription for phenazopyridine hydrochloride


(Pyridium). The nurse should teach the client that this drug is used to treat urinary
tract infections by:
1. Releasing formaldehyde and providing bacteriostatic action.
2. Potentiating the action of the antibiotic.
3. Providing an analgesic effect on the bladder mucosa.
4. Preventing the crystallization that can occur with sulfa drugs.
Phenazopyridine hydrochloride (Pyridium) is a urinary analgesic that works directly
on the bladder mucosa to relieve the distressing symptoms of dysuria.

PELVIC INFLAMMATORY DISEASE


Transurethral resection of the prostate (TURP) is associated with all of the following
except:
Outpatient procedure in most cases

SYSTEMIC LUPUS ERYTHEMATOSUS


1. Which statements below are INCORRECT about Systemic Lupus Erythematosus?
Select all that apply:
A. Lupus mainly affects Black, Asian, and Hispanic men.
B. Lupus is a chronic condition that has periods of flare-ups and remission.
C. The joints and skin are rarely affected in lupus.
D. Systemic Lupus Erythematosus (SLE) is the most common form of lupus.
The answers are A and C. Options B and D are correct statements about lupus.
Option A is wrong because lupus mainly affects Black, Asian, and Hispanic WOMEN
(not men). Option C is wrong because lupus DOES affect the joints and skin along
with many other systems of the body (heart, lungs, kidneys, blood system, brain
etc.).
2. What is the name of the process in the body that is responsible for the death of a
cell?
A. Mitosis
B. Osmosis
C. Apoptosis
D. Meiosis
The answer is C: Apoptosis. In lupus, this process is not occurring effectively,
specifically the phagocytosis of the apoptotic bodies.

3. During the death process of a cell, which immune system cell is responsible for
ingesting the fragments of the dying cell?
A. T-cells
B. B-cells
C. Macrophages
D. Neutrophils
The answer is C. Macrophages (also known as phagocytes) digest the tiny
fragments of the dying cell called apoptotic bodies. In lupus, this process is either
occurring too slowly or not performed correctly (but either way the immune system
sees the nuclear material from these bodies as foreign “antinuclear antigens” and
forms autoantibodies against them and this causes immune complexes to form…and
inflammation of body systems).

4. Which statements are true about the pathogenesis of Systemic Lupus


Erythematosus? Select all that apply:
A. The phagocytosis process is not occurring properly, which causes the nuclear
material inside the dying cell to be seen as a foreign invader and antinuclear
antibodies are created.
B. During cell death the nucleus of the cell fails to condense, which leads to the
development of autoantibodies.
C. Immune complexes form and attach within the structures of important organs,
joints, and tissues, and this causes inflammation.
D. Neutrophil activity is decreased and this causes B-cells to attack phagocytes and
nuclear material within the cells, which leads to the activation of the complement
cascade system and inflammation.
The answers are A and C. These are correct statements about the pathogenesis of
SLE. In a nutshell, apoptosis is ineffective (specifically how the
macrophages/phagocytes consume the fragmented parts of the dying cell during
apoptosis). These fragmented parts of the dying cell are not consumed (either the
consumption by the macrophages is too slow or they are not consumed), and the
nuclear material inside the small bodies spill into the extracellular space. The
immune system recognizes this as an antigen “antinuclear antigens” and forms
antinuclear antibodies to attack them. The antibodies attack the antigens and form
immune complexes. These immune complexes now start to “float” around in the
body and get stuck in various organs, joints, and tissues. This leads to the activation
of the complement system and causes inflammation. This inflammation damages the
structure (hence the organs/tissues/joint)s.

5. A patient is diagnosed with Systemic Lupus Erythematosus (SLE). You note the
patient has a red rash that starts on the nose and expands onto the cheeks of the
face. This is known as what type of rash?
A. Discoid
B. Malar
C. Miliaria
D. Eczema
The answer is B: Malar (it’s also called a butterfly rash).

6. A patient with Systemic Lupus Erythematosus (SLE) takes the medication


Azathioprine. Which statement by the patient requires re-education?
A. “I’ve made sure all my vaccines are up-to-date, and I’m getting the Shingles
vaccine next week.”
B. “This medication can increase my chances for developing infection.”
C. “This medication is “steroid-sparing” meaning it may help lower the amount of
steroids I may need to manage my lupus.”
D. “This medication lowers my immune system.”
The answer is A. Azathioprine is an immunosuppressant. Patients can have
vaccines while taking this medication but NOT live vaccines…like shingles, MMR
etc.

7. A patient is undergoing testing for the evaluation of Systemic Lupus


Erythematosus. What lab findings below are associated with this condition? Select
all that apply:
A. Decreased ESR and CRP
B. Positive ANA
C. Positive Anti-dsDNA
D. Negative Anti-Sm
E. Low C3 and C4
The answers are B, C, and E. A positive ANA (anti-nuclear antibodies) demonstrates
there are autoantibodies the body created against the nuclei of the dying cells
(almost all patient with lupus will have a positive ANA), but these anti-nuclear
antibodies can also be present with other autoimmune disorders. A positive
anti-dsDNA (anti-double stranded DNA anti-body) is a particular anti-nuclear
antibody found in some patients with lupus and is not typically present in patients
who don’t have lupus. An INCREASE in ESR and CRP would be found in a patient
with lupus (especially during a flare) along with a POSITIVE Anti-Sm antibody (which
is a particular antibody found in the nucleus that is present in lupus).
8. A patient with Systemic Lupus Erythematosus is experiencing a complication
called Lupus Nephritis. What are some signs and symptoms that correlate with this
complication of SLE? Select all that apply:
A. Decreased Creatinine
B. Increased BUN
C. 48 hour urinary output of 720 mL
D. Proteinuria
E. Weight loss
F. Edema in upper and lower extremities
The answers are B, C D, and F. Lupus nephritis is a serious complication of SLE. It is
a condition that affects the functional units of the kidneys called the nephrons. In
severe cases, renal failure develops which leads to an INCREASED BUN/Creatinine,
low UOP (<30 mL/hr), proteinuria, weightGAIN, and swelling in the upper and lower
extremities.

9. Which medication below used to treat Systemic Lupus Erythematosus decreases


inflammation quickly, is not for long-term usage, and can lead to weight gain,
susceptibility to infection, diabetes, and osteoporosis?
A. Hydroxychloroquine
B. Prednisone
C. Azathioprine
D. Belimumab
The answer is B. The question above is describing steroids. Prednisone is a steroid
medication. These are medications used to treat lupus. They decrease inflammation
quickly, are not for long-term usage, and can lead to weight gain, susceptibility to
infection, diabetes, and osteoporosis.

10. A patient with Systemic Lupus Erythematosus is prescribed Hydroxychloroquine.


Select all the educational points you will include in the patient’s education about this
medication:
A. Hydroxychloroquine is considered a Biologic medication.
B. It is used long-term to help prevent flares.
C. This medication doesn’t produce results immediately. Therefore, the patient may
not see results for a couple of months.
D. This medication binds with a protein that supports the activity of B-cells, which
decreases the activity of B-cells.
E. The patient must have their eyes checked regularly to monitor the retina while
taking this medication.
The answers are B, C, and E. Hydroxychloroquine is an antimalarial medication. It
decreases antibodies attacking the body (option D describes a biologic like
Belimumab). If used long-term it can damage the retinas of the eyes (the patient
needs a minimum of 1 eye exam per year while taking this med). It also takes a
couple of months to start working.
11. A patient is prescribed Belimumab for treatment of lupus. The patient has been
taking medication for one month. Which finding below during a patient assessment
requires further evaluation?
A. The patient reports trouble sleeping and loss of energy.
B. The patient declines a flu shot via the intranasal route.
C. The patient says they have not noticed a change in symptoms.
D. The patient administers the medication in the subcutaneous tissue on the
abdomen.
The answer is A. Belimumab is a Biologic that binds with a protein that supports the
activity of B-cells to decrease the activity of B-cells, which decreases antibody
attacks and decreases inflammation. Depression and suicide are serious side effects
of this medication. Option A demonstrates the patient may be experiencing
depression and further evaluation is needed. Option B is correct because the patient
should avoid LIVE vaccines like the flu vaccine via the intranasal route, Option C is
correct because this medication can take up to 6 months to start working, and Option
D is correct because this medication can be given via injection (subq fat of the upper
legs or abdomen) or infusion.

12. A 26-year-old female with Systemic Lupus Erythematosus has been hospitalized
with a flare-up. The patient is now recovered and is about to be discharged. The
patient expresses that she wants to have a baby soon. How long should the woman
be advised to be in remission of this condition before trying to conceive a baby?
A. 2 months
B. 2 years
C. 6 months
D. 1 year
The answer is C. Women with lupus, who want to become pregnant, need to make
sure their lupus has been in control (hence remission….no flare-ups) for at least 6
months before conceiving. This is because there is a risk of miscarriage and clotting
issues. Pregnancy and the post-partum period can cause flares.

13. You’re providing education to a Systemic Lupus Erythematosus support group


about preventing flares. Which statement by a participant requires re-education
about this topic?
A. “Emotional stress and illness are triggers for a flare-up.”
B.”I always wear large-brimmed hats and long sleeves when I’m outside.”
C. “Exercise should be avoided due to the physical stress it causes on the body.”
D. “I will make it a priority to receive my yearly influenza vaccine.
The answer is C. Exercise is a very important step in preventing lupus flares. It helps
maintain joint stability and manages weight. Exercise should not be limited but
encouraged to as much as the patient can tolerate. Remember LESS for less flares:
Lower stress (avoid overworking, emotional, illness, and use techniques to have
prevent stress), Exercise (helps joints and manages weight), Sleep (need more than
8 hours to prevent the body from getting too exhausted), Sun Protection (sunscreen
and large-brimmed hats…sunlight can activate a flare).

A 26-year-old woman has been diagnosed with early systemic lupus erythematosus
(SLE) involving her joints. In teaching the patient about the disease, the nurse
includes the information that SLE is a(n):
A. Hereditary disorder of women but usually does not show clinical symptoms unless
a woman becomes pregnant.
B. Autoimmune disease of women in which antibodies are formed that destroy all
nucleated cells in the body.
C. Disorder of immune function, but it is extremely variable in its course, and there is
no way to predict its progression.
D. Disease that causes production of antibodies that bind with cellular estrogen
receptors, causing an inflammatory response.
Rationale: SLE has an unpredictable course, even with appropriate treatment.
Women are more at risk for SLE, but it is not confined exclusively to women. Clinical
symptoms may worsen during pregnancy but are not confined to pregnancy or the
perinatal period. All nucleated cells are not destroyed by the antinuclear antibodies.
The inflammation in SLE is not caused by antibody binding to cellular estrogen
receptors.

A patient with an acute exacerbation of systemic lupus erythematosus (SLE) is


hospitalized with incapacitating fatigue, acute hand and wrist pain, and proteinuria.
The health care provider prescribes prednisone (Deltasone) 40 mg twice daily.
Which nursing action should be included in the plan of care?
A. Institute seizure precautions.
B. Reorient to time and place PRN.
C. Monitor intake and output.
D. Place on cardiac monitor.
Lupus nephritis is a common complication of SLE, and when the patient is taking
corticosteroids, it is especially important to monitor renal function. There is no
indication that the patient is experiencing any nervous system or cardiac problems
with the SLE.

A patient with systemic lupus erythematosus (SLE) who has a facial rash and
alopecia tells the nurse, "I hate the way I look! I never go anywhere except here to
the health clinic." An appropriate nursing diagnosis for the patient is:
A. Activity intolerance related to fatigue and inactivity.
B. Impaired skin integrity related to itching and skin sloughing.
C. Social isolation related to embarrassment about the effects of SLE.
D. Impaired social interaction related to lack of social skills.
The patient's statement about not going anywhere because of hating the way he or
she looks supports the diagnosis of social isolation because of embarrassment about
the effects of the SLE. Activity intolerance is a possible problem for patients with
SLE, but the information about this patient does not support this as a diagnosis. The
rash with SLE is nonpruritic. There is no evidence of lack of social skills for this
patient.

A patient with polyarthralgia with joint swelling and pain is being evaluated for
systemic lupus erythematosus (SLE). The nurse knows that the serum test result
that is the most specific for SLE is the presence of:
A. Rheumatoid factor.
B. Anti-Smith antibody (Anti-Sm).
C. Antinuclear antibody (ANA).
D. Lupus erythematosus (LE) cell prep.
The anti-Sm is an antibody found almost exclusively in SLE. The other blood tests
are also used in screening but are not as specific to SLE.

Following instruction for a patient with newly diagnosed systemic lupus


erythematosus (SLE), the nurse determines that teaching about the disease has
been effective when the patient says:
A. "I should expect to have a low fever all the time with this disease."
B. "I need to restrict my exposure to sunlight to prevent an acute onset of
symptoms."
C. "I should try to ignore my symptoms as much as possible and have a positive
outlook."
D. "I can expect a temporary improvement in my symptoms if I become pregnant."

A 19-year-old patient who is taking azathioprine (Imuran) for systemic lupus


erythematosus has a check-up before leaving home for college. The health care
provider writes all of these orders. Which one should the nurse question?
A. Naproxen (Aleve) 200 mg BID
B. Give measles-mumps-rubella (MMR) immunization
C. Draw anti-DNA titer
D. Famotidine (Pepcid) 20 mg daily
Live virus vaccines, such as rubella, are contraindicated in a patient taking
immunosuppressive drugs. The other orders are appropriate for the patient.

A client is suspected of having systemic lupus erythematosus. The nurse monitors


the client, knowing that which of the following is one of the initial characteristic signs
of systemic lupus erythematosus?
A. Weight gain
B. Subnormal temperature
C. Elevated red blood cell count
D. Rash on the face across the bridge of the nose

The nurse is assigned to care for a client with systemic lupus erythematosus (SLE).
The nurse plans care knowing that this disorder is:
A. A local rash that occurs as a result of allergy
B. A disease caused by overexposure to sunlight
C. An inflammatory disease of collagen contained in connective tissue
D. A disease caused by the continuous release of histamine in the body

The nurse is assigned to care for a client admitted to the hospital with a diagnosis of
systemic lupus erythematosus (SLE). The nurse reviews the health care provider's
prescriptions. Which of the following medications would the nurse expect to be
prescribed?
A. Antibiotic
B. Antidiarrheal
C. Corticosteroid
D. Opioid analgesic
Treatment of SLE is based on the systems involved and symptoms. Treatment
normally consists of anti-inflammatory drugs, corticosteroids, and
immunosuppressants. The incorrect options are not standard components of
medication therapy for this disorder.

A nurse is collecting data on a client who complains of fatigue, weakness, malaise,


muscle pain, joint pain at multiple sites, anorexia, and photosensitivity. Systemic
lupus erythematosus (SLE) is suspected. The nurse further checks for which of the
following that is also indicative of the presence of SLE?
A. Emboli
B. Ascites
C. Two hemoglobin S genes
D. Butterfly rash on cheeks and bridge of nose

A nurse is reviewing the plan of care for a client who has systemic lupus
erythematosus (SLE). The client reports fatigue, joint tenderness, swelling, and
difficulty urinating. Which of the following laboratory findings should the nurse
anticipate ? (Select all that apply.)
A. Positive ANA
B. Increased hemoglobin
C. 2+ urine protein
D. Increased serum C3 and C4 complement
E. Elevated BUN
A positive antinuclear antibody (ANA) titer is an expected finding in a client who has
SLE. The ANA test identifies the presence of antibodies produced against the client's
own DNA. Increased urine protein is an expected finding due to renal involvement as
a result of SLE. Elevated BUN is an expected finding due to renal involvement in a
client who has SLE.

A nurse is providing teaching about self-care to a client who has SLE. Which of the
following statements by the client indicates a need for further teaching?
A. "I should avoid sun exposure."
B. "I will apply powder to any skin rash."
C. "I should use a mild hair shampoo."
D. "I will call my doctor if I have a cough."

A nurse is providing teaching to a client who has a new prescription for prednisone
(Deltasone). Which will the following be included in the teaching? (Select all that
apply.)
A. Hypotension can occur.
B. Weight gain is expected.
C. Abdominal striae may appear.
D. Loss of appetite may be present.
E. Moon facies may be evident.
Prednisone causes fluid retention which results in weight gain. Prednisone causes
weight gain, especially in the abdomen, and can result in the
appearance of abdominal striae. Moon facies (rounding of the face due to an
accumulation of fatty tissue) is an adverse effect while taking this medication.

A nurse is admitting a client who has suspected SLE. Which of the following clinical
findings support this diagnosis?
A. Weight loss
B. Petechiae on thighs
C. Increased hair growth
D. Alopecia
Areas of hair loss are an expected finding in a client who has SLE.

A nurse is caring for a client who has SLE and is experiencing an episode of
Raynaud's phenomenon. Which of the following clinical findings should the nurse
anticipate?
A. Swelling of joints of the fingers
B. Pallor of toes with cold exposure
C. Feet become reddened with ambulation
D. Client report of intense feeling of heat in the fingers
Pallor of the extremities occurs in Raynaud's phenomenon in a client who has SLE
and has been exposed to cold or stress.

IMMUNITY
1. Your patient is having a sudden and severe anaphylactic reaction to a medication.
You immediately stop the medication and call a rapid response. The patient’s blood
pressure is 80/52, heart rate 120, and oxygen saturation 87%. Audible wheezing is
noted along with facial redness and swelling. As the nurse you know that the first
initial treatment for this patient’s condition is?
A. IV Diphenhydramine
B. IV Normal Saline Bolus
C. IM Epinephrine
D. Nebulized Albuterol
The answer is C. IM or subq Epinephrine is the first-line treatment for anaphylaxis.
Epinephrine will cause vasoconstriction (this will increase the blood pressure and
decrease swelling) and bronchodilation (this will dilate the airways).

2. A 25-year-old female is admitted to the ER in anaphylactic shock due to a bee


sting. According to the patient’s mother, the patient is severely allergic to bees and
was recently stung by one. This type of anaphylactic reaction is known as a?
A. Type I Hypersensivity Reaction
B. Type II Hypersensivity Reaction
C. Type III Hypersensivity Reaction
D. Type IV Hypersensivity Reaction
The answer is A. Type I Hypersensitivity Reactions are immediate and cause
anaphylaxis. It occurs when an antigen (the allergen….in this case bee venom)
attaches to immunoglobulin E (IgE) antibodies. These antibodies are created due to
this allergen and attach to the mast cells and basophils.

3. Anaphylactic shock can occur due to either an immunological or


non-immunological cause. Select ALL the CORRECT statements about the
differences between an immunological reaction (anaphylactic) and
non-immunological reaction (anaphylactoid):
A. “In an immunological reaction (anaphylactic) IgE antibodies are created and they
attach to mast cells and basophils.”
B. “An immunological reaction (anaphylactic) requires a patient to be sensitized for
anaphylactic shock to occur.”
C. “A non-immunological reaction (anaphylactoid) causes the same reaction as an
anaphylactic reaction, but it’s not due to immunoglobulin IgE antibodies.”
D. “Some common substances that cause a non-immunological reaction
(anaphylactoid) are IV contrast dyes and NSAIDS.”
E. “A patient does not have to be sensitized for a non-immunological reaction
(anaphylactoid) to occur and it can happen with first time exposure.”
All the answers are correct: A, B, C, D, E

4. During anaphylactic shock the mast cells and basophils release large amounts of
histamine. What effects does histamine have on the body during anaphylactic
shock? Select all that apply:
A. Decreases capillary permeability
B. Vasodilation of vessels
C. Decreases heart rate
D. Shifts intravascular fluid to interstitial space
E. Constricts the airways
F. Stimulates contraction of GI smooth muscles
G. Inhibits the production of gastric secretions
H. Itching
The answers are B, D, E, F, and H.

5. You’re providing education to a patient, who has a severe peanut allergy, on how
to recognize the signs and symptoms of anaphylactic shock. Select all the signs and
symptoms associated with anaphylactic shock:
A. Hyperglycemia
B. Difficulty speaking
C. Feeling dizzy
D. Hypertension
E. Dyspnea
F. Itchy
G. Vomiting and Nausea
H. Fever
I. Slow heart rate
The answers are B, C, E, F, and G. Patients who are in anaphylactic shock will have
signs and symptoms associated with the effects of histamine.

6. Your patient is started on an IV antibiotic to treat a severe infection. During


infusion, the patient uses the call light to notify you that she feels a tight sensation in
her throat and it’s making it hard to breathe. You immediately arrive at the room and
assess the patient. While auscultating the lungs you note wheezing. You also notice
that the patient is starting to scratch the face and arms, and on closer inspection of
the face you note redness and swelling that extends down to the neck and torso. The
patient’s vital signs are the following: blood pressure 89/62, heart rate 118 bpm, and
oxygen saturation 88% on room air. You suspect anaphylactic shock. Select all the
appropriate interventions for this patient:
A. Slow down the antibiotic infusion
B. Call a rapid response
C. Place the patient on oxygen
D. Prepare for the administration of Epinephrine
The answers are B, C, and D.

7. A patient is in anaphylactic shock. The patient has a severe allergy to peanuts and
mistakenly consumed an eggroll containing peanut ingredients during his lunch
break. The patient is given Epinephrine intramuscularly. As the nurse, you know this
medication will have an effect on the body?
A. It will prevent a recurrent attack.
B. It will cause vasoconstriction and decrease the blood pressure.
C. It will help dilate the airways.
D. It will help block the effects of histamine in the body.
The answer is C
8. You’re providing care to a patient in anaphylactic shock. What is NOT a typical
medical treatment for this condition, and if ordered the nurse should ask for an order
clarification?
A. IV Diphenhydramine
B. Epinephrine
C. Corticosteroids
D. Isotonic intravenous fluids
E. IV Furosemide
The answer is E. Furosemide is a loop-diuretic. This medication removes extra fluid
from the blood volume. This is NOT used as treatment in anaphylactic shock.

9. You’re assessing a patient’s knowledge on how to use their EpiPen in case of an


anaphylactic reaction. You’re using an EpiPen trainer device to teach the patient.
What demonstrated by the patient shows the patient knows how to administer the
medication? Select all that apply:
A. The patient injects the medication in the subq tissue of the abdomen.
B. The patient massages the site after injection.
C. The patient administers the injection through the clothes.
D. The patient aspirates before injecting the medication.
The answers are B and C. EpiPen is an auto-injector that is administered in the
middle of the outer thigh. It is not given in the abdomen. The patient should massage
the site for 10 seconds after administration to increase absorption. It can be
administered through clothes, if needed. Aspiration is not required for administration
of this medication.

10. What is the MOST important step a nurse can take to prevent anaphylactic shock
in a patient?
A. Assessing, documenting, and avoiding all the patient allergies
B. Administering Epinephrine
C. Administering Corticosteroids
D. Establishing IV access
The answer is A. This is the MOST important and easiest step a nurse can take in
preventing anaphylactic shock in a patient.

11. A patient is having an anaphylactic reaction to an IV medication. What is the


FIRST action the nurse should take?
A. Administer Epinephrine
B. Call a Rapid Reponse
C. Stop the medication
D. Administer a breathing treatment
The answer is C. The FIRST step the nurse should take is to immediately remove
the allergen. This would be stopping the medication, and then call a rapid response.
The nurse should maintain the airway and start CPR (if needed) until help arrives.
12. What is the BEST position for a patient in anaphylactic shock?
A. Lateral recumbent
B. Supine with legs elevated
C. High Fowler’s
D. Semi-Fowler’s
The answer is B. This position will increase venous return to the heart, which will
help increase cardiac output and blood pressure.

Which of the following is a general characteristic of hypersensitivity reactions?


a. Immune responsiveness is depressed.
b. Antibody is involved in all reactions.
c. Either self-antigen or heterologous antigen may be involved.
d. The antigen triggering the reaction is a harmful one.

Which of the following is associated with an increase in IgE production?


a. Transfusion reaction
b. Activation of Th1 cells
c. Reaction to poison ivy
d. HDN

Which of the following would cause a positive DAT test?


a. Presence of IgG on red cells
b. Presence of C3b or C3d on red cells
c. A transfusion reaction due to preformed antibody
d. Any of the above

All of the following are associated with type I hypersensitivity except


a. release of preformed mediators from mast cells.
b. activation of complement.
c. cell-bound antibody bridged by antigen.
d. an inherited tendency to respond to allergens.

Which newly synthesized mediator has a mode of action similar to that of histamine?
a. LTB4
b. Heparin
c. ECF-A
d. PGD2

Which of the following is associated with anaphylaxis?


a. Buildup of IgE on mast cells
b. Activation of complement
c. Increase in cytotoxic T cells
d. Large amount of circulating IgG
To determine if a patient is allergic to rye grass, the best test to perform is
a. total IgE testing.
b. skin prick test.
c. DAT.
d. complement fixation.

Which condition would result in HDN?


a. Buildup of IgE on mother's cells
b. Sensitization of cytotoxic T cells
c. Exposure to antigen found on both mother and baby red cells
d. Prior exposure to foreign red cell antigen

What is the immune mechanism involved in type III hypersensitivity reactions?


a. Cellular antigens are involved.
b. Deposition of immune complexes occurs in antibody excess.
c. Only heterologous antigens are involved.
d. Tissue damage results from exocytosis.

Which of the following explains the difference between type II and type III
hypersensitivity reactions?
a. Type II involves cellular antigens.
b. Type III involves IgE.
c. IgG is involved only in type III reactions.
d. Type II reactions involve no antibody.

A 37-year-old woman received two units of packed red blood cells following a
surgical procedure. She had been transfused once before. Five days after surgery,
she experienced a slight fever and some hemoglobin in her urine, indicating a
delayed transfusion reaction. A DAT test on a blood sample was positive. Which of
the following statements best describes this reaction?
a. The patient had IgM antibodies to the red cells transfused.
b. The patient's reaction was due to an anamnestic response.
c. Only IgE was coating the transfused red blood cells.
d. The antibody present reacted best at room temperature.

ARTHRITIS
1. During a routine health check-up visit a patient states, “I’ve been experiencing
severe pain and stiffness in my joints lately.” As the nurse, you will ask the patient
what questions to assess for other possible signs and symptoms of rheumatoid
arthritis? Select-all-that-apply:
A. “Does the pain and stiffness tend to be the worst before bedtime?”
B. “Are you experiencing fatigue and fever as well?”
C. “Is your pain and stiffness symmetrical on the body?”
D. “Is your pain and stiffness aggravated by extreme temperature changes?”
The answers are B and C. Patients with RA will experience pain and stiffness in the
morning (for more than 30 minutes) , not bedtime. It is common for patients to have a
fever and be fatigued…remember RA affects the whole body not just the joints. It will
also affect the same joints on the opposite side of the body. Therefore, if the right
wrist is inflamed, painful, and stiff the left wrist will be as well. RA is NOT aggravated
by extreme temperatures. This is found in osteoarthritis.

2. True or False: Rheumatoid arthritis tends to affect women more than men and
people who are over the age of 60.
False: Yes, RA tends to affect women more than men BUT it can affect all
ages…most commonly 20-60 years old.

3. Identify the correct sequence in how rheumatoid arthritis develops:


A. Development of pannus, synovitis, ankylosis
B. Anklyosis, development of pannus, synovitis
C. Synovitis, development of pannus, anklyosis
D. Synovitis, anklyosis, development of pannus
The answer is C. The body attacks (specifically the WBCs) the synovium of the joint.
The synovium becomes inflamed and this process is called synovitis. The
inflammation of the synovium leads to thickening and the formation of a pannus,
which is a layer of vascular fibrous tissue. The pannus will grow so large it will
damage the bone and cartilage within the joint. The space in between the joints will
disappear and anklyosis will develop, which is the fusion of the bone.

4. A patient with severe rheumatoid arthritis is scheduled for a procedure called an


arthrodesis. The nursing student you are precepting asks what type of procedure this
is. Your response is:
A. “It is a procedure where the affected joint is removed and each end of the bones
found within that joint are fused together.”
B. “It is a procedure that involves replacing the joint with an artificial one.”
C. “It is a procedure where the surgeon goes in with a scope and cleans out the
affected joint.”
D. “It is a procedure where the synovium is completely removed within the joint,
which helps decrease inflammation of the joint.
The answer is A. An arthrodesis (also called joint fusion) is where the affected joint is
removed and the bones within it are fused together. Option B describes a joint
replacement. Option C is known as a surgical cleaning. Option D is known as a
synovectomy.

5. A 58 year old female is experiencing a flare-up with rheumatoid arthritis. While


assisting the patient with her morning routine, the patient verbalizes a pain rating of 7
on 1-10 scale in the right and left wrist along with severe stiffness. You note the wrist
joints to be red, warm, and swollen. What nonpharmalogical nursing interventions
can you provide to this patient to help alleviate pain and stiffness?
Select-all-that-apply:
A. Exercise the affected joints
B. Assist the patient with a warm shower or bath
C. Perform deep massage therapy to the wrist joints
D. Assist the patient with applying wrist splints
The answers are B and D. During flare-ups of RA the joint should be rested (not
exercised) and should not be deep massaged because this can further damage the
joint (in addition cause the patient more pain). Heat therapy, like a warm shower or
bath, will help alleviate the stiffness. Furthermore, cold therapy can be used to
reduce the inflammation along with splinting the affected joints to protect and rest
them.

6. You are providing education to a patient, who was recently diagnosed with
rheumatoid arthritis, about physical exercise. Which statement made by the patient is
correct?
A. “It is best I try to incorporate a moderate level of high impact exercises weekly into
my routine, such as running and aerobics.”
B. “I will be sure to rest joints that are experiencing a flare-up, but I will try to
maintain a weekly regime of range of motion exercises along with walking and riding
a stationary bike.”
C. “It is important I perform a range of motion exercises during joint flare-ups and
incorporate low-impact exercises into my daily routine.”
D. “Physical exercise should be limited to only a range of motion exercises to
prevent further joint damage.”
The answer is B. During flare-ups of RA the patient should rest the joint. However, it
is important the patient performs range of motion exercises along with LOW-IMPACT
exercise weekly (such as stationary bike riding, walking, water aerobics etc.). This
will help with increasing the patient’s energy level along with muscle strength and
maintain joint health.

7. Disease-modifying antirheumatic drugs (DMARDS) are used to treat rheumatoid


arthritis. Select-all-the drugs below that are DMARDS:
A. Dexamethasone (Decadron)
B. Hydroxychloroquine (Plaquenil)
C. Teriparatide (Forteo)
D. Calcitonin
E. Leflunomide (Arava)
F. Methotrexate (Trexall)
The answers are B, E, and F. These are DMARDs that can be prescribed for RA.
Option A is a corticosteroid. Option C and D are sometimes prescribed in
osteoporosis.
8. A patient with rheumatoid arthritis is experiencing sudden vision changes. Which
medication found in the patient’s medication list can cause retinal damage?
A. Hydroxychloroquine (Plaquenil)
B. Lefluomide (Arava)
C. Sulfasalazine (Azulfidine)
D. Methylprednisolone (Medrol)
The answer is A. This medication is a DMARD and can cause retinal damage.
Therefore, the patient should be monitored for vision changes.

9. You’re providing care to a patient with severe rheumatoid arthritis. While


performing the head-to-toe nursing assessment, you note the patient’s overall skin
color to be pale and the patient looks exhausted. You ask the patient how she is
feeling, and she says “I’m so tired. I can’t even get out of this bed without getting
short of breath.” Which finding on the patient’s morning lab work may confirm a
complication that can be experienced with rheumatoid arthritis?
A. Potassium 3.2 mEq/L
B. Hemoglobin 7 g/dL
C. Sodium 135 mEq/L
D. WBC count 6,500
The answer is B. Patients with RA can experience anemia. A hemoglobin level can
be helpful in diagnosing anemia (a normal level in females is 12 to 15.5 g/dL). The
patient’s signs and symptoms above are classic findings in anemia.

10. A physician suspects a patient may have rheumatoid arthritis due to the patient’s
presenting symptoms. What diagnostic testing can be ordered to help a physician
diagnose rheumatoid arthritis? Select all that apply:
A. Rheumatoid factor
B. Uric acid level
C. Erythrocyte sedimentation
D. Dexa-Scan
E. X-ray imaging
The answers are A, C, and E. These are diagnostic tests to help diagnose RA.
Option B is used in gout, and option D is used with osteoporosis.

On a visit to the clinic, a client reports the onset of early symptoms of rheumatoid
arthritis. The nurse should conduct a focused assessment for:
1. Limited motion of joints.
2. Deformed joints of the hands.
3. Early morning stiffness.
4. Rheumatoid nodules.

A client with rheumatoid arthritis states, "I can't do my household chores without
becoming tired. My knees hurt whenever I walk." Which nursing diagnosis would be
most appropriate?
1. Activity intolerance related to fatigue and pain.
2. Self-care deficit related to increasing joint pain.
3. Ineffective coping related to chronic pain.
4. Disturbed body image related to fatigue and joint pain.

Of the clients listed below, who is at risk for developing rheumatoid arthritis (RA)?
Select all that apply.
1. Adults between the ages of 20 and 50 years.
2. Adults who have had an infectious disease with the Epstein-Barr virus.
3. Adults that are of the male gender.
4. Adults who possess the genetic link, specifically HLA-DR4.
5. Adults who also have osteoarthritis.
Rheumatoid arthritis (RA) affects women three times more often than men, between
the ages of 20 and 55 years. Research has determined that RA occurs in clients who
have had infectious disease, such as the Epstein-Barr virus. The genetic link,
specifically HLA-DR4, has been found in 65% of clients with RA

A client is in the acute phase of rheumatoid arthritis. Which of the following should
the nurse identify as lowest priority in the plan of care?
1. Relieving pain.
2. Preserving joint function.
3. Maintaining usual ways of accomplishing tasks.
4. Preventing joint deformity.
Maintaining usual ways of accomplishing tasks would be the lowest priority during
the acute phase. Rather, the focus is on developing less stressful ways of
accomplishing routine tasks. Pain relief is a high priority during the acute phase
because pain is typically severe and interferes with the client's ability to function.
Preserving joint function and preventing joint deformity are high priorities during the
acute phase to promote an optimal level of functioning and reduce the risk of
contractures.

The nurse teaches a client about heat and cold treatments to manage arthritis pain.
Which of the following client statements indicates that the client still has a knowledge
deficit?
1. "I can use heat and cold as often as I want."
2. "With heat, I should apply it for no longer than 20 minutes at a time."
3. "Heat-producing liniments can be used with other heat devices."
4. "Ten to 15 minutes per application is the maximum time for cold applications."
Heat-producing liniment can produce a burn if used with other heat devices that
could intensify the heat reaction.

The client with rheumatoid arthritis tells the nurse, "I have a friend who took gold
shots and had a wonderful response. Why didn't my physician let me try that?"
Which of the following responses by the nurse would be most appropriate?
1. "It's the physician's prerogative to decide how to treat you. The physician has
chosen what is best for your situation."
2. "Tell me more about your friend's arthritic condition. Maybe I can answer that
question for you."
3. "That drug is used for cases that are worse than yours. It wouldn't help you, so
don't worry about it."
4. "Every person is different. What works for one client may not always be effective
for another."
The basic principle of therapeutic communication and a therapeutic relationship is
honesty

The teaching plan for the client with rheumatoid arthritis includes rest promotion.
Which of the following would the nurse expect to instruct the client to avoid during
rest periods?
1. Proper body alignment.
2. Elevating the part.
3. Prone lying positions.
4. Positions of flexion.
Positions of flexion should be avoided to prevent loss of functional ability of affected
joints.

After teaching the client with rheumatoid arthritis about measures to conserve energy
in activities of daily living involving the small joints, which of the following, if stated by
the client, would indicate the need for additional teaching?
1. Pushing with palms when rising from a chair.
2. Holding packages close to the body.
3. Sliding objects.
4. Carrying a laundry basket with clenched fingers and fists.
Carrying a laundry basket with clenched fingers and fists is not an example of
conserving energy of small joints

After teaching the client with severe rheumatoid arthritis about prescribed
methotrexate (Rheumatrex), which of the following statements indicates the need for
further teaching?
1. "I will take my vitamins while I'm on this drug."
2. "I must not drink any alcohol while I'm taking this drug."
3. "I should brush my teeth after every meal."
4. "I will continue taking my birth control pills."

A 25-year-old client taking hydroxychloroquine (Plaquenil) for rheumatoid arthritis


reports difficulty seeing out of her left eye. Correct interpretation of this assessment
finding indicates which of the following?
1. Development of a cataract.
2. Possible retinal degeneration.
3. Part of the disease process.
4. A coincidental occurrence.

The client diagnosed with RA is being seen in the outpatient clinic. Which preventive
care should the nurse include in the regularly scheduled clinic visits?
1. Perform joint x-rays to determine progression of the disease.
2. Send blood to the lab for an erythrocyte sedimentation rate.
3. Recommend the flu and pneumonia vaccines.
4. Assess the client for increasing joint involvement.

The client with RA has nontender, movable nodules in the subcutaneous tissue over
the elbows and shoulders. Which statement is the scientific rationale for the
nodules?
1. The nodules indicate a rapidly progressive destruction of the affected tissue.
2. The nodules are small amounts of synovial fluid that have become crystallized.
3. The nodules are lymph nodes which have proliferated to try to fight the disease.
4. The nodules present a favorable prognosis and mean the client is better.

The nurse is assessing a client diagnosed with RA. Which assessment findings
warrant immediate intervention?
1. The client complains of joint stiffness and the knees feel warm to the touch.
2. The client has experienced one (1)-kg weight loss and is very tired.
3. The client requires a heating pad applied to the hips and back to sleep.
4. The client is crying, has a flat facial affect, and refuses to speak to the nurse.

The client diagnosed with RA who has been prescribed etanercept, a tumor necrosis
factor alpha inhibitor, shows marked improvement. Which instruction regarding the
use of this medication should the nurse teach?
1. Explain the medication loses its efficacy after a few months.
2. Continue to have checkups and lab work while taking the medication.
3. Have yearly magnetic resonance imaging to follow the progress.
4. Discuss the drug is taken for three (3) weeks and then stopped for a week.

The client diagnosed with RA has developed swan-neck fingers. Which referral is
most appropriate for the client?
1. Physical therapy.
2. Occupational therapy.
3. Psychiatric counselor.
4. Home health nurse
The occupational therapist assists the client in the use of the upper half of the body,
fine motor skills, and activities of daily living. This is needed for the client with
abnormal fingers.
The nurse is planning the care for a client diagnosed with RA. Which intervention
should be implemented?
1. Plan a strenuous exercise program.
2. Order a mechanical soft diet.
3. Maintain a keep-open IV.
4. Obtain an order for a sedative.

The 20-year-old female client diagnosed with advanced unremitting RA is being


admitted to receive a regimen of immunosuppressive medications. Which question
should the nurse ask during the admission process regarding the medications?
1. "Are you sexually active, and, if so, are you using birth control?"
2. "Have you discussed taking these drugs with your parents?"
3. "Which arm do you prefer to have an IV in for four (4) days?"
4. "Have you signed an informed consent for investigational drugs?"

The nurse is caring for clients on a medical floor. Which client should the nurse
assess first?
1. The client diagnosed with RA complaining of pain at a "3" on a 1-to-10 scale.
2. The client diagnosed with SLE who has a rash across the bridge of the nose.
3. The client diagnosed with advanced RA who is receiving antineoplastic drugs IV.
4. The client diagnosed with scleroderma who has hard, waxlike skin near the eyes.

The nurse and a licensed practical nurse are caring for clients in a rheumatologist's
office. Which task can the nurse assign to the licensed practical nurse?
1. Administer methotrexate, an antineoplastic medication, IV.
2. Assess the lung sounds of a client with RA who is coughing.
3. Demonstrate how to use clothing equipped with Velcro fasteners.
4. Discuss methods of birth control compatible with treatment medications.

A client with rheumatoid arthritis tells the nurse, "I know it is important to exercise my
joints so that I won't lose mobility, but my joints are so stiff and painful that exercising
is difficult." Which of the following responses by the nurse would be most
appropriate?
1. "You are probably exercising too much. Decrease your exercise to every other
day."
2. "Tell the physician about your symptoms. Maybe your analgesic medication can be
increased."
3. "Stiffness and pain are part of the disease. Learn to cope by focusing on activities
you enjoy."
4. "Take a warm tub bath or shower before exercising. This may help with your
discomfort."
Superficial heat applications, such as tub baths, showers, and warm compresses,
can be helpful in relieving pain and stiffness.
Which of the following statements should the nurse include in the teaching session
when preparing a client for arthrocentesis? Select all that apply.
1. "A local anesthetic agent may be injected into the joint site for your comfort."
2. "A syringe and needle will be used to withdraw fluid from your joint."
3. "The procedure, although not painful, will provide immediate relief."
4. "We'll want you to keep your joint active after the procedure to increase blood
flow."
5. "You will need to wear a compression bandage for several days after the
procedure."

A client with osteoarthritis will undergo an arthrocentesis on his painful edematous


knee. What should be included in the nursing plan of care? Select all that apply.
1. Explain the procedure.
2. Administer preoperative medication 1 hour before surgery.
3. Instruct the client to immobilize the knee for 2 days after the surgery.
4. Assess the site for bleeding.
5. Offer pain medication.

A postmenopausal client is scheduled for a bone-density scan. To plan for the client's
test, what should the nurse communicate to the client?
1. Request that the client remove all metal objects on the day of the scan.
2. Instruct the client to consume foods and beverages with a high content of calcium
for 2 days before the test.
3. Inform the client that she will need to ingest 600 mg of calcium gluconate by
mouth for 2 weeks before the test.
4. Tell the client that she should report any significant pain to her physician at least 2
days before the test.
Metal will interfere with the test. Metallic objects within the examination field, such as
jewelry, earrings, and dental amalgams, may inhibit organ visualization and can
produce unclear images.

A physician orders a lengthy X-ray examination for a client with osteoarthritis. Which
of the following actions by the nurse would demonstrate client advocacy?
1. Contact the X-ray department and ask the technician if the lengthy session can be
divided into shorter sessions.
2. Contact the physician to determine if an alternative examination could be
scheduled.
3. Provide a dose of acetaminophen (Tylenol).
4. Cancel the examination because of the hard X-ray table.
Shorter sessions will allow the client to rest between the sessions.

Which of the following should the nurse assess when completing the history and
physical examination of a client diagnosed with osteoarthritis?
1. Anemia.
2. Osteoporosis.
3. Weight loss.
4. Local joint pain.

A client with osteoporosis needs education about diet and ways to increase bone
density. Which of the following should be included in the teaching plan? Select all
that apply.
1. Maintain a diet with adequate amounts of vitamin D, as found in fortified milk and
cereals.
2. Choose good calcium sources, such as figs, broccoli, and almonds.
3. Use alcohol in moderation because a moderate intake has no known negative
effects.
4. Try swimming as a good exercise to maintain bone mass. 5. Avoid the use of
high-fat foods, such as avocados, salad dressings, and fried foods.

Which of the following statements indicates that the client with osteoarthritis
understands the effects of capsaicin (Zostrix) cream?
1. "I always wash my hands right after I apply the cream."
2. "After I apply the cream, I wrap my knee with an elastic bandage."
3. "I keep the cream in the cabinet above the stove in the kitchen."
4. "I also use the same cream when I get a cut or a burn."

At which of the following times should the nurse instruct the client to take ibuprofen
(Motrin), prescribed for left hip pain secondary to osteoarthritis, to minimize gastric
mucosal irritation?
1. At bedtime.
2. On arising.
3. Immediately after a meal.
4. On an empty stomach.
Drugs that cause gastric irritation, such as ibuprofen, are best taken after or with a
meal, when stomach contents help minimize the local irritation.

The client diagnosed with osteoarthritis states, "My friend takes steroid pills for her
rheumatoid arthritis. Why don't I take steroids for my osteoarthritis?" Which of the
following is the best explanation?
1. Intra-articular corticosteroid injections are used to treat osteoarthritis.
2. Oral corticosteroids can be used in osteoarthritis.
3. A systemic effect is needed in osteoarthritis.
4. Rheumatoid arthritis and osteoarthritis are two similar diseases.

After teaching a group of clients with osteoarthritis about using regular exercise,
which of the following client statements indicates effective teaching?
1. "Performing range-of-motion exercises will increase my joint mobility."
2. "Exercise helps to drive synovial fluid through the cartilage."
3. "Joint swelling should determine when to stop exercising."
4. "Exercising outdoors year-round promotes joint relaxation."

The home health nurse is doing a follow-up visit to a patient with recently diagnosed
rheumatoid arthritis (RA). Which assessment made by the nurse indicates that more
patient teaching is needed?
A. The patient sleeps with two pillows under the head.
B. The patient has been taking 16 aspirins daily.
C. The patient requires a 2 hour midday nap.
D. The patient sits on a stool when preparing meals.

A patient with an exacerbation of rheumatoid arthritis (RA) is taking prednisone


(Deltasone) 40 mg daily. Which of these assessment data obtained by the nurse
indicate that the patient is experiencing a side effect of the medication?
a. The patient has experienced a recent 5-pound weight loss.
B. The patient's erythrocyte sedimentation rate (ESR) has increased.
C. The patient's blood glucose is 166 mg/dl.
D. The patient has no improvement in symptoms.

The health care provider plans to prescribe methotrexate (Rheumatrex) to a patient


with newly diagnosed rheumatoid arthritis (RA). The patient tells the nurse, "That
drug has too many side effects; I would rather wait until my joint problems are worse
before beginning any drugs." The most appropriate response by the nurse is
a."You should tell the doctor how you feel so the two of you can make a decision
together."
b."It is important to start methotrexate early in order to decrease the joint damage."
c."Methotrexate is not expensive and will be cheaper to take than other possible
drugs."
d."Methotrexate is very effective and has no more side effects than the other
available drugs.
Disease-modifying anti-rheumatic drugs (DMARDs) are prescribed early to prevent
the joint degeneration that occurs as soon as the first year with RA.

The biologic agent anakinra (Kineret) is prescribed for a patient who has moderately
severe rheumatoid arthritis (RA). When teaching the patient about this drug, the
nurse will include information about
a.symptoms of gastrointestinal (GI) irritation or bleeding.
b.self-administration of subcutaneous injections.
c.taking the medication with at least 8 oz of fluid.
d.avoiding concurrently taking aspirin or NSAIDs.
Anakinra is administered by subcutaneous injection. GI bleeding is not a side effect
of this medication. Because the medication is injected, instructions to take it with 8
oz of fluid would not be appropriate. The patient is likely to be concurrently taking
aspirin or NSAIDs and these should not be discontinued.
When helping a patient with rheumatoid arthritis (RA) plan a daily routine, the nurse
informs the patient that it is most helpful to start the day with
a.a warm bath followed by a short rest.
b.a 10-minute routine of isometric exercises.
c.stretching exercises to relieve joint stiffness.
d.active range-of-motion (ROM) exercise

Prednisone (Deltasone) is prescribed for a patient with an acute exacerbation of


rheumatoid arthritis. When the patient has a follow-up visit 1 month later, the nurse
recognizes that the patient's response to the treatment may be best evaluated by
a.blood glucose testing.
b.liver function tests.
c.serum electrolyte levels.
d.C-reactive protein level.
C-reactive protein is a marker for inflammation, and a decrease would indicate that
the corticosteroid therapy was effective.

When teaching range-of-motion exercises to a patient who is having an acute


exacerbation of rheumatoid arthritis (RA) with joint pain and swelling in both hands,
the nurse teaches the patient that
a.affected joints should not be exercised when pain is present.
b.cold applications before exercise will decrease joint pain.
c.exercises should be performed passively by someone other than the patient.
d.regular walking may substitute for range-of-motion (ROM) exercises on some
days.
Cold application is helpful in reducing pain during periods of exacerbation of RA.
Because the joint pain is chronic, patients are instructed to exercise even when joints
are painful. ROM exercises are intended to strengthen joints as well as improve
flexibility, so passive ROM alone is not sufficient. Recreational exercise is
encouraged but is not a replacement for ROM exercises.

The health care provider prescribes methotrexate (Rheumatrex) for a 28-year-old


woman with stage II moderate rheumatoid arthritis (RA). When obtaining a health
history from the patient, the most important information for the nurse to communicate
to the health care provider is that the patient has
a.a history of infectious mononucleosis as a teenager.
b.a family history of age-related macular degeneration of the retina.
c.been trying to have a baby before her disease becomes more severe.
d.been using large doses of vitamins and health foods to treat the RA.

The client with early-stage RA is being discharged from the outpatient clinic. Which
discharge instruction should the nurse teach regarding the use of non steroidal
antiinflammatory drugs (NSAIDs)?
1. Take an over-the-counter medication for the stomach.
2. Drink a full glass of water with each pill.
3. If a dose is missed, double the medication at the next dosing time.
4. Avoid taking the NSAID on an empty stomach.

The nurse is preparing to administer morning medications. Which medication should


the nurse administer first?
1. The pain medication to a client diagnosed with RA.
2. The diuretic medication to a client diagnosed with SLE.
3. The steroid to a client diagnosed with polymyositis.
4. The appetite stimulant to a client diagnosed with OA.
Pain medication is important and should be given before the client's pain becomes
worse.

Because of the inflammatory process in RA, a pannus forms in the joint. What is a
pannus?
A. Scar tissue restricting the joint
B. Vascular granulation tissue in the joint
C. Necrotic tissue sloughing into the joint
D. Fluid encapsulated in the joint

What common musculoskeletal health problem is often associated with RA?


A. Paget's disease
B. Fibromyalgia
C. Marfan syndrome
D. Osteoporosis

The nurse is providing teaching for a patient with RA who is receiving methotrexate.
Which teaching points must the nurse include?
A. The medication is given in a low dose once a week
B. Methotrexate is an immunosuppressant medication
C. Expect some increase in swelling while taking this medication
D. Avoid crowds of people and people who are ill
E. Report any mouth sores to the health care provider immediately

1. Osteoarthritis develops due to the deterioration of the synovium within the joint
that can lead to complete bone fusion.
The answer is FALSE: Osteoarthritis is the most common type of arthritis that
develops due to the deterioration of the HYALINE CARTILAGE (not synovium) of the
bone. This can lead to bone breakdown and osteophytes formation (bone spurs).

2. Which patients below are at risk for developing osteoarthritis?


Select-all-that-apply:
A. A 65 year old male with a BMI of 35.
B. A 59 year old female with a history of taking long term doses of corticosteroids.
C. A 55 year old male with a history of repeated right knee injuries.
D. A 60 year old female with high uric acid levels.
The answers are A and C. The risk factors for developing OA include: older age,
being overweight (BMI >25), repeated injuries to the weight bearing joints, genetics.
Option B is at risk for osteoporosis, and option D is at risk for gout.

3. During a head-to-toe assessment of a patient with osteoarthritis, you note bony


outgrowths on the distal interphalangeal joints. You document these findings as:
A. Bouchard’s Nodes
B. Heberden’s Nodes
C. Neurofibromatosis
D. Dermatofibromas
The answer is B. Bony outgrowths found on the DISTAL interphalangeal joint
(closest to the fingernail and furthest away from the body) is called Heberden’s
Node. If the bony outgrowth was found on the PROXIMAL interphalangeal joint
(middle joint of the finger…closest to the body) it is called Bouchard’s Node.

4. A patient with osteoarthritis is describing their signs and symptoms. Which signs
and symptoms below are NOT associated with osteoarthritis? Select-all-that-apply:
A. Morning stiffness greater than 30 minutes
B. Experiencing grating during joint movement
C. Fever and Anemia
D. Symmetrical joint involvement
E. Pain and stiffness tends to be worst at the end of the day
The answers are: A, C, D. These options are signs and symptoms found with
rheumatoid arthritis NOT osteoarthritis. In OA: morning stiffness is LESS than 30
minutes, it is NOT systemic as RA (so fever and anemia will not be present), and it is
asymmetrical (both joints are not involved). Pain and stiffness will actually be worse
at the end of the day compared to the beginning due to overuse of the joints.

5. A patient with osteoarthritis has finished their first physical therapy session. As the
nurse you want to evaluate the patient’s understanding of the type of exercises they
should be performing regularly at home as self-management. Select all the
appropriate types of exercise stated by the patient:
A. Jogging
B. Water aerobics
C. Weight Lifting
D. Tennis
E. Walking
The answers are B, C, E. The patient wants to perform exercises that are low impact
like: walking, water aerobics, stationary bike riding along with strength training (lifting
weights: helps strengthen muscles around the joint), ROM: improves the mobility of
the joint and decreases stiffness. It is important that patients with OA avoid high
impact exercises that will increase stress on weight bearing joints such as
running/jogging, jump rope, tennis, or any type of exercise with both feet off the
ground.

6. A 63 year old patient has severe osteoarthritis in the right knee. The patient is
scheduled for a knee osteotomy. You are providing pre-op teaching about this
procedure to the patient. Which statement made by the patient is correct about this
procedure?
A. “This procedure will realign the knee and help decrease the amount of weight
experienced on my right knee.”
B. “A knee osteotomy is also called a total knee replacement.”
C. “A knee osteotomy is commonly performed for patients who have osteoarthritis in
both knees.”
D. “This procedure will realign the unaffected knee and help alleviate the amount of
weight experienced on the right knee.”
The answer is A. A knee osteotomy is NOT known as a total knee replacement. A
knee osteotomy can be used as an alternative for a total knee replacement but is not
the same thing. In addition, a knee osteotomy is performed when there is OA on only
one side of the knee.

7. A patient newly diagnosed with osteoarthritis asks about the medication


treatments for their condition. Which medication is NOT typically prescribed for OA?
A. NSAIDs
B. Topical Creams
C. Oral corticosteroids
D. Acetaminophen (Tylenol)
The answer is C. Intra-articular corticosteroids (an injection in the joint) are
commonly prescribed rather than oral corticosteroids. Remember OA is within the
joint…not systemic so oral corticosteroids are not as effective. All the other
medications listed are prescribed in OA.

8. You receive your patient back from radiology. The patient had an x-ray of the hips
and knees for the evaluation of possible osteoarthritis. What findings would appear
on the x-ray if osteoarthritis was present? Select-all-that-apply:
A. Increased joint space
B. Osteophytes
C. Sclerosis of the bone
D. Abnormal sites of hyaline cartilage
The answers are B and C. The joint space would be DECREASED not increased in
OA. In addition, an x-ray cannot show hyaline cartilage…therefore, the cartilage
cannot be assessed on an x-ray. The radiologist would be looking for osteophytes
(bone spurs), sclerosis of the bone (abnormal hardening of the bones), and
decreased joint space.
1. During a home health visit you are helping a patient with gout identify foods in
their pantry they should avoid eating. Select all the foods below the patient should
avoid:
A. Sardines
B. Whole wheat bread
C. Sweetbreads
D. Crackers
E. Craft beer
F. Bananas
The answers are A, C, and E. A patient with gout should avoid foods high in
PURINES. These include most red meats, organ meats (liver, kidneys,
sweetbreads), alcohol (especially beer).

2. Gout is a type of arthritis that occurs due to the accumulation of ____________ in


the blood that causes needle-like crystals to form around the joints.
A. purines
B. creatinine
C. uric acid
D. amino acids
The answer is C. In gout, uric acid accumulates in the blood, which leads to the
formation of needle-like crystals around the joints.

3. Identify which patient below is at MOST risk for developing gout:


A. A 56 year old male who reports consuming foods low in purines.
B. A 45 year old male with a BMI of 40 who reports taking hydrochlorothiazide and
aspirin.
C. A 39 year old female hospitalized with bulimia that has a BMI of 24.
D. A 27 year old female with ulcerative colitis.
The answer is B. Gout is due to high levels of uric acid in the blood. This can either
be due to the kidney’s inability to excrete uric acid out of the body or the body is
producing too much uric acid. Some causes that can lead to increased uric acid
levels include: being overweight (BMI >25 is considered overweight), usage of
aspirin or diuretics (thiazides (hydrochlorothiazide) or loop), HIGH consumption of
purines, dehydration, renal problems. Option B is the only patient MOST at risk for
developing gout.

4. A patient is post-op from surgery. The patient has a history of gout. While
performing a head-to-toe assessment, you assess the patient for signs and
symptoms of gout. As the nurse, you know that gout tends to start at what site?
A. Elbow
B. Big toe
C. Thumb or index finger
D. Knees
The answer is B. Most patients tend to have an acute attack of gout that begins in
the big toe. Remember that patients who have a history of gout or who are
experiencing a hospitalization (due to the physical stress on the body) are at risk for
an acute gout attack. Therefore, the nurse should assess the patient for this during
the head-to-toe assessment.

5. A 75 year old male is admitted for chronic renal failure. You note that the patient
has white/yellowish nodules on the helix of the ear and fingers. The patient reports
they are not painful. As you document your nursing assessment findings, you will
document this finding as?
A. Nodosa
B. Keloid
C. Dermoid
D. Tophi
The answer is D. Tophi are white/yellowish nodules that are urate crystals. They start
to form together in large masses and can be found under the skin (helix ears,
elbows, fingers, toes etc.), joints, bursae, bones, which can lead to bone deformity
and joint damage. Patients with chronic renal failure are at risk for chronic gout due
to the kidney’s inability to remove uric acid from the body.

6. You’re developing a nursing care plan for a patient with gout present in the right
foot. What specific nursing interventions will you include in this patient’s plan of
care? Select all that apply:
A. Encourage fluid intake of 2-3 liter per day.
B. Provide patients with foods high in purine with each meal daily.
C. Place the patient's right foot on a foot board while the patient is in bed.
D. Administer PRN dose of Aspirin for a pain rating greater than 5 on 1-10 scale.
E. Apply alternating cold and warm compresses to the right foot as tolerated by the
patient daily.
The answers are A, C, E. These options are correct nursing intervention for this
patient. Option B is wrong because the patient should consume food LOW in purines
(remember purines increase uric acid levels). Option D is wrong because the patient
should AVOID aspirin. Aspirin (even low doses) increase uric acid levels.

7. A patient is ordered by the physician to take Allopurinol (Zyloprim) for treatment of


gout. You’ve provided education to the patient about this medication. Which
statement by the patient requires you to re-educate them about this medication?
A. “This medication will help relieve the inflammation and pain during an acute
attack.”
B. “It is important I have regular eye exams while taking this medication.”
C. “I will not take large doses of vitamin C supplements while taking this medication.”
D. “Allopurinol decreases the production of uric acid.”
The answer is A. Allopurinol is used to PREVENT gout attacks….not treat an acute
attack. NSAIDs and Colchicine are best for treating the inflammation and pain during
a gout attack. Allopurinol can cause vision changes, therefore the patient should
receive annual eye exams along with avoiding large doses of vitamin C due to the
risk of renal calculi formation.

8. During the 1000 medication pass, your patient reports to you that he is having
muscle pain and tingling in his fingers and toes. You note that the patient also has a
grayish color to his lips. You immediately notify the doctor. In addition, you would
hold which medication that is scheduled to be administered at 1000?
A. Ibuprofen
B. Prednisone
C. Colchicine
D. Aspirin
The answer is C. The signs and symptoms presenting in this patient are classic
signs of Colchicine toxicity. Therefore, the nurse should not administer Colchicine.
This medication can also cause GI upset and neutropenia.

9. It is important for patients with gout to avoid medications that can increase uric
acid levels. Which medication below is NOT known to increase uric acid levels?
A. Aspirin
B. Niacin
C. Cyclosporine
D. Tylenol
The answer is D. Option A-C increases uric acid. Option D does not.

10. You are providing a free clinic seminar to participants about gout. Which
statement by a participant about the occurrence of gout is correct?
A. “Gout attacks tend to awake the person out of their sleep in the middle of the
night.”
B. “The pain felt with gout tends to be intense during the first 30 minutes.”
C. “It is best for a patient experiencing gout to tightly bandage the affected extremity.”
D. “Typically acute gout attacks are predictable and tend to occur once or twice a
week.”
The answer is A. This is the only correct statement about gout. Option B is wrong
because the pain felt with gout tends to intensify within 4-24 hours (not 30 minutes).
Option C is wrong because any type of pressure (even the pressure of bed linens)
can majorly increase the pain felt with gout. Option D is wrong because gout attacks
come on suddenly and may only occur once and tend to have several months or a
year between attacks.

A nurse is providing a preventive teaching discussion with a client at risk for


osteoarthritis. Which guideline should be included in this discussion related to​
exercise?
A)Exercise is not recommended
B)If there is pain with​exercise, keep​going, this is building muscle
C)Participate in regular​exercise, including walking or swimming
D)Perform heavy weightlifting exercises three times per week

A client with possible osteoarthritis is scheduled for a synovial fluid analysis. The
nurse should explain to the client that this diagnostic test is being completed for what​
reason?
A)To rule out inflammatory arthritis and gout
B)To identify irregular joint space narrowing
C)To evaluate for increased density of subchondral bone
D)To determine the extent of joint damage
Joint fluid analysis is used to detect​inflammation, bacteria, and uric acid crystals in
order to rule out inflammatory arthritis and gout. This test will not provide information
on the extent of the joint damage. This test cannot identify the amount of joint space
that has narrowed. This test cannot evaluate bone density

The nurse is providing education at a community health fair about osteoarthritis.


Which are the general clinical manifestations associated with osteoarthritis that the
nurse should include when providing this education?
​A)Crepitus with movement of joint
B)Abrupt onset
C)Mild fever
D)Joint pain with activity
E)Pain and stiffness at night

The nurse is planning care for a client with osteoarthritis. Which potential problem is
priority for the nurse to​address?
A)Chronic pain
B)Fluid volume deficit
C)Impaired skin integrity
D)Impaired cardiac output
E)Impaired physical mobility

The nurse is providing education about home care for a client with osteoarthritis of
the knees. Which information should the nurse include during this educational​
session?
A)Encouraging heavy lifting to maintain muscle strength
B)Installing handrails in bathroom
C)Taking pain medications as ordered
D)Continuing activity with repetitive movement
E)Using assistive devices to minimize stress placed on affected joint

The nurse is providing education to a client who is scheduled for an osteotomy.


Which information should the nurse include about this​procedure?
A)The procedure will stop osteoarthritis from progressing.
B)Realignment of the joint will occur.
C)The joint will be reconstructed.
D)Excess debris will be flushed out.
An osteotomy is performed to realign the joint or to shift the joint load toward areas
of less cartilage damage.An osteotomy will not stop the process of
osteoarthritis.Joint​arthroplasty, not an​osteotomy, is a procedure that reconstructs
the​joint.Arthroscopy, not an​osteotomy, is a procedure used to debride the joint by
smoothing rough cartilage and flushing out excess debris.

Which diagnostic test will best determine the cause of joint damage with​
osteoarthritis?
A)Synovial fluid analysis
B)Joint​x-ray
C)Electromyogram​(EMG)
D)MRI of joint

Which surgical treatment can be used to realign a joint that is affected by​
osteoarthritis?
A)Joint biopsy
B)Joint arthroplasty
C)Arthroscopy
D)Osteotomy
An osteotomy is an incision into or through the bone that can realign or shift the joint
load into a better aligned point or toward a less damaged part of the joint.

Which health promotion activities support a healthy lifestyle for clients with​
osteoarthritis? Select all
A)Increase dietary intake of calcium
B)Maintain a normal weight
C)Use assistive devices as needed
D)Limit participation in ROM exercises
E)Use soft chairs and recliners for rest
Maintaining a normal weight places less strain on the joints than carrying additional
weight. Assistive devices such as grab​bars, a shower​chair, or​long-handled
grippers help the client to maintain an independent lifestyle in safety. ROM exercises
assist the client to maintain maximal use of joint mobility and are an important
component in the exercise plan.

Which interventions support effective management of chronic pain that is


experienced by a client with​osteoarthritis?
A)Applying cool compresses to painful joints to reduce inflammation
B)Using firm support in chairs and mattresses to properly align the body
C)Limiting isometric exercises to reduce strain on the joints
D)Encouraging resting painful joints
E)Teaching proper posture and good body mechanics for activities of mobility

Mrs. Gladek is a​70-year-old White woman who has experienced progressive


symptoms of osteoarthritis over the past 5 years. In addition to taking prescribed
analgesics and​anti-inflammatory medications, she is discussing interventions that
she implemented into her daily lifestyle to manage her disease condition. Which
activity would you suggest to Mrs. Gladek to provide additional exercise
opportunities for​her
A)Doubles tennis for older adults
B)Meditation
C)Interval training to jog a 5K
D)Water aerobic activities

Mrs. Kingsley is a​64-year-old African American woman who has returned to the
healthcare​provider's office for​follow-up of the osteoarthritic condition of her knees.
Although both knees are​affected, today her left knee is more swollen than the right
knee. During your​assessment, Mrs. Kingsley asks​you, "If I am losing the cartilage
in my​knees, why do my knees look​larger?" What would be an appropriate​
response?
A)Although the cartilage is​destroyed, you may be building up more bone in the
knee.
B)Since your condition has​progressed, your knees have developed contractures
and this increases the size of the knees.
C)As your joint cartilage​reduces, with​exercise, muscle mass​increases, which
makes the knees look larger.
D)With​osteoarthritis, sometimes inflammation increases the size of your knees or
fluid​build-up occurs.
Inflammation causes swelling of the knee​joint, which makes the joint appear larger.
Joint effusion or fluid​build-up may also occur. An increase in bone or muscle tissue
does not occur in osteoarthritis. Flexion contractures may develop with osteoarthritis
of the knee​joint, but this will not result in an increase in the size of the joint.

The nurse is providing education to a community group about developing


osteoarthritis. What​joints, commonly​affected, will the nurse include in the
educational​session?
A)Neck, shoulders, and ankles
B)Ankles, feet, and spine
C)Hands, knees, and hips
D)Knees, feet, and spine

The nurse is providing home care teaching to a client diagnosed with osteoarthritis.
Which statement is appropriate for the nurse to include in the teaching session for
this​client?
A)"When you begin your strengthening​exercises, it is appropriate to start with a
large weight and work your way​down."
B)​"Balance and agility exercises can help maintain daily living skills and have been
recommended by your healthcare​provider."
C)"Stretching all muscle groups for 30 minutes each day has been recommended by
the healthcare​provider."
D)"Water exercises should not be tried because water buoyancy increases force on
the​joints

A client diagnosed with osteoarthritis of the hands presents with bony lumps on the
end joints of her digits. How will the nurse document this finding in the medical​
record?
A)Lymph node swelling
B)Bouchard nodes
C)Osteoblasts
D)Heberden nodes
Heberden nodes are bony lumps occurring at the end joint of the digits in a client
with OA. Bouchard nodes occur in the middle joint of the digits.

A client complains of knee pain during an appointment at a medical clinic. After


reviewing the client​'s medical​record, the nurse notes the client has been taking​
over-the-counter NSAIDs, with no pain relief. The nurse can anticipate the
healthcare provider writing a prescription for what medication at ​discharge?
A)Morphine sulfate
B)Celecoxib​(Celebrex)
C)Acetaminophen​(Tylenol)
D)Naproxen​(Aleve)
Celecoxib​(Celebrex) is a prescription​NSAID, COX-2 inhibitor that relieves pain
associated with osteoarthritis.

A client experiencing severe pain that cannot be managed through pain medications
or nonpharmacologic therapy may need surgical intervention. Which surgical
treatments will the nurse include in the teaching session with other members of the
healthcare​team?
A)Serum hyaluronic acid
B)Osteotomy
C)Arthroplasty
D)Joint fusion

The nurse is planning care for a client with osteoarthritis of the hips. Which
intervention will assist in addressing the potential problem of physical​mobility?
A)Recommend a local support group for the client and spouse.
B)Teach active and passive range of motion exercises.
C)Educate the client about proper skin care.
D)Encourage the client to avoid all physical activity.

DIABETES MELLITUS
1. Which of the following symptoms do NOT present in hyperglycemia?
A. Extreme thirst
B. Hunger
C. Blood glucose <60 mg/dL
D. Glycosuria
The answer is C.

2. Type 1 diabetics typically have the following clinical characteristics:


A. Thin, young with ketones present in the urine
B. Overweight, young with no ketones present in the urine
C. Thin, older adult with glycosuria
D. Overweight, adult-aged with ketones present in the urine
The answer is A.

3. A patient with diabetes has a morning glucose of 50. The patient is sweaty, cold,
and clammy. Which of the following nursing interventions is the MOST important?
A. Recheck the glucose level
B. Give the patient ½ cup (4 oz) of fruit juice
C. Call the doctor
D. Keep the patient nothing by mouth
The answer is B .

4. Which of the following patients is at most risk for Type 2 diabetes?


A. A 6 year old girl recovering from a viral infection with a family history of diabetes.
B. A 28 year old male with a BMI of 49.
C. A 76 year old female with a history of cardiac disease.
D. None of the options provided.
The answer is B. Remember Type 2 diabetes risk factors are related to
lifestyle….being obese is a risk factor (BMI >30 in males is considered obese). So,
the 28 year old male with a BMI of 49 is most at risk for Type 2.

5. The _____ ______ secrete insulin which are located in the _______.
A. Alpha cells, liver
B. Alpha cells, pancreas
C. Beta cells, liver
D. Beta cells, pancreas
The answer is D.
6. A 36-year-old male is newly diagnosed with Type 2 diabetes. Which of the
following treatments do you expect the patient to be started on initially?
A. Diet and exercise regime
B. Metformin BID by mouth
C. Regular insulin subcutaneous
D. None, monitoring at this time is sufficient enough
The answer is A.

7. Which of the following statements are true regarding Type 2 diabetes treatment?
A. Insulin and oral diabetic medications are administered routinely in the treatment of
Type 2 diabetes.
B. Insulin may be needed during times of surgery or illness.
C. Insulin is never taken by the Type 2 diabetic.
D. Oral medications are the first line of treatment for newly diagnosed Type 2
diabetics.
The answer is B.

8. What statement or statements are INCORRECT regarding Diabetic Ketoacidosis?


A. DKA occurs mainly in Type 1 diabetics.
B. Ketones are present in the urine in DKA.
C. Cheyne-stokes breathing will always be present in DKA.
D. Severe hypoglycemia is a hallmark sign in DKA.
E. Options C & D
The answer is E.

9. A patient who has diabetes is nothing by mouth as prep for surgery. The patient
states they feel like their blood sugar is low. You check the glucose and find it to be
52. The next nursing intervention would be to:
A. Administer Dextrose 50% IV per protocol
B. Continue to monitor the glucose
C. Give the patient 4 oz of fruit juice
D. None, this is a normal blood glucose reading
The answer is A. This question requires critical thinking because the patient is NPO
for surgery and can NOT eat but is experiencing hypoglycemia. Normally, you could
give the patient 15 grams of a simple carbohydrate like 4 oz of fruit juice or soda,
glucose tablets, gel etc. per hypoglycemia protocol However, the patient can NOT
eat due to surgery prep. Therefore the nurse would need to administer Dextrose
50% IV per protocol to help increase the blood glucose and recheck the glucose
level.

10. A Type 2 diabetic may have all the following signs or symptoms EXCEPT:
A. Blurry vision
B. Ketones present in the urine
C. Glycosuria
D. Poor wound healing
The answer is B.

1. Which of the following patient statements about the diabetic diet regime is correct?
A. “I’ll try to consume about 20% carbs and 40% fats on a daily basis.”
B. “Foods that are high in mono and poly fats are avocados, olives, and nuts.”
C. “Meat increases the glycemic index; therefore, I should only consume 5% of
them on a daily basis.”
D. “I should completely avoid starchy vegetables like potatoes and corn.”

2. A patient with diabetes asks you about what type of exercise they should perform
throughout the week. The best response is:
A. Lifting weights
B. Sprinting
C. Swimming
D. Jumping

3. Which of the following statements are INCORRECT about exercise management


for the diabetic patient?
A. “I will check my blood glucose prior to exercise. If it is less than 200 I will eat a
complex carb snack prior to exercising.”
B. “I plan on exercising for an extended period. So I will check my blood glucose
prior, during, and after exercising.”
C. “My blood glucose is 268 and I have ketones in my urine. Therefore, I will
avoid exercising today.”
D. All of the options are correct statements.

4. A patient has a blood glucose of 45 and is sweating, cold, and clammy. The
patient is conscious. What is your next nursing intervention?
A. Recheck the blood glucose in 5 minutes.
B. Give the patient 15 grams of a complex carbohydrate.
C. No intervention is needed because this is normal blood glucose.
D. Give the patient 15 grams of a simple carbohydrate.

5. A patient with Type 2 Diabetes is started on the medication Glyburide. Which of


the following statements by the patient causes concern?
A. “I will monitor my blood glucose regularly because I know this medication can
cause low blood sugar.”
B. “I will consume no more than 8 oz. of alcohol per week.”
C. “I will continue monitoring my diet and participating in exercise while taking
this medication.”
D. “This medication works by stimulating the beta cells in the pancreas to make
insulin.”
6. A patient is scheduled to take a morning dose of Metformin. The patient is
scheduled for surgery tomorrow. Which of the following nursing interventions are
correct?
A. Administer the medication as ordered.
B. Hold the dose and notify the doctor for further orders.
C. Administer the medication as ordered but hold tomorrow's morning’s dose.
D. Check the patient’s blood glucose prior to administering the medication.

7. A patient is scheduled to take 10 units of Humulin N at 1100. When is the patient


most susceptible for hypoglycemia?
A. 1900
B. 1300
C. 1130
D. 1500

8. You administered 5 units of Humalog at 0800. What is the ONSET and


DURATION of this medication?
A. Onset: 15 minutes, Duration: 3 hours
B. Onset: 2 hours, Duration: 16 hours
C. Onset: 30 minutes, Duration: 1 hour
D. Onset: 2 hours, Duration: 24 hours

9. A patient taking the medication Precose asks when it is the best time to take this
medication. Your response is:
A. With the first bite of food
B. 1 hour prior to eating
C. 1 hour after eating
D. At bedtime

10. A patient has a blood glucose of 400. Which of the following medications could
be the cause of this?
A. Glyburide
B. Atenolol
C. Bactrim
D. Prednisone

11. A patient is scheduled to take 5 units of Humulin R and 10 units of NPH. What is
the proper way of mixing these insulins?
A. These insulins cannot be mixed, therefore, should be drawn up in different
syringes.
B. Draw-up the Humulin R insulin first and then the NPH insulin.
C. Draw-up 2.5 units of NPH, then 10 units of Humulin R, and then finish drawing
up 2.5 units of NPH.
D. Draw-up the NPH insulin first and then the Humulin R insulin.
12. A patient newly diagnosed with diabetes is about to be discharged home. You
are watching the patient administer insulin. Which of the following actions causes
you to re-educate them?
A. They massaged the site after administering the insulin.
B. They injected fat into their thighs.
C. They used an opposite side for injection compared to the last insulin injection.
D. They engaged in safety after administering the medication.

13. A patient with diabetes is experiencing a blood glucose of 275 when waking.
What is a typical treatment for this phenomenon?
A. None, this is a normal blood glucose reading.
B. The patient may need a night time dose of an intermediate-acting insulin to
counteract the morning hyperglycemia.
C. A bedtime snack may prevent this phenomenon.
D. This is known as the Somogyi effect and requires decreasing the bedtime
dose of insulin.

14. True or False: The Somogyi effect causes the patient to experience an increase
in their blood glucose during the hours of 2-3 am.
False

15. Which of the following insulins has no peak but a duration of 24 hours?
A. NPH
B. Novolog
C. Lantus
D. Humulin N

16. A patient is scheduled to take 7 units of Humulin R at 0830. You administer


Humulin R at 0900 in the right thigh. When do you expect this medication to peak?
A. 1300
B. 0930
C. 1100
D. 1700

17. When is a patient most susceptible to hypoglycemic symptoms after the


administration of insulin?
A. Onset
B. Peak
C. Duration
D. Duration & Peak

18. Which of the following insulins can be administered intravenously?


A. NPH
B. Lantus
C. Humulin R
D. Novolog

An 8-year-old with type I DM is complaining of a headache and dizziness and is


visibly perspiring. The nurse caring for the child should do which of the following?
1. Administer glucagon intramuscularly.
2. Offer the child 8 oz of milk.
3. Administer rapid-acting insulin (lispro).
4. Offer the child 8 oz of water or calorie-free liquid.
Milk is best to give for mild hypoglycemia, which would present with the symptoms
described.

The nurse is teaching the family about caring for their 7-year-old, who has been
diagnosed with type I DM. What information should the nurse provide about
type I DM?
1. Best managed through diet, exercise, and oral medication.
2. Can be prevented by proper nutrition and activity.
3. Characterized mainly by insulin resistance.
4. Characterized mainly by insulin deficiency.
Individuals with type I DM do not produce insulin. If one does not produce insulin,
type I DM is the diagnosis.

The most appropriate nursing diagnosis for a patient with type I DM is which of the
following?
1. Risk for infection related to reduced body defences.
2. Impaired urinary elimination (enuresis).
3. Risk for injury related to medical treatment.
4. Anticipatory grieving.

The nurse caring for a patient with type I DM is teaching how to self-administer
insulin. The proper injection technique is which of the following?
1. Position the needle with the bevel facing downward before injection.
2. Spread the skin prior to intramuscular injection.
3. Aspirate for blood return prior to injection.
4. Elevate the subcutaneous tissue before injection.
Skin tissue is elevated to prevent injection into the muscle when giving a
subcutaneous injection. Insulin is only given subcutaneously.

The nurse is caring for a child who complains of constant hunger, constant
thirst,frequent urination, and recent weight loss without dieting. The nurse can expect
that care for this child will include which of the following?
1. Limiting daily fluid intake.
2. Weight management consulting.
3. Strict intake and output monitoring.
4. Frequent blood glucose testing.

The school nurse is talking to a 14-year-old about managing type I DM. Which of the
following statements indicates the student's understanding of the disease?
1. "It really does not matter what type of carbohydrate I eat as long as I take the
right amount of insulin."
2. "I should probably have a snack right after gym class."
3. "I need to cut back on my carbohydrate intake and increase my lean protein
intake."
4. "Losing weight will probably help me decrease my need for insulin."
A carbohydrate is a carbohydrate, and insulin dosing is based on blood sugar level
and carbohydrates to be eaten.

The nurse is teaching a group of young adults regarding non modifiable risk factors
for the development of type 1 diabetes mellitus. Which attendee statement indicates
a need for further​instruction? (Select all that​apply.)
A.​"Type 1 diabetes mellitus can be passed on from one recessive gene from one​
parent."
B.​"There are genes such as the​HLA-DR3 and​HLA-DR4 genes that can cause type
1 diabetes​mellitus."
C.​"Type 1 diabetes mellitus can be caused by exposure to excessive heat and​
temperatures."
D.​"I can develop type 1 diabetes mellitus from bacterial​infections."
E.​"Type 1 diabetes mellitus is caused by exposure to processing of metals and​
proteins."

A young client is admitted for lethargy and weight loss. Which clinical manifestation
supports the​nurse's suspicion of diagnosis of type 1 diabetes​mellitus? (Select all
that​apply.)
A.Fever
B.Blurred vision
C.Weight gain
D.Glucosuria
E.Polyuria

A teacher sends a child to the school nurse due to frequent thirst and urination. Upon​
assessment, the nurse suspects the child has type 1 diabetes mellitus. Which
question should the nurse ask to gain data to support this​suspicion?
A.​"When did you last see your healthcare​provider?"
B.​"How is your​appetite?"
C.​"Do you play outside a​lot?"
D.​"Have you noticed any bruises on your​legs?"
The nurse is teaching the parents of a child with a new diagnosis of type 1 diabetes
mellitus. Which information should the nurse include regarding the pathophysiology
of the​disease?
A.Delta cell destruction causes type 1 diabetes mellitus.
B.Hyperglycemia happens when​50% of alpha cells are damaged.
C.Beta cells need help producing insulin.
D.Beta cells are destroyed.

The nurse is working a health fair and teaching the public about risk factors for type
1 diabetes mellitus. Which ethnicity would the nurse include as having the highest
risk in the United​States?
A.Hispanic
B.Caucasian American
C.African American
D.Asian American

The nurse is conducting discharge teaching with a client who has been newly
diagnosed with type 1 diabetes mellitus. Which statement from the client indicates
the need for additional​teaching?
A.​"It is important to test my blood sugar at least four times a​day."
B.​"I need to stay hydrated during the​day."
C.​"As long as​I'm in my​house, I can walk​barefoot."
D.​"I need to be alert for​infections."
Clients with diabetes should always wear shoes to protect their feet from injury. The
client should be alert for infection or​injuries, stay well​hydrated, and test the blood
sugar four times a day.

The nurse is caring for a child diagnosed with type 1 diabetes mellitus. The nurse
should teach the child and parents that insulin dosing is based on which​item?
A.Weight
B.Urine output
C.Diet
D.Age
Insulin dose is based on​diet, specifically carbohydrate intake. Insulin dose is not
based on​weight, age, or urine output.

Which finding in the medical record indicates a client has good control of type 1
diabetes​mellitus?
A.Blood pressure​150/90 mmHg
B.Free of amputations
C.Hemoglobin A1C​5.4%
D.Fasting blood sugar 200​mg/dL
The nurse is teaching a group of clients newly diagnosed with type 1 diabetes
mellitus. Which information should the nurse include in the​teaching?
A.​"Monitor blood glucose levels​weekly."
B.​"Take beta blockers daily to control blood​pressure."
C.​"Have routine pedicures​performed."
D.​"Schedule regular ophthalmology​visits."
The client with type 1 diabetes mellitus is at high risk for retinal damage.​Therefore,
the nurse would teach the client to schedule regular ophthalmology visits to monitor
vision.

Which statement made by a client with type 1 diabetes mellitus indicates an


understanding of instruction provided regarding disease​management? (Select all
that​apply.)
A.​"I should administer insulin during the day in multiple​injections."
B.​"I should maintain my hemoglobin A1C levels at or below​8%."
C.​"I should count calories consumed to determine insulin needs for each​day."
D.​"I should trim my toenails at an angle to prevent cutting the​skin."
E.​"I should obtain blood glucose levels prior to each insulin​injection."

The nurse is caring for a child who is hospitalized for the treatment of diabetic
ketoacidosis​(DKA). The​child's parents ask why their child is receiving potassium.
Which response by the nurse is​accurate?
A.​"Potassium is administered to treat​acidosis."
B.​"Potassium is administered to treat cerebral​edema."
C.​"Potassium is administered to treat​hypokalemia."
D.​"Potassium is administered to decrease blood glucose​levels."

A client with type 1 diabetes mellitus is being taught to monitor her blood glucose
level. Which factor affecting accurate glucose monitoring should the nurse include in
the​instruction? (Select all that​apply.)
A.Medication overdoses
B.White blood cell​(WBC) count
C.Low hematocrit level
D.Creatinine level
E.High hematocrit level

The nurse is providing teaching to a client with a new diagnosis of type 1 diabetes
mellitus. The nurse should instruct the client about incorporating which treatment to
help manage the​disease? (Select all that​apply.)
A.Exercise
B.Fluid restriction
C.Medication
D.Daily weight checking
E.Nutrition
The nurse is teaching a child with type 1 diabetes mellitus and his family about sick
day guidelines. Which statement by the family indicates successful​teaching?
A.​"We will test for ketones when the blood glucose level reaches 160​mg/dL."
B.​"We will test for ketones when the blood glucose level reaches 180​mg/dL."
C.​"We will test for ketones when the blood glucose level reaches 200​mg/dL."
D.​"We will test for ketones when the blood glucose level reaches 240​mg/dL."
Rationale: Blood glucose levels of 160​mg/dL, 180​mg/dL, and 200​mg/dL are​
elevated, but they would not require testing for ketones. Once the blood glucose
level exceeds 240​mg/dL, the child and family should test the urine for ketones.

The nurse is developing a teaching plan for carbohydrate counting for a client newly
diagnosed with type 1 diabetes mellitus. Which type of carbohydrate should the
nurse instruct the client to​restrict?
A.Complex carbohydrates
B.Dietary fructose
C.Refined sugars
D.Simple sugars
Refined sugars come from sugar cane and are used as natural sweeteners.

The nurse is taking a health history from a client who has type 1 diabetes mellitus.
Which client symptom may indicate the development of​complications? (Select all
that​apply.)
A.Dizziness
B.Numbness in the feet
C.Quick wound healing
D.Vision changes
E.Frequent voiding of urine

The nurse is caring for a client who received a daily​intermediate-acting insulin dose
at​8:00 a.m. At which time of the day should the nurse provide the client a snack to
prevent​hypoglycemia?
A.​11:00 a.m.
B.​9:00 p.m.
C.​6:00 p.m.
D.​2:00 p.m.
Intermediate-acting​(NPH) insulin peaks 6-8 hours after the injection.​Therefore, the
nurse would prepare a snack for the client beginning at​2:00 p.m.

Which action by a parent of a​12-year-old child with a new diagnosis of type 1


diabetes mellitus indicates a need for further​teaching?
A.Counting carbohydrates with the child
B.Allowing the child to check blood sugars
C.Discouraging​after-school sports
D.Scheduling a baseline exam with an ophthalmologist
The nurse is managing care for a client weighing 165 pounds who was admitted for
the treatment of diabetic ketoacidosis​(DKA). Which intervention would be most
appropriate for the nurse to include in the plan of​care? (Select all that​apply.)
A.Place the client on a telemetry monitor.
B.Provide a​high-protein diet.
C.Give 100 mL of normal saline bolus.
D.Measure intake and output every hour.
E.Administer​sliding-scale regular insulin.

The nurse is caring for a client with a​long-term history of type 1 diabetes mellitus
who has developed peripheral vascular disease. The nurse is unable to palpate the​
client's pedal pulses and the skin is cold to the touch. Which​long-term goal is most
appropriate for this​client?
A.The client will remain free of injury.
B.The​client's skin integrity will remain intact.
C.The client will remain free from infection.
D.The​client's fasting blood glucose levels will stay between 70 and 110​mg/dL.
Rationale: The client has impaired circulation as evidenced by cold skin and absent
pedal pulses that indicate a risk for impaired skin integrity due to gangrene. There is
no evidence the client is at risk for injury or has an infection. Having fasting blood
glucose levels in the normal range indicates good disease​management, but it does
not relate to the impaired circulation.

The nurse is caring for a child with type 2 diabetes mellitus. Which item in this​child's
history should the nurse recognize as a risk factor for this​disease? (Select all that​
apply.)
A.​High-fat diet
B.Race
C.Sex
D.Obesity
E.Family history

The nurse is conducting a health fair to screen for type 2 diabetes mellitus. Which
participant should the nurse consider to be at highest​risk?
A.​30-year-old nurse who works in an intensive care unit
B.​60-year-old retired architect who works at job site
C.​50-year-old office worker who sits at the computer
D.​40-year-old kindergarten teacher who works in a classroom

After performing a health history and physical assessment for a​client, the nurse
suspects type 2 diabetes mellitus. Which assessment finding is consistent with the​
nurse's suspicion?​(Select all that​apply.)
A.Decreased urination
B.Hyperglycemia
C.Extreme thirst
D.Acanthosis nigricans
E.Hypertension
Rationale: Symptoms that would lead the nurse to conclude the client has type 2
diabetes mellitus are extreme​thirst, hyperglycemia,​hypertension, and acanthosis
nigricans​(a condition in which the skin is velvety in texture and brownish black in
color with hyperkeratotic​plaques). A client with type 2 diabetes mellitus would have
increased and not decreased urination.

The nurse is caring for a client newly diagnosed with type 2 diabetes mellitus. Prior
to any teaching about​medications, the client informs the​nurse, "I cannot give myself
any​injections." How should the nurse​respond?
A.​"Insulin administration helps with better blood glucose​management."
B.​"It is understandable to be upset about a new medical​diagnosis."
C.​"Why do you think you will have to give yourself​injections?"
D.​"Type 2 diabetes mellitus can usually be managed with​pills, diet, and​exercise."

Which information should the school nurse provide when teaching a group of
adolescents the risk factors for type 2 diabetes​mellitus?
A.Get sufficient exercise and activity.
B.Monitor blood glucose levels.
C.Increase carbohydrate intake.
D.Limit the amount of protein intake.
Rationale: Frequently, children with type 2 diabetes mellitus develop the disease
from a sedentary lifestyle and obesity.​

The nurse is performing a physical assessment of a child. Which assessment finding


should cause the nurse to suspect type 2 diabetes​mellitus?
A.Presence of acanthosis nigricans
B.Body mass index 21 ​kg/m2
C.Blood pressure of​110/78 mmHg
D.Pale mucous membranes

The nurse is teaching the caregivers of an adolescent with a new diagnosis of type 2
diabetes mellitus what they should do every 3 months to monitor the disease. The
adolescent is currently taking metformin​(Glucophage). Which information should the
nurse​include? (Select all that​apply.)
A.Discuss alcohol and drug use.
B.Review blood glucose logs.
C.Obtain an eye exam.
D.Monitor hemoglobin A1C.
E.Assess injection sites.
The nurse preceptor is teaching a new graduate nurse about hypoglycemic agents
used to treat type 2 diabetes mellitus. Which information should the preceptor
include related to how these medications lower blood​sugar? (Select all that​apply.)
A.Stimulate hormones for hemodilution
B.Prevent breakdown of glycogen
C.Increase breakdown of insulin
D.Increase insulin secretion
E.Increase uptake of glucose by cells

The nurse is teaching a group of older adults with type 2 diabetes mellitus. Which
complication of the disease should the nurse​include? (Select all that​apply.)
A.Pulmonary disease
B.Functional disabilities
C.Polypharmacy
D.Cognitive impairment
E.Autoimmune diseases

A client recently diagnosed with type 2 diabetes mellitus reports difficulty managing
the disease. To which professional should the nurse refer the client for help with
caloric​intake?
A.Primary healthcare provider
B.Dietitian
C.Social worker
D.Personal trainer

A client is admitted with hyperosmolar hyperglycemic state​(HHS) and a blood


glucose level of 550​mg/dL. Which intervention should the nurse expect to include in
the plan of​care? (Select all that​apply.)
A.Give normal saline intravenously.
B.Assess level of orientation.
C.Obtain blood for hemoglobin A1C.
D.Monitor serum potassium levels.
E.Provide education about type 2 diabetes mellitus.

A client newly diagnosed with type 2 diabetes mellitus asks the nurse how to​"get rid​
of" this disease. How should the nurse​respond?
A.​"Type 2 diabetes mellitus cannot be cured. It will eventually progress to type 1​
diabetes."
B.​"You seem concerned about this diagnosis and we will do our best to help you
control​it."
C.​"You will always have type 2 diabetes mellitus. You cannot get rid of​it."
D.​"Type 2 diabetes mellitus can sometimes be eliminated by weight​loss, diet, and​
exercise."
The healthcare provider prescribes metformin​(Glucophage) to a client with newly
diagnosed type 2 diabetes mellitus. Which information should the nurse provide to
the​client?
A.This medication is used for clients who are unable to inject insulin.
B.This medication is only used in the adult population due to side effects.
C.This medication can take up to 3 months to show effectiveness.
D.This medication is unsafe for use by pregnant and lactating women.

The nurse is caring for a​15-year-old child newly diagnosed with type 2 diabetes
mellitus. Which task should the nurse expect to be completed quarterly for this​child?
(Select all that​apply.)
A.Make a foot assessment.
B.Review glucose records.
C.Discuss​alcohol, tobacco, and drug use.
D.Refer for an eye exam.
E.Measure fasting glucose levels.

Which information should the nurse provide the client with type 2 diabetes​mellitus?
A.Inspect your feet on a daily basis for open sores.
B.Treat hyperglycemia with concentrated sweets.
C.Include 100 minutes per week of activity and exercise.
D.Increase carbohydrate consumption in the diet.

A client with blood glucose of 450​mg/dL is diagnosed with hyperosmolar


hyperglycemic state​(HHS). Which assessment finding should the nurse​expect?
A.Increase in urinary output
B.Open wound to the foot
C.Lower extremity edema
D.Capillary refill of 2 seconds

The nurse is developing a plan of care for a client with ineffective peripheral tissue
perfusion related to microvascular changes. Which assessment finding supports this
nursing​diagnosis?
A.Absent pedal pulses
B.Capillary refill of 3 seconds
C.Fasting blood glucose of 100​mg/dL
D.Hemoglobin A1C of​6.4%
Rationale: Absence of pedal pulses indicates the peripheral tissue is not receiving
adequate oxygenation and in turn is the basis of the nursing diagnosis Tissue​
Perfusion: Peripheral, Ineffective.

Which suggestion should the nurse provide to a client with newly diagnosed type 2
diabetes mellitus regarding ways to increase​activity? (Select all that​apply.)
A.Play card games at home.
B.Take stairs at work.
C.Limit computer time.
D.Use a fitness tracker.
E.Get a workout buddy.

The nurse is evaluating the plan of care for an obese client diagnosed with type 2
diabetes mellitus 6 months prior. Which finding indicates the client is successfully
managing the​disease?
A.Hemoglobin A1C of​10.0%
B.Weight loss of 40 pounds
C.New foot wound with purulent drainage
D.Fasting blood sugars averaging 150​mg/dL

1. A patient with newly diagnosed type 2 diabetes mellitus asks the nurse what
""type 2"" means in relation to diabetes. The nurse explains to the patient that type 2
diabetes differs from type 1 diabetes primarily in that with type 2 diabetes
a. the pt is totally dependent on an outside source of insulin
b. there is a decreased insulin secretion and cellular resistance to insulin that is
produced
c. the immune system destroys the pancreatic insulin-producing cells
d. the insulin precursor that is secreted by the pancreas is not activated by the liver

The benefits of using an insulin pump include all of the following except:
a. By continuously providing insulin they eliminate the need for injections of insulin
b. They simplify management of blood sugar and often improve A1C
c. They enable exercise without compensatory carbohydrate consumption
d. They help with weight loss

A 54-year-old patient admitted with type 2 diabetes, asks the nurse what "type 2"
means. Which of the following is the most appropriate response by the nurse?
"1. ""With type 2 diabetes, the body of the pancreas becomes inflamed."
2. "With type 2 diabetes, insulin secretion is decreased and insulin resistance is
increased."
3. "With type 2 diabetes, the patient is totally dependent on an outside source of
insulin."
4. "With type 2 diabetes, the body produces autoantibodies that destroy b-cells in the
pancreas.""

A client is admitted to the hospital with signs and symptoms of diabetes mellitus.
Which findings is the nurse most likely to observe in this client? Select all that apply:
1. Excessive thirst
2. Weight gain
3. Constipation
4. Excessive hunger
5. Urine retention
6. Frequent, high-volume urination

A client is brought to the emergency department in an unresponsive state, and a


diagnosis of hyperglycemic hyperosmolar nonketotic syndrome is made. The nurse
would immediately prepare to initiate which of the following anticipated physician's
prescriptions?
1. Endotracheal intubation
2. 100 units of NPH insulin
3. Intravenous infusion of normal saline
4. Intravenous infusion of sodium bicarbonate

A client is taking Humulin NPH insulin daily every morning. The nurse instructs the
client that the mostlikely time for a hypoglycemic reaction to occur is:
A) 2-4 hours after administration
B) 4-12 hours after administration
C) 16-18 hours after administration
D) 18-24 hours after administration

A client who is started on metformin and glyburide would have initially presented with
which symptoms?
a. Polydispisa, polyuria, and weight loss
b. weight gain, tiredness, & bradycardia
c. irritability, diaphoresis, and tachycardia
d. diarrhea, abdominal pain, and weight loss

A client with diabetes mellitus demonstrates acute anxiety when first admitted for the
treatment of hyperglycemia. The most appropriate intervention to decrease the
client's anxiety would be to
1. administer a sedative
2. make sure the client knows all the correct medical terms to understand what is
happening
3. ignore the signs and symptoms of anxiety so that they will soon disappear
4. convey empathy, trust, and respect toward the client
The most appropriate intervention is to address the client's feelings related to the
anxiety

PYELONEPHRITIS
1. You’re assessing your patients during morning rounding. Which patient below is at
MOST risk for developing a urinary tract infection?
A. A 25 year old patient who finished a regime of antibiotics for strep throat 10 weeks
ago.
B. A 55 year old female who is post-op day 7 from hip surgery.
C. A 68 year old male who is experiencing nausea and vomiting.
D. A 87 year old female with Alzheimer’s disease who is experiencing bowel
incontinence.

2. A 36 year old female, who is 29 weeks pregnant, reports she is experiencing


burning when voiding. The physician orders a urinalysis. Which statement by the
patient demonstrates she understands how to collect the specimen?
A. “I’ll hold the cup firmly against the urethra while collecting the sample.”
B. “I will cleanse back to front with the antiseptic wipe before peeing in the cup.”
C. “First, I will pee a small amount of urine in the toilet and then collect the rest in the
cup.”
D. “I will be sure to drink a lot of fluids to keep the urine diluted before peeing into the
cup.”

3. During a head-to-toe assessment on a patient with a possible urinary tract


infection, you perform costovertebral angle percussion. The costovertebral angle is
found?
A. between the bottom of the 12th rib and spine
B. between the right upper quadrant and umbilicus
C. between the sternal notch and angle of Louis
D. between the ischial spine and umbilicus

4. A 76 year old female is admitted due to a recent fall. The patient is confused and
agitated. The family members report that this is not normal behavior for the patient.
They explain that the patient is very active in the community and cares for herself.
Based on the information you have gathered about the patient, which physician’s
order takes priority?
A. “Collect a urinalysis”
B. “Collect a T3 and T4 level”
C. “Insert a Foley Catheter”
D. “Keep patient NPO”

5. The physician orders a urine culture on your patient in room 5505 with a urinary
tract infection. In addition, the patient is ordered to start IV Bactrim
(Sulfamethoxazole/Trimethoprim). How will you proceed with following this order?
A. First, hang the antibiotic, and then collect the urine culture.
B. First, hang the antibiotic and when the antibiotic is finished infusing, collect the
urine culture.
C. First, collect the urine culture, and then hang the antibiotic.
D. First, collect the urine culture and then hold the dose of the antibiotic until the
urine culture is back from the lab.
6. A patient with a urinary tract infection is taking Bactrim
(Sulfamethoxazole/Trimethoprim). As the nurse you know, is it important that the
patient consumes 2.5 to 3 L of fluid per day to prevent any of the following
complications?
A. Brown urine
B. Crystalluria
C. Renal Stenosis
D. Renal Calculi

7. You’re providing discharge teaching to a female patient on how to prevent urinary


tract infections. Which statement is INCORRECT?
A. “Void immediately after sexual intercourse.”
B. “Avoid wearing tight fitting underwear.”
C. “Try to void every 2-3 hours”
D. “Use scented sanitary napkins or tampons during menstruation.”

8. A patient, who is having spasms and burning while urinating due to a UTI, is
prescribed “Pyridium” (Phenazopyridine). Which option below is a normal side effect
of this drug?
A. Hematuria
B. Crystalluria
C. Urethra mucous
D. Orange colored urine

9. You’re caring for a patient with an indwelling catheter. The patient complains of
spasm like pain at the catheter insertion site. Which of the following options below
are other signs and symptoms the patient could experience or the nurse could
observe if a urinary tract infection was present? SELECT-ALL-THAT-APPLY:
A. Increased WBC
B. Crystalluria
C. Positive McBurney’s Sign
D. Feeling the need to void even though a catheter is present
E. Dark and cloudy urine
F. Cramping

10. On your nursing care plan for a patient with a urinary tract infection, which of the
following would be appropriate nursing interventions? SELECT-ALL-THAT-APPLY:
A. Encourage voiding every 2-3 hours while awake.
B. Restrict fluid intake to 1-2 liters per day.
C. Monitor intake and output daily.
D. The patient verbalizes the importance of using vaginal sprays to decrease
reoccurrence of urinary tract infections prior to discharge home.
CANCER
1. Which statements best describe tumor grading? Select all that apply:
A. Tumor grading provides information on how tumor cells look under a microscope.
B. Cancer can be graded using the TNM classification system.
C. Tumor grading describes the location and size of the main tumor, and if the
cancer has spread to other parts of the body like the lymph nodes or other organs.
D. A biopsy is used to help grade a tumor.
The answers are A and D. These options are correct regarding tumor grading.
Options B and C describe cancer staging.

2. Your patient’s tumor grading report stated “well differentiated, Grade I”. You know
what this means?
A. The patient’s tumor cells were different in appearance and arrangement compared
to what normal cells possess.
B. This type of cancer is considered high grade.
C. This type of cancer is low grade because the tumor cells have an appearance and
arrangement that is similar to normal cells.
D. This grade of cancer tends to spread and grow quickly.
The answer is C.

3. A tumor was shown to possess cells that were moderately differentiated. This is
considered a?
A. Grade I
B. Grade II
C. Grade III
D. Grade IV
The answer is B: Grade II

4. True or False: All forms of cancer use the same grading system.
The answer is FALSE.
Some forms of cancer have their own grading systems, like breast cancer which
uses the Nottingham system.

5. Which statement below is an incorrect statement about tumor staging?


A. Staging describes the main tumor’s location and size, and if the cancer has
spread to other parts of the body like the lymph nodes or other organs.
B. It’s useful in developing a treatment plan and if the patient can participate in a
clinical trial.
C. A cancer stage designated at the time of the cancer diagnosis can change
overtime.
D. A CT scan, MRI, x-ray, and ultrasound can be used to help stage a cancer.
The answer is C. This statement is incorrect. All the other statements are true. It’s
important to note that the cancer stage designated at the time of the cancer
diagnosis does NOT change, and if the cancer spreads or metastases that
information is added onto the original category designation.

6. The TNM staging system for cancer can be used to help stage what type of
cancer?
A. Brain cancer
B. Spinal cord cancer
C. Leukemia
D. Colon cancer
The answer is D. The TNM staging system is not used for brain, spinal cord or blood
cancers (including leukemia). It is used for solid tumor cancers.

7. What is the meaning of cancer in situ?


A. The tumor is found in its original place and has not spread from its original
location.
B. The tumor is formed outside its original place and has spread within its localized
site.
C. The tumor is cancerous and may spread.
D. The tumor is distant from its original place.
The answer is A.

8. The TNM staging system has been used to stage your patient’s cancer. The report
says: T1N0M0 What is the meaning of N0?
A. Cancer in regional lymph node can’t be measured
B. No cancer present in regional lymph node
C. Cancer in distant lymph node can’t be measured
D. No cancer found in other body parts
The answer is B: No cancer present in regional lymph node

9. The category “T” in the TNM system stands for:


A. Time
B. Tumor (primary)
C. Tissue (primary)
D. Tumor (secondary)
The answer is B.

10. The category “N” in the TNM system stands for:


A. Number
B. Necrosis
C. Nodes
D. Normal
The answer is C.

11. The category “M” in the TNM system gives detail about:
A. if the cancer has spread to other parts of the body and if this is the case, how
much and the location of it
B. the number and location of the lymph nodes that have cancer
C. the size of the tumor that is growing into other tissues
D. if the cancer cells appear abnormal
The answer is A.

12. What TNM classification below best describes this finding: very large tumor with
1 regional lymph node involvement, and spread to two regional organs?
A. T1N1M2
B. T4N1M1
C. T1N1M1
D. T4N1M2
The answer is B. T4N1M1

13. Which TNM classification has the poorest prognosis?


A. T0N0M0
B. T2N2M1
C. T4N1M0
D. T4N3M1
The answer is D: T4N3M1

14. On a patient’s TNM classification you see a lowercase “p” (pT1) in front of the T
on the patient report. This mean?
A. The cancer has been restaged.
B. The cancer was staged based on the findings after surgery.
C. The cancer was staged before the treatment began and was based on test results
and assessment findings.
D. The cancer was staged after therapy was administered.
The answer is B. The “p” represents pathological staging and this classification is
given based on the findings after surgery.

15. Which stage of cancer is called metastatic cancer and means the cancer has
spread to other parts of the body beyond where the cancer started?
A. Stage II
B. Stage 0
C. Stage III
D. Stage I
E. Stage IV
The answer is E: Stage IV

A nurse cares for a client who has a genetic mutation that increases the risk for
colon cancer. The client states that he does not want any family to know about this
result. How should the nurse respond?
a. "It is required by law that you inform your siblings and children about this result so
that they also can be tested and monitored for colon cancer."
b. "It is not necessary to tell your siblings because they are adults, but you should tell
your children so that they can be tested before they decide to have children of their
own."
c. "It is not required that you tell anyone about this result. However, your siblings and
children may also be at risk for colon cancer and this information might help them."

A nurse cares for a client who recently completed genetic testing that revealed that
she has a BRCA1 gene mutation. Which actions should the nurse take next? (Select
all that apply.)
a. Discuss potential risks for other members of her family.
b. Assist the client to make a plan for prevention and risk reduction.
c. Disclose the information to the medical insurance company.
d. Recommend the client complete weekly breast self-examinations.
e. Assess the client's response to the test results.
f. Encourage support by sharing the results with family members.

A nurse obtains health histories when admitting clients to a medical-surgical unit.


With which client should the nurse discuss predisposition genetic testing?
a. Middle-aged woman whose mother died at age 48 of breast cancer
b. Young man who has all the symptoms of rheumatoid arthritis
c. Pregnant woman whose father has sickle cell disease
d. Middle-aged man of Eastern European Jewish ancestry

The nursing instructor explains the difference between normal cells and benign
tumor cells. What information does the instructor provide about these cells?
a. Benign tumors grow through invasion of other tissue.
b. Benign tumors have lost their cellular regulation from contact inhibition.
c. Growing in the wrong place or time is typical of benign tumors.
d. The loss of characteristics of the parent cells is called anaplasia.

A nurse has taught a client about dietary changes that can reduce the chances of
developing cancer. What statement by the client indicates the nurse needs to provide
additional teaching?
a. "Foods high in vitamin A and vitamin C are important."
b. "I'll have to cut down on the amount of bacon I eat."
c. "I'm so glad I don't have to give up my juicy steaks."
d. "Vegetables, fruit, and high-fiber grains are important."
To decrease the risk of developing cancer, one should cut down on the consumption
of red meats and animal fat. The other statements are correct.

A nurse is participating in primary prevention efforts directed against cancer. In which


activities is this nurse most likely to engage? (Select all that apply.)
a. Demonstrating breast self-examination methods to women
b. Instructing people on the use of chemoprevention
c. Providing vaccinations against certain cancers
d. Screening teenage girls for cervical cancer
e. Teaching teens the dangers of tanning booths

A client receiving intravenous chemotherapy asks the nurse the reason for wearing a
mask, gloves, and gown while administering drugs to the client. What is the nurse's
best response?
a."These coverings protect you from getting an infection from me."
b."I am preventing the spread of infection from you to me or any other client here."
c."The policy is for any nurse giving these drugs to wear a gown, gloves, and mask."
d."The clothing protects me from accidentally absorbing these drugs."

A client is on chemotherapy and has a platelet count of 25,000. Which intervention is


most important to teach this client?
a. "Eat a low-bacteria diet."
b."Take your temperature daily."
c."Use a soft-bristled toothbrush."

A client with chemotherapy-induced bone marrow suppression has received


filgrastim (Neupogen). Which laboratory finding indicates that this therapy is effective
for the client?
a.Hematocrit is 28%.
b.Hematocrit is 38%.
c.Segmented neutrophil count is 2500/mm3.
d.Segmented neutrophil count is 3500/mm3.
Filgrastim is a single-lineage growth factor that stimulates the maturation and release
of only segmented neutrophils. This drug is not given unless the neutrophil count is
dangerously low. The near-normal range of neutrophils indicates effective therap

A nurse is reviewing the white blood cell count with a differential for a client receiving
chemotherapy for cancer. Which finding alerts the nurse to the possibility of sepsis?
a.Total white blood cell count is 9000/mm3.
b.Lymphocytes outnumber basophils.
c."Bands" outnumber "segs."
d.Monocyte count is 1800/mm3.

The nurse is administering a combination of three different antineoplastic drugs to a


patient who has metastatic breast cancer. Which statement best describes the
rationale for combination therapy?
a. There will be less nausea and vomiting.
b.Increased cancer-cell killing will occur.
c.The drugs will prevent metastasis.
d.Combination therapy reduces the need for radiation therapy.

When giving chemotherapy as cancer treatment, the nurse recognizes that toxicity to
rapidly growing normal cells also occurs. Which rapidly growing normal cells are also
harmed by chemotherapy? (Select all that apply.)
a.Bone marrow cells
b.Retinal cells
c.Hair follicle cells
d.Nerve myelin cells
e. Gastrointestinal (GI) mucous membrane cells

One patient has cancer of the bone; another has cancer in the connective tissues of
the thigh muscles; a third patient has cancer in the vascular tissues. These patients
have a type of tumor referred to as a
a.sarcoma.
b.leukemia.
c.carcinoma.
D.lymphoma.
Sarcomas are malignant tumors that arise from connective tissues. These tissues
can be found in bone, cartilage, muscle, blood, lymphatic, and vascular tissues. The
other options are incorrect.

A patient who has cancer is about to begin chemotherapy. The patient asks the
nurse why two chemotherapeutic agents are being used instead of just one. Which
response by the nurse is correct?
a. "The drugs may be given in less toxic doses if two drugs are used."
b. "Two agents used together can have synergistic effects."
c. "Use of two drugs will increase tumorcidal activity in the G0 phase of the cell."
d. "Using two agents will shorten the length of time chemotherapy is needed."

The nurse is teaching a patient who will begin receiving targeted therapy for cancer.
The patient asks how targeted therapy differs from other types of chemotherapies.
The nurse will explain that targeted therapy
a. damages cancer cell DNA to prevent cell replication.
b. directly kills or damages cancerous cells.
c. interferes with specific molecules in cancer cells.
d. prevents metastasis of cancer cells.

A client with vaginal cancer is being treated with a radioactive vaginal implant. The
client's husband asks the nurse if he can spend the night with his wife. The nurse
should explain that:
A. Overnight stays by family members are against hospital policy.
B. There is no need for him to stay because staffing is adequate.
C. His wife will rest much better knowing that he is at home.
D. Visitation is limited to 30 minutes when the implant is in place.

A client has cancer of the pancreas. The nurse should be most concerned about
which nursing diagnosis?
A.Alteration in nutrition
B.Alteration in bowel elimination
C. Alteration in skin integrity
D. Ineffective individual coping

The registered nurse is making assignments for the day. Which client should be
assigned to the pregnant nurse?
A.The client with HIV
B. The client with a radium implant for cervical cancer
C. The client with RSV (respiratory synctial virus)
D.The client with cytomegalovirus

Which health care worker should not be assigned to care for the client with a radium
implant for vaginal cancer?
A.The doctor who is six months postpartum
B.The Nursing Assistant who is pregnant
C.The RN who is allergic to iodine
D.The RN with a three-year-old at home

A client is admitted with Ewing's sarcoma. Which symptoms would be expected due
to this tumor's location?
A. Hemiplegia
B. Aphasia
C. Nausea
D. Bone Pain
Sarcoma is a type of bone cancer, therefor, bone pain would be expected

A client with acute leukemia develops a low white blood cell count. In addition to the
institute of isolation the nurse should:
A. Request that food be served with disposable utensils
B. Ask the client to wear a mask when visitors are present
C. Prep IV with mild soap, water, and alcohol
D. Provide foods in seal single serving packages

A client with leukemia is receiving Trimetrexate. After reviewing the client's chart, the
physician orders Wellcovorin (leucovorin calcium). The rationale for administering
leucovorin calcium to a client receiving Trimetrexate is to:
A. Treat iron-deficiency anemia caused by chemotherapeutic agents
B. Create a synergistic effect that shortens treatment time
C. Increase the number of circulating neutrophils
D. Reverse drug toxicity and prevent tissue damage
Leucovorin is the antidote for Methotrexate and Trimetrexate which are folic acid
antagonists. Leucovorin is a folic acid derivative.

A 33-year-old male is being evaluated for possible acute leukemia. Which of the
following would the nurse inquire about as a part of the assessment?
A. The client collects stamps as a hobby.
B. The client recently lost his job as a postal worker.
C. The client had radiation for treatment of Hodgkin's disease as a teenager.
D. The client's brother had leukemia as a child.

An African American client is admitted with acute leukemia. The nurse is assessing
for signs and symptoms of bleeding. Where is the best site for examining the
presence of petechiae?
A. The abdomen
B. The thorax
C. The earlobes
D. The soles of the feet

A client with acute leukemia is admitted to the oncology unit. Which of the following
would be most important for the nurse to inquire?
A. "Have you noticed a change in sleeping habits recently?"
B. "Have you had a respiratory infection in the last 6 months?"
C. "Have you lost weight recently?"
D. "Have you noticed changes in your alertness?"
The client with leukemia is at risk for infection and has often had recurrent
respiratory infections during the previous 6 months.

Which of the following would be the priority nursing diagnosis for the adult client with
acute leukemia?
A. Oral mucous membrane, altered related to chemotherapy
B. Risk for injury related to thrombocytopenia
C. Fatigue related to the disease process
D. Interrupted family processes related to life-threatening illness of a family member
The client with acute leukemia has bleeding tendencies due to decreased platelet
counts, and any injury would exacerbate the problem.

A client being treated for advanced breast cancer with chemotherapy reports that
she must be allergic to one of her drugs because her entire face is swollen. What
assessment does the nurse perform?
A. Asks whether the client has other known allergies
B. Checks the capillary refill on fingernails bilaterally
C. Examines the client's neck and chest for edema and engorged veins
D. Compares blood pressure measured in the right arm with that in the left arm
The client's swollen face indicates possible superior vena cava syndrome, which is
an oncologic emergency.

A client receiving high-dose chemotherapy who has bone marrow suppression has
been receiving daily injections of epoetin alfa (Procrit). Which assessment finding
indicates to the nurse that today's dose should be held and the health care provider
notified?
A. Hematocrit of 28%
B. Total white blood cell count of 6200 cells/mm3
C. Blood pressure change from 130/90 mm Hg to 148/98 mm Hg
D. Temperature change from 99° F (37.2° C) to 100° F (37.8° C)

Which statement made by the client allows the nurse to recognize whether the client
who is receiving brachytherapy for ovarian cancer understands the treatment plan?
A. "I may lose my hair during this treatment."
B. "I must be positioned in the same way during each treatment."
C. "I will have a radioactive device in my body for a short time."
D. "I will be placed in a semi private room for company."
Brachytherapy refers to short-term insertion of a radiation source.

The client receiving chemotherapy will experience the lowest level of bone marrow
activity and neutropenia during which period?
A. Peak
B. Trough
C. Nadir
D. Adjuvant
The lowest point of bone marrow function is referred to as the nadir.

The registered nurse is teaching a nursing student about the importance of


observing for bone marrow suppression during chemotherapy. Select the person
who displays bone marrow suppression.
A. Client with hemoglobin of 7.4 and hematocrit of 21.8
B. Client with diarrhea and potassium level of 2.9 mEq/L
C. Client with 250,000 platelets
D. Client with 5000 white blood cells/mm3
Bone marrow suppression causes anemia, leukopenia, and thrombocytopenia; this
client has anemia demonstrated by low hemoglobin and hematocrit.

Which client problem does the nurse set as the priority for the client experiencing
chemotherapy-induced peripheral neuropathy?
A. Potential for lack of understanding related to side effects of chemotherapy
B. Risk for Injury related to sensory and motor deficits
C. Potential for ineffective coping strategies related to loss of motor control
D. Altered sexual function related to erectile dysfunction
The highest priority is safety.

Which intervention will be most helpful in preventing disseminated intravascular


coagulation (DIC)?
A. Monitoring platelets
B. Administering packed red blood cells
C. Using strict aseptic technique to prevent infection
D. Administering low-dose heparin therapy for clients on bedrest

When caring for a client with suspected syndrome of inappropriate antidiuretic


hormone secretions (SIADH), the nurse reviews the medical record to uncover which
signs and symptoms are consistent with this syndrome?
(SATA)
A. Hyponatremia
B. Mental status changes
C. Azotemia
D. Bradycardia
E. Weakness

The nurse anticipates administering which medication to treat hyperuricemia


associated with tumor lysis syndrome (TLS)?
A. Recombinant erythropoietin (Procrit)
B. Allopurinol (Zyloprim)
C. Potassium chloride
D. Radioactive iodine 131
Tumor lysis syndrome results in hyperuricemia, Allopurinol decreases uric acid
production and is indicated in TLS.

Which manifestation of an oncologic emergency requires the nurse to contact the


health care provider immediately?
A. New onset of fatigue
B. Edema of arms and hands
C. Dry cough
D. Weight gain
Edema of the arms and hands indicates worsening compression of the superior vena
cava consistent with superior vena cava syndrome. The compression must be
relieved immediately, often with radiation therapy, because death can result without
timely intervention.

Which precaution is most important for the nurse to teach a client receiving radiation
therapy for head and neck cancer?
A. Avoid eating red meat during treatment.
B. Pace your leisure activities to prevent fatigue.
C. See your dentist twice yearly for the rest of your life.
D. Avoid using headphones or headsets until your hair grows back.

Combinations of antineoplastic drugs are frequently used for which purpose? (Select
all that apply.)
A Prevent drug resistance
B Lower cost of treatment
C Decrease drug tolerance
D Provide a synergistic action
E Decrease the severity of adverse effects.

The client receives nolvadex (tamoxifen) for treatment of breast cancer. She asks the
nurse why the medicine works. What is the best response by the nurse?
1. "Tamoxifen works by blocking estrogen receptors on breast tissue."
2. "Tamoxifen works by inhibiting the cellular mitosis of breast cancer."
3. "Tamoxifen works by inhibiting the metabolism of breast cancer cells."
4. "Tamoxifen works by binding to the DNA of breast cancer cells."

In staging and grading neoplasm, the TNM system is used. TNM stands for:
A Time, neoplasm, mode of growth
B. Tumor, node, metastasis
C Tumor, neoplasm, mode of growth
D Time, node, metastasis

Breast self examination (BSE) is one of the ways to detect breast cancer earlier. The
nurse is conducting a health teaching to female clients in a clinic. During evaluation
the clients are asked to state what they learned. Which of the following statements
made by a client needs further teaching about BSE? (FURTHER INSTRUCTION key
words)
A "BSE is done after menstruation."
B. "BSE palpation is done by starting at the center going to the periphery in a circular
motion."
C "BSE can be done in either supine or standing position."
D "BSE should start from age 20."

Which of the following clients is most at risk for developing multiple myeloma?
A A 20-year-old Asian woman
B A 30-year-old White man
C A 50-year-old Hispanic woman
D A 60-year-old Black man

An elderly female patient is receiving the drug megestrol (Megace). Which is the
most likely reason megestrol is ordered for this patient?
a.Migraine headaches
b.Osteoporosis
c.Appetite stimulant

The nurse is reviewing the laboratory results of a client receiving chemotherapy. The
platelet count is 10,000 cells/mm. Based on this laboratory value, the priority nursing
assessment is which of the following?
A Assess level of consciousness
B Assess temperature
C Assess bowel sounds
D Assess skin turgor

A 56-year-old woman is currently receiving radiation therapy to the chest wall for
recurrent breast cancer. She calls her health care provider to report that she has
pain while swallowing and burning and tightness in her chest. Which of the following
complications of radiation therapy is most likely responsible for her symptoms?
A Hiatal hernia
B Stomatitis
C Radiation enteritis
D Esophagitis

The nurse is caring for a client who is receiving chemotherapy. Which of the
following would be expected as a result of the massive cell destruction that occurred
from the chemotherapy?
A Leukopenia.
B Anemia.
C Thrombocytopenia.
D Hyperuricemia.

The client with non-Hodgkin's lymphoma is being managed with vincristine


(Vincasar). Which of the following indicates a side effect specific to this medication?
A. Alopecia.
B Numbness in the toes.
C Chest heaviness.
D Weight gain.
A side effect specific to this medication is peripheral neuropathy. It can be
manifested as numbness and tingling sensation in the finger and toes.

A patient is receiving chemotherapy for the treatment of cancer. The nurse


anticipates nadir to occur in
A. 2 days.
B. 8 days.
C. 15 days.
D. 30 days
*Nadir is when the bone marrow and WBC are at their lowest*
The oncologist has told the patient that he or she has a benign tumor in the liver. The
patient asks the nurse, "What is the main difference between benign and malignant
tumors?" Which answer by the nurse is correct?
1 "Malignant tumors usually are encapsulated."
2 "Malignant tumors have a rare recurrence rate."
3 "Benign tumors do not invade and spread to other organs."
4 "Malignant tumors require less nutrients for their cells than benign tumors."

The nurse providing care for a patient with suspected cancer recalls that the only
diagnostic procedure that is definitive for a diagnosis of cancer is:
1MRI
2Biopsy
3CT scan
4Tumor marker

LUNG CANCER
The nurse is taking the social history of a client diagnosed with SCLC. Which
information is significant for this disease?
1. Worked with asbestos for a short time many years ago.
2. Has no family Hx of this type of lung cancer.
3. Has numerous tattoos on upper and lower arms.
4. Has smoked 2 packs of cigarettes/day for 20 years.

The nurse writes a problem of 'impaired gas exchange' for a client diagnosed with
cancer of the lung. Which interventions should be included for the plan of care?
Select all that apply.
1. Apply O2 via nasal cannula.
2. Have the dietician plan for 6 small meals per day.
3. Place the client in resp. isolation.
4. Assess vital signs for fever.
5. Listen to lung sounds every shift.

The nurse is discussing cancer statistics with a group from the community. Which
information about death rates from lung cancer is accurate?
1. LC is the number 2 cause of cancer deaths in both men and women.
2. LC is the number 1 cause of cancer deaths in both men and women.
3. LC deaths are not significant in relation to other cancers.
4. LC deaths have continued to increase in the male population.

The client diagnosed with lung cancer has been told that the cancer has
metastasized to the brain. Which intervention should the nurse implement?
1. Discuss implementing an advance directive.
2. Explain the use of chemotherapy for brain involvement.
3. Teach the client to discontinue driving.
4. Have the significant other make decisions for the client.
This is a terminal situation. End of life decisions must be made.

The client diagnosed with lung cancer is in an investigational program and receiving
a vaccine to treat the cancer. Which information regarding investigational regimens
should the nurse teach?
1. Investigational regimens provide a better chance of survival for the client.
2. Investigational treatments have not been proven helpful to clients.
3. Clients will be paid to participate in an investigational program.
4. Only clients that are dying qualify for investigational treatments.

The client diagnosed with LC is being discharged. Which statement made by the
client indicates that more teaching is needed?
1. It doesn't matter if I smoke now. I already have cancer.
2. I should see the oncologist at my scheduled appointment.
3. If I begin to run a fever I should notify my HCP.
4. I should plan for periods of rest throughout the day.

The client is 4 hours post-lobectomy for lung cancer. Which assessment data
warrant immediate intervention by the nurse?
1. Intake of 1500 mL IV and output of 1000 mL.
2. 450 mL of bright red drainage in the chest tube.
3. Complaining of pain at a 10 on a 1-10 scale.
4. Absent lung sound on the side of surgery.
This is about a pint of blood and could indicate a hemorrhage. HINT: Blood is always
a priority.

The client is admitted to the outpatient surgery center for a bronchoscopy to rule out
lung cancer. Which info should the nurse teach?
1. The test will confirm the MRI results.
2. The client can eat and drink immediately after the test.
3. The HCP can do a biopsy of the tumor through the scope.
4. There is no discomfort associated with this procedure.

The client with oat cell carcinoma tells the nurse, "i am so tired of all this. I might as
well just end it all." Which should be the nurse's first response?
1. "This must be hard for you. Would you like to talk?"
2. Tell the HCP of the client's statement.
3. Refer clients to social workers.
4. Find out if the client has a plan to carry out suicide.
Statements about taking his own life should be followed up to see if there is a plan
which would indicate an emergency.
Which clinical manifestation would the nurse expect to find in newly diagnosed
intrinsic LC?
1. Dysphagia
2. Foul smelling breath
3. Hoarseness
4. Weight loss

Which psychosocial problem would the nurse write for the client diagnosed with LC
metastasis to the brain?
1. Seizures
2. Grieving
3. Body Image
4. Nutrition

LIVER CANCER
1. Which condition is NOT a known cause of cirrhosis?
A. Obesity
B. Alcohol consumption
C. Blockage of the bile duct
D. Hepatitis C
E. All are known causes of Cirrhosis
The answer is E. All of these conditions can cause cirrhosis.

2. The liver receives its blood supply from two sources. One of these sources is
called the _________________, which is a vessel network that delivers blood
_____________ in nutrients but ________ in oxygen.
A. hepatic artery, low, high
B. hepatic portal vein, high, low
C. hepatic lobule, high, low
D. hepatic vein, low, high
The answer is B. Majority of the blood flow to the liver comes from the hepatic portal
vein. This vessel network delivers blood HIGH in nutrients (lipids, proteins, carbs
etc.) from organs that aid in the digestion of food, but the blood is POOR in oxygen.
The organs connected to the hepatic portal vein are: small/large intestine, pancreas,
spleen, stomach. Rich oxygenated blood comes from the hepatic artery to the liver.

3. A patient with late-stage cirrhosis develops portal hypertension. Which of the


following options below are complications that can develop from this condition?
Select all that apply:
A. Increase albumin levels
B. Ascites
C. Splenomegaly
D. Fluid volume deficient
E. Esophageal varices
The answers are B, C, and E. Portal Hypertension is where the portal vein becomes
narrow due to scar tissue in the liver, which is restricting the flow of blood to the liver.
Therefore, pressure becomes increased in the portal vein and affects the organs
connected via the vein to the liver. The patient may experience ascites, enlarged
spleen “splenomegaly”, and esophageal varices etc.

4. Your patient with cirrhosis has severe splenomegaly. As the nurse you will make it
a priority to monitor the patient for signs and symptoms of? Select all that apply:
A. Thrombocytopenia
B. Vision changes
C. Increased PT/INR
D. Leukopenia
The answers are A, C, and D. A patient with an enlarged spleen (splenomegaly) due
to cirrhosis can experience thrombocytopenia (low platelet count), increased PT/INR
(means it takes the patient a long time to stop bleeding), and leukopenia (low white
blood cells). The spleen stores platelets and WBCs. An enlarged spleen can develop
due to portal hypertension, which causes the platelets and WBCs to become stuck
inside the spleen due to the increased pressure in the hepatic vein (hence lowering
the count and the body’s access to these important cells for survival).

5. A patient is admitted with hepatic encephalopathy secondary to cirrhosis. Which


meal option selection below should be avoided with this patient?
A. Beef tips and broccoli rabe
B. Pasta noodles and bread
C. Cucumber sandwich with a side of grapes
D. Fresh salad with chopped water chestnuts
The answer is A. Patients who are experiencing hepatic encephalopathy are having
issues with toxin build up in the body, specifically ammonia. Remember that
ammonia is the byproduct of protein breakdown, and normally the liver can take the
ammonia from the protein breakdown and turn it into urea (but if the cirrhosis is
severe enough this can’t happen). Therefore, the patient should consume foods
LOW in protein until the encephalopathy subsides. Option A is very high in protein
while the others are low in protein. Remember meats, legumes, eggs, broccoli rabe,
certain grains etc. are high in protein.

6. During your morning assessment of a patient with cirrhosis, you note the patient is
disoriented to person and place. In addition while assessing the upper extremities,
the patient’s hands demonstrate a flapping motion. What lab result would explain
these abnormal assessment findings?
A. Decreased magnesium level
B. Increased calcium level
C. Increased ammonia level
D. Increased creatinine level
The answer is C. Based on the assessment findings and the fact the patient has
cirrhosis, the patient is experiencing hepatic encephalopathy. This is due to the
buildup of toxins in the blood, specifically ammonia. The flapping motion of the hands
is called “asterixis”. Therefore, an increased ammonia level would confirm these
abnormal assessment findings.

7. You are receiving a shift report on a patient with cirrhosis. The nurse tells you the
patient’s bilirubin levels are very high. Based on this, what assessment findings may
you expect to find during your head-to-toe assessment? Select all that apply:
A. Frothy light-colored urine
B. Dark brown urine
C. Yellowing of the sclera
D. Dark brown stool
E. Jaundice of the skin
F. Bluish mucous membranes
The answers are B, C, and E. High bilirubin levels are because the hepatocytes are
no longer able to properly conjugate the bilirubin because they are damaged. This
causes bilirubin to leak into the blood and urine (rather than entering the bile and
being excreted in the stool). Therefore, the bilirubin stays in the blood and will enter
the urine. This will cause the patient to experience yellowing of the skin, sclera of the
eyes, and mucous membranes (“jaundice”) and have dark brown urine. The stools
would be CLAY-COLORED not dark brown (remember bilirubin normally gives stool
its brown color but it will be absent).

8. A 45 year old male has cirrhosis. The patient reports concern about the
development of enlarged breast tissue. You explain to the patient that this is
happening because?
A. The liver cells are removing too much estrogen from the body which causes the
testicles to produce excessive amounts of estrogen, and this leads to gynecomastia.
B. The liver is producing too much estrogen due to the damage to the liver cells,
which causes the level to increase in the body, and this leads to gynecomastia.
C. The liver cells are failing to recycle estrogen into testosterone, which leads to
gynecomastia.
D. The liver cells are failing to remove the hormone estrogen properly from the body,
which causes the level to increase in the body, and this leads to gynecomastia.
The answer is D.

9. You’re providing an in-service to new nurse graduates about esophageal varices


in patients with cirrhosis. You ask the graduates to list activities that should be
avoided by a patient with this condition. Which activities listed are correct: Select all
that apply
A. Excessive coughing
B. Sleeping on the back
C. Drinking juice
D. Alcohol consumption
E. Straining during a bowel movement
F. Vomiting
The answers are A, D, E, and F. Esophageal varices are dilated vessels that are
connected from the throat to the stomach. They can become enlarged due to portal
hypertension in cirrhosis and can rupture (this is a medical emergency). The patient
should avoid activities that could rupture these vessels, such as excessive cough,
vomiting, drinking alcohol, and constipation (straining increases thoracic pressure.)

10. While providing mouth care to a patient with late-stage cirrhosis, you note a
pungent, sweet, musty smell to the breath. This is known as:
A. Metallic Hepatico
B. Fetor Hepaticus
C. Hepaticoacidosis
D. Asterixis
The answer is B.

11. The physician orders Lactulose 30 mL by mouth per day for a patient with
cirrhosis. What findings below demonstrate the medication is working effectively?
Select all that apply:
A. Decrease albumin levels
B. Decrease in Fetor Hepaticus
C. Patient is stuporous.
D. Decreased ammonia blood level
E. Presence of asterixis
The answer is B and D. A patient with cirrhosis may experience a complication called
hepatic encephalopathy. This will cause the patient to become confused (they may
enter into a coma), have pungent, musty smelling breath (fetor hepaticus), asterixis
(involuntary flapping of the hands) etc. This is due to the buildup of ammonia in the
blood, which affects the brain. Lactulose can be prescribed to help decrease the
ammonia levels. Therefore, if the medication is working properly to decrease the
level of ammonia the patient would have improving mental status (NOT stuporous),
decreased ammonia blood level, decreasing or absence of asterixis, and decreased
ammonia blood level.

12. ________ reside in the liver and help remove bacteria, debris, and old red blood
cells.
A. Hepatocytes
B. Langerhan cells
C. Enterocytes
D. Kupffer cells
The answer is D. Kupffer cells perform this function and are one of the two types of
cells found in the liver lobules (the functional units of the liver). These cells play a
role in helping the hepatocytes turn parts of the old red blood cells into bilirubin.

13. Which of the following is NOT a role of the liver?


A. Removing hormones from the body
B. Producing bile
C. Absorbing water
D. Producing albumin
The answer is C. The liver does not absorb water. The intestines are responsible for
this function.

A nurse is conducting a group counseling session for patients with liver cancer. One
of the participants begins to cry and is upset that his terminal cancer was not
diagnosed earlier. Which statements by other members of the group should be
corrected by the nurse?
"Maybe you should request a needle biopsy to confirm the diagnosis."

A nurse is caring for a young man with a history of alcoholic cirrhosis. He presents to
the clinic with pain in the upper right quadrant, weight loss, and fever. Which action
by the nurse is most appropriate?
Talk to the health care provider about ordering an abdominal CT scan.

A nurse is caring for a patient who is diagnosed with liver cancer. As part of the initial
diagnostic testing, the provider orders a chest CT scan. The patient asks why this
test is necessary. How should the nurse respond?
"The CT scan is used to determine if there is any metastasis."

The nurse is educating people at a health fair about primary liver cancer. Which
statement made by a participant indicates a need for further teaching?
"I drink about 4-6 beers or cocktails every night when I come home from work."

The nurse is completing the preoperative checklist for a patient with primary liver
cancer who is undergoing a partial hepatectomy. Which statement made by the
patient indicates the patient understands the procedure?
"The surgeon is going to remove the cancerous tumors."

A nurse is caring for an older patient who was just informed that his or her liver
cancer is not responding to chemotherapy. The patient has a history of alcoholic
cirrhosis and does not have any family living nearby. Which option should the nurse
discuss with the patient?
Finding supportive hospice care
The nurse is assessing a patient with diagnosed early primary liver cancer. The
patient asks the nurse about signs that the cancer is progressing. How should the
nurse respond?
You may notice pain in the right upper quadrant of your abdomen

The nurse is working with a patient admitted with symptoms of fatigue, edema,
ascites, jaundice, and a mass in the upper right quadrant. The health care provider
has ordered an abdominal CT scan and an alpha-fetoprotein blood level. What is the
rationale for the health care provider choosing an abdominal CT scan and bloodwork
instead of a liver biopsy to confirm the diagnosis?
A liver biopsy introduces a needle into the tumor and can spread the disease

The nurse is reviewing the diagnostic reports for a patient admitted with a palpable
mass in the liver. Which finding in the medical record suggests metastatic liver
cancer as opposed to primary liver cancer?
Masses noted in the brain, breast, and colon

The nurse is preparing the patient for a transarterial chemoembolization of a liver


tumor. Which statement made by the patient indicates an understanding of the
procedure?
This procedure cuts off the blood supply to the tumor

A patient is seen in the oncology clinic for follow-up after treatment of primary liver
cancer with sorafenib. Which finding would indicate an improvement in the patient's
status?
Decreased waist size

The nurse is caring for a patient who underwent a partial hepatectomy for primary
liver cancer a few days ago. Which assessment findings suggest possible
complications? Select all
Jaundice
Changes in mental status

THYROID CANCER
1. Fill in the blank regarding the negative feedback loop for thyroid hormone
production: The ______________ produces TRH (Thyrotropin-Releasing Hormone)
which causes the anterior pituitary gland to produce _______________ which in turn
causes the thyroid gland to release _______ and _______.
A. Thalamus, CRH (Corticotropin-releasing hormone) TSH (thyroid-stimulating
hormone) and T4
B. Hypothalamus, TSH (thyroid-stimulating hormone), T3 and T4
C. Posterior pituitary gland, TSH (thyroid-stimulating hormone), T3 and T4
D. Hypothalamus, CRH (Corticotropin-releasing hormone), TSH (thyroid-stimulating
hormone), T3 and TSH
The answer is B: Hypothalamus, TSH (thyroid-stimulating hormone), T3 and T4

2. A patient reports they do not eat enough iodine in their diet. What condition are
they most susceptible to?
A. Pheochromocytoma
B. Hyperthyroidism
C. Thyroid Storm
D. Hypothyroidism
The answer is D: Hypothyroidism…Iodine helps make T3 and T4….if a person does
not consume enough iodine they are at risk for developing HYPOTHYROIDISM.

3. A patient has an extremely high T3 and T4 level. Which of the following signs and
symptoms DO NOT present with this condition?
A. Weight loss
B. Intolerance to heat
C. Smooth skin
D. Hair loss
The answer is D: Hair loss

4. A patient is being discharged home for treatment of hypothyroidism. Which


medication is most commonly prescribed for this condition?
A. Tapazole
B. PTU (Propylthiouracil)
C. Synthroid
D. Inderal
The answer is C: Synthroid is the only medication listed that treats hypothyroidism.
All the other medications are used for hyperthyroidism.

5. You are performing discharge teaching with a patient who is going home on
Synthroid. Which statement by the patient causes you to re-educate the patient
about this medication?
A. “I will take this medication at bedtime with a snack.”
B. “I will never stop taking the medication abruptly.”
C. “If I have palpitations, chest pain, intolerance to heat, or feel restless, I will notify
the doctor.”
D. “I will not take this medication at the same time I take my Carafate.”
The answer is A: Synthroid is best taken in the MORNING on an empty stomach. All
the other statements are correct about taking Synthroid.

6. The thyroid hormones, T3 and T4, play many roles in the human body. Which of
the following functions are performed by T3 and T4? Note: Select all that apply
A. Storing calories
B. Increasing the Heart Rate
C. Stimulating the Sympathetic Nervous System
D. Decreasing the body’s temperature
E. Regulating TSH produced by the anterior pituitary gland
The answers are B, C, and E. T3 and T4 burn calories (not store them) and increase
body temperature (not decrease).

7. A patient is admitted with complaints of palpitations, excessive sweating, and


unable to tolerate heat. In addition, the patient voices concern about how her
appearance has changed over the past year. The patient presents with protruding
eyeballs and pretibial myxedema on the legs and feet. Which of the following is the
likely cause of the patient’s signs and symptoms?
A. Thyroiditis
B. Deficiency of iodine consumption
C. Grave’s Disease
D. Hypothyroidism
The answer is C: Grave’s Disease

8. A patient who is in her first trimester of pregnancy is diagnosed with


hyperthyroidism. Which medication do you suspect the patient will be started on?
A. Propylthiouracil (PTU)
B. Radioactive Iodine
C. Tapazole
D. Synthroid
The answer is A: Propylthiouracil (PTU) is the only anti-thyroid medication that can
be used during the 1st trimester of pregnancy.

9. Which of the following are treatment options for hyperthyroidism? Please select all
that apply:
A. Thyroidectomy
B. Methimazole
C. Liothyronine Sodium “Cytomel”
D. Radioactive Iodine
The answers are A, B,and D. Liothyronine Sodium “Cytomel” is a treatment for
hypothyroidism. All the other options are for hyperthyroidism.

10. A patient was recently discharged home for treatment of hypothyroidism and was
ordered to take Synthroid for treatment. The patient is re-admitted with signs and
symptoms of the following: heart rate 42, blood pressure 70/56, blood glucose 55,
and body temperature of 96.8 ‘F. The patient is very fatigued and drowsy. The family
reports the patient has not been taking Synthroid since being discharged home from
the hospital. Which of the following conditions is this patient most likely
experiencing?
A. Thryoid Storm
B. Myxedema Coma
C. Iodism
D. Toxic Nodular Goiter
The answer is B: Myxedema Coma…The red flags in this question are the patient’s
signs/symptoms and the report from the family the patient hasn’t been taking the
prescribed Synthroid. The patient is showing signs and symptoms of extreme
hypothyroidism known as Myxedema coma (which is life-threatening if not treated).

11. A patient is being educated on how to take their anti-thyroid medication. Which of
the following statements are INCORRECT?
A. “I will continue taking aspirin daily.”
B. “I will take this medication at the same time every day.”
C. “It may take a while before I notice that the medication is helping my condition.”
D. “I will avoid foods containing high levels of iodine.”
The answer is A: The patient needs to be instructed NOT to take aspirin because it
increases thyroid hormones. All the other statements are correct.

12. A patient with hypothyroidism is having pain 6 on 1-10 scale in the right hip due
to recent hip surgery. Which of the following medications are NOT appropriate for
this patient? Select all that apply:
A. Fentanyl
B. Tylenol
C. Morphine
D. Dilaudid
The answers are A, C, and D. Patients who have hypothyroidism are very sensitive
to narcotics and should take NON-NARCOTICS for pain relief. Fentanyl, Morphine,
and Dilaudid are all narcotics, whereas Tylenol is not.

13. A patient is 6 hours post-opt from a thyroidectomy. The surgical site is clean, dry
and intact with no excessive swelling noted. What position is best for this patient to
be in?
A. Fowler’s
B. Prone
C. Trendelenburg
D. Semi-Fowler’s
The answer is D: Semi-Fowler’s

14. Which of the following signs and symptoms causes concern and requires nursing
intervention for a patient who recently had a thyroidectomy?
A. Heart rate of 120, blood pressure 220/102, temperature 103.2 ‘F
B. Heart rate of 35, blood pressure 60/43, temperature 95.3 ‘F
C. Soft hair, irritable, diarrhea
D. Constipation, drowsiness, goiter
The answer is A. A patient is at risk for experiencing thyroid storm after a
thyroidectomy because of manipulation of the thryroid gland that could cause
excessive T3 and T4 to enter into the bloodstream during removal of the gland.
Therefore, heart rate of 120, blood pressure 220/102, temperature 103.2 ‘F are
classic signs of thyroid storm and this requires nursing intervention.

15. ___________ is an autoimmune disorder where the body attacks the thyroid
gland that causes it to stop releasing T3 and T4. The patient is likely to have the
typical signs/symptoms of hypothyroidism, however, they may present with what
other sign as well?
A. Myxedema coma; joint pain
B. Thyroid storm; memory loss
C. Hashimoto’s Thyroiditis; goiter
D. Toxic nodular goiter (TNG); goiter
The answer is C: Hashimoto’s Thyroiditis; goiter

16. Which of the following side effects are possible for a patient taking an anti-thyroid
medication?
A. Agranulocytosis and aplastic anemia
B. Tachycardia
C. Skin discoloration
D. Joint pain and eczema
The answer is A: Agranulocytosis and aplastic anemia

17. A patient is receiving radioactive iodine treatment for hyperthyroidism. What will
you include in your patient education to this patient about this type of treatment?
A. Taste changes and swollen salivary glands
B. Constipation
C. Excessive thirst
D. Sun protection

The client is diagnosed with hypothyroidism. Which signs/symptoms should the


nurse expect the client to exhibit?
1. Complaints of extreme fatigue and hair loss.
2. Exophthalmos and complaints of nervousness.
3. Complaints of profuse sweating and flushed skin.
4. Tetany and complaints of stiffness of the hands.

The nurse identifies the client problem "risk for imbalanced body temperature" for the
client diagnosed with hypothyroidism. Which intervention should be included in the
plan of care?
1. Discourage the use of an electric blanket.
2. Assess the client's temperature every two (2) hours.
3. Keep the room temperature cool.
4. Space activities to promote rest.
External heat sources (heating pads,electric or warming blankets) should be
discouraged because they increase the risk of peripheral vasodilation and vascular
collapse.

The client diagnosed with hypothyroidism is prescribed the thyroid hormone


levothyroxine (Synthroid). Which assessment data indicate the medication has been
effective?
1. The client has a three (3)-pound weight gain.
2. The client has a decreased pulse rate.
3. The client's temperature is WNL.
4. The client denies any diaphoresis.
The client with hypothyroidism frequently has a subnormal temperature,so a WNL
indicates the medication is effective.

Which nursing intervention should be included in the plan of care for the client
diagnosed with hyperthyroidism?
1. Increase the amount of fiber in the diet.
2. Encourage a low-calorie, low-protein diet.
3. Decrease the client's fluid intake to 1,000 mL/day.
4. Provide six (6) small, well-balanced meals a day.

The client is admitted to the intensive care department diagnosed with


myxedemacoma. Which assessment data warrant immediate intervention by the
nurse?
1. Serum blood glucose level of 74 mg/dL.
2. Pulse oximeter reading of 90%.
3. Telemetry reading showing sinus bradycardia.
4. The client is lethargic and sleeps all the time.

Which medication order should the nurse question in the client diagnosed with
untreated hypothyroidism?
1. Thyroid hormones.
2. Oxygen.
3. Sedatives.
4. Laxatives.

Which statement made by the client makes the nurse suspect the client is
experiencing hyperthyroidism?
1. "I just don't seem to have any appetite anymore."
2. "I have a bowel movement about every 3 to 4 days."
3. "My skin is really becoming dry and coarse."
4. "I have noticed all my collars are getting tighter."
The 68-year-old client diagnosed with hyperthyroidism is being treated with
radioactive iodine therapy. Which interventions should the nurse discuss with the
client?
1. Explain it will take up to a month for symptoms of hyperthyroidism to subside.
2. Teach the iodine therapy will have to be tapered slowly over one (1) week.
3. Discuss the client will have to be hospitalized during the radioactive therapy.
4. Inform the client after therapy the client will not have to take any medication.

The nurse is teaching the client diagnosed with hyperthyroidism. Which information
should be taught to the client? Select all that apply.
1. Notify the HCP if a three (3)-pound weight loss occurs in two (2) days.
2. Discuss ways to cope with emotional lability.
3. Notify the HCP if taking over-the-counter medication
4. Carry a medical identification card or bracelet.
5. Teach how to take thyroid medications correctly.

The nurse is providing an in-service on thyroid disorders. One of the attendees asks
the nurse, "Why don't the people in the United States get goiters as often?" Which
statement by the nurse is the best response?
1. "It is because of the screening techniques used in the United States."
2. "It is a genetic predisposition rare in North Americans."
3. "The medications available in the United States decrease goiters."
4. "Iodized salt helps prevent the development of goiters in the United States."

The nurse is preparing to administer the following medications. Which medication


should the nurse question administering?
1. The thyroid hormone to the client who does not have a T3, T4 level.
2. The regular insulin to the client with a blood glucose level of 210 mg/dL.
3. The loop diuretic to the client with a potassium level of 3.3 mEq/L.
4. The cardiac glycoside to the client who has a digoxin level of 1.4 mg/dL.

Which signs/symptoms should make the nurse suspect the client is experiencing a
thyroid storm?
1. Obstipation and hypoactive bowel sounds.
2. Hyperpyrexia and extreme tachycardia.
3. Hypotension and bradycardia.
4. Decreased respirations and hypoxia.
Hyperpyrexia (high fever) and heart rate above 130 beats per minute are signs of
thyroid storm, a severely exaggerated hyperthyroidism

A client is admitted to an emergency department, and a diagnosis of myxedema


coma is made. Which action would the nurse prepare to carry out initially?
1. Warm the client.
2.Maintain a patent airway.
3.Administer thyroid hormone.
4.Administer fluid replacement.

The nurse is preparing a client with a new diagnosis of hypothyroidism for discharge.
The nurse determines that the client understands discharge instructions if the client
states that which symptoms are associated with this diagnosis? Select all that apply.
1.Tremors
2.Weight loss
3.Feeling cold
4.Loss of body hair
5.Persistent lethargy
6.Puffiness of the face

A client has been diagnosed with hyperthyroidism. Which signs and symptoms may
indicate thyroid storm, a complication of this disorder? Select all that apply.
1.Fever
2.Nausea
3.Lethargy
4.Tremors
5.Confusion
6.Bradycardia

The nurse should include which interventions in the plan of care for a client with
hypothyroidism? Select all that apply.
1.Provide a cool environment for the client.
2.Instruct the client to consume a high-fat diet.
3.Instruct the client about thyroid replacement therapy.
4.Encourage the client to consume fluids and high-fiber foods in the diet.
5.Inform the client that iodine preparations will be prescribed to treat the disorder.
6.Instruct the client to contact the health care provider (HCP) if episodes of chest
pain occur

The nurse is seeing four clients today in the endocrinology clinic. Which client would
the nurse expect to be at highest risk for Hashimoto​disease?
A. A​50-year-old man with hypothyroidism
B. A​40-year-old woman with a goiter
C. A​60-year-old African American man
D. A​10-year-old child with congenital hypothyroidism

A client with a family history of hyperthyroidism asks the​nurse, "What can increase
my risk of developing this​disorder?" Which response by the nurse is​accurate?
(Select All That Apply)
A.​"Viral infections can cause the onset of​hyperthyroidism."
B. "Arthritis can lead to the development of​hyperthyroidism.
C. ​"Invasive neck surgery can impact thyroid​functioning."
D. "Smoking can increase your risk for acquiring this disease.

The nurse is teaching a group of adults at a community health fair about


hypothyroidism. Which risk factor should the nurse include in the​presentation?
(Select all that​apply.)
A. Thyroid surgery
B. Radioactive iodine treatment
C. Autoimmune disease
D. Radiation of the neck
E. Male sex

Which priority should the nurse include in the teaching plan for a client with Graves​
disease? (Select all that​apply.)
A. Drink six to eight glasses of water a day.
B. Eat a​low-calorie diet.
C. Tape your eyelids closed at night.
D. Weigh yourself daily.
E. Take antithyroid drugs as prescribed.

The nurse is monitoring a client receiving levothyroxine sodium for hypothyroidism.


Which findings indicate the presence of a side effect associated with this
medication? (Select All That Apply)
A) Insomnia
B) Weight Loss
C) Bradycardia
D) Constipation
E) Mild Heat Intolerance

The nurse is caring for a client newly diagnosed with Graves disease. The client
asks the nurse how the goiter occurred. Which factor should the nurse include in the​
response? (Select all that​apply.)
A.The thyroid gland enlarges.
B.Antibodies bind to the​thyroid-stimulating hormones.
C.The thyroid cells become hypoactive.
D.The​client's tissues form antigens.
E.The​client's tissues form antibodies.

An adult client reports a weight gain and feeling cold all the time. Which condition
should the nurse​suspect?
A.Hypothyroidism
B.Chronic renal failure
C.Hyperthyroidism
D.Depression
A​heel-stick screening of a newborn reveals the presence of T4 deficiency along with
elevated​thyroid-stimulating hormone​(TSH). The infant is diagnosed with
hypothyroidism. Which information should the nurse provide the​parents?
A.The child will need lifelong thyroid medication supplementation.
B.The child will eventually grow out of this and no longer need treatment.
C.The child will be involved in infertility treatment later in life.
D.The child will require evaluation for radioactive iodine.

The nurse in the fertility clinic is working with a female client who has had repeated
miscarriages. Which information in the​client's history may be a precipitating​factor?
A.History of toxic multinodular goiter
B.Type 2 diabetes mellitus
C.Uncontrolled hypothyroidism
D.Hyperemesis gravidarum

The nurse is caring for a newborn diagnosed with hyperthyroidism after birth.
Ongoing assessments during the first year of life should be conducted to monitor for
which​alteration? (Select all that​apply.)
A.Respiratory difficulties
B.Heart failure
C.Bradycardia
D.Premature fontanelle closure
E.Nonpalpable thyroid gland

The nurse is teaching an older adult how to manage Graves disease. Which
information should the nurse​include?
A.Use of levothyroxine​(Synthroid) and lab monitoring
B.The schedule for lifelong radioactive iodine treatments
C.Preparation for surgical removal of the thyroid
D.The administration schedule for an antithyroid drug

Which treatment should the nurse anticipate for a client who is newly diagnosed with​
hypothyroidism?
A.Partial thyroidectomy
B.Treatment with synthetic hormone
C.Radiation
D.Nonsteroidal​anti-inflammatory medications

The nurse reviews the laboratory results for a client and notes that the T4 level is
low. Which prescription should the nurse anticipate the healthcare provider to​
prescribe?
A.Radioactive iodine
B.Thyroid replacement
C.Antithyroid medications
D.Beta blocker

The nurse is preparing a client with hyperthyroidism for radioactive iodine


treatments. Which information should the nurse provide to the client prior to this​
procedure? (Select all that​apply.)
A.That the client may need lifelong thyroid replacement
B.That radioactive iodine is given intravenously
C.How to measure the radial pulse
D.That the end results are immediately seen
E.That hospitalization is usually required

The client with hypothyroidism asks the nurse why the​thyroid-stimulating hormone​
(TSH) level is increased if the thyroid is not working properly. Which response by the
nurse is​accurate?
A."Your TSH level is increased due to an inadequately functioning negative hormonal
feedback​process."
B."Your TSH level is increased because the thyroid is working harder to produce
more​hormones."
C."Your TSH level is increased from a malfunction in the​hypothalamus, leading to
thyroid​insufficiency."
D."Your TSH level is increased due to an increase in metabolism noted in clients with​
hypothyroidism."

The nurse is teaching colleagues about hyperthyroidism. Which statement by a


colleague indicates understanding of an indication for a​thyroidectomy? (Select all
that​apply.)
A."The client may require a thyroidectomy for cosmetic​reasons, such as a large​
goiter."
B."A total thyroidectomy is performed to treat cancer of the​thyroid."
C."The client will not need surgery as long as she takes antithyroid​medication."
D."A thyroidectomy may be performed if the thyroid is placing pressure on the​
esophagus."
E."A thyroidectomy may be performed if the thyroid is compromising the​airway."

Which physical assessment parameter is most appropriate for the nurse to include
when assessing the client for possible​hyperthyroidism? (Select all that​apply.)
A.Vision test
B.Vital signs
C.Deep tendon reflexes
D.Confusion
E.Weight loss

A client reports hoarseness and feelings of tightness in the throat. During the​
examination, the nurse notes visible swelling at the base of the​neck, neck vein​
distention, a rapid​pulse, and sweating. The nurse should suspect which condition in
this​client?
A.toxic multinodular goiter
B.Pretibial myxedema
C.Exophthalmos
D.Graves disease
Graves disease involves an enlargement of the thyroid gland due to overproduction
of thyroid hormones.​

Which manifestation should the nurse monitor when caring for a client on thyroid
hormone​(TH) replacement​therapy? (Select all that​apply.)
A.Report of dizziness
B.Symptoms of hyperthyroidism
C.Stable vital signs
D.Improvement of symptoms of hypothyroidism
E.Decrease in appetite

Which physical assessment is most appropriate to include when identifying thyroid​


problems? (Select all that​apply.)
A.Auscultation
B.Percussion
C.Palpation
D.Medication history
E.Observation

Which nursing intervention is most appropriate for a client experiencing a thyroid​


storm?
A.Padding the side rails
B.Replacing lost fluids
C.Cooling the client
D.Administering antithyroid medication

COLON CANCER
The occupational health nurse is preparing a presentation to a group of factory
workers about preventing colon cancer. Which information should be included?
1. Wear a high filtration mask around chemicals.
2. Eat several servings of cruciferous vegetables daily.
3. Take a multivitamin daily.
4. Do not engage in high-risk sexual behavior.

The nurse is admitting a male client with a diagnosis of adenocarcinoma of the


rectosigmoid colon. Which assessment data support this diagnosis?
1. Reports up to 20 bloody stools per day.
2. States he has a feeling of fullness after a heavy meal.
3. Has diarrhea alternating with constipation.
4. Complaints of RLQ pain with rebound tenderness.

85 y.o. male client diagnosed with colon cancer asks the nurse, "Why did I get colon
cancer?" Which is the best response to colon cancer?
1. Lack of fiber in the diet.
2. Greatest incidence among those younger than 50.
3. Has no known risk factors.
4. Rare among male clients.

The nurse is planning care of a client who has had an abdominal perineal resection
for colon cancer. Which interventions should the nurse implement? Select all that
apply:
1. Provide meticlulous skin care to stoma.
2. Assess the flank incision.
3. Maintain the indwelling catheter.
4. Irrigate the JP drains every shift.
5. Position the client semi-recumbent.

The client who has had an abdominal perineal resection is being discharged. Which
info should a nurse teach?
1. The stoma should be a white, blue, or purple color.
2. Limit ambulation to prevent the pouch from coming off.
3. Take pain meds when the pain level is at 8.
4. Empty pouch when 1/3 to 1/2 full.

The nurse caring for the pt 1day post op sigmoid resection notes a moderate amount
of dark reddish brown drainage on the midline abdominal incision. Which intervention
is first?
1. Mark the drainage on the dressing with the time and date.
2. Change the dressing immediately using sterile technique.
3. Notify the health care provider immediately.
4. Reinforce the dressing with a sterile gauze pad.

The pt complains to the nurse of unhappiness with the HCP. Which intervention
should the nurse do next?
1. Call HCP and suggest he or she talk to pt
2. Determining what about the HCP is bothering pt.
3. Notify the nursing supervisor to arrange a new HCP to take over.
4. Explain that pt. has to keep HCP till after discharge.
The pt with a new colostomy is being discharged. Which statement indicates a need
for further teaching?
1. If I notice any skin breakdown I will call HCP.
2. I should drink only liquids until the colostomy starts to work.
3. I should not take a tub bath until the HCP says it's ok.
4. I should not drive or lift more than 5 pounds.

The nurse is caring for pts in an outpatient clinic. Which info should the nurse teach
regarding the American Cancer Society's recommendations for early detection of
colon cancer?
1. Beginning at age 60, a digital rectal exam should be done annually.
2. After pt reaches middle age, yearly fecal occult test.
3. At age 50, a colonoscopy, then once every 5-10 years.
4. A flexible sigmoidoscopy should be done yearly after age 40.

The nurse writes a psychosocial problem of "risk for altered sexual functioning
related to new colostomy. Which intervention should the nurse implement?
1. Tell pt. that there should be no intimacy for at least 3 months.
2. Ensure that the pt and partner are able to change the ostomy pouch.
3. Demonstrate with charts possible sexual positions for the pt. to assume.
4. Teach the pt. to protect the pouch from being dislodged during sex.

The pt presents with a complete blockage of the large intestine from a large tumor.
Which HCP's order would the nurse question?
1. Obtain consent for a colonoscopy and biopsy.
2. Start an IV of 0.9% saline at 125 mL/hr.
3. Administer 3 liters of GoLytely.
4. Give tap water enemas until it is clear.

The nurse is interviewing a male client about his past medical history. Which
preexisting condition may lead the nurse to suspect that a client has colorectal
cancer?
a. Duodenal ulcers
b. Hemorrhoids
c. Weight gain
d. Polyps

A client undergoes a colonoscopy for colorectal cancer screening. During the


procedure three small polyps were removed. Which nursing procedures are
necessary when caring for the client immediately after the colonoscopy?
a. Monitor vital signs and inform her that there may be a small amount of blood in her
stool.
b. Observe signs and symptoms of bowel perforation, monitor vital signs, and inform
her that there may be a small amount of blood in her stool and to report excessive
blood loss.
c. Observe signs and symptoms of bowel perforation, monitor vital signs, and inform
her to follow a clear liquid diet.
d. Monitor vital signs and inform her that there may be a small amount of blood in her
stool, and tell her not to drive for two days

A nurse is teaching a client about the risk factors associated with colorectal cancer.
The nurse determines that further teaching related to colorectal cancer is necessary
if the client identifies which of the following as an associated risk factor?
a. Age younger than 50
b. history of colorectal polyps
c. family history of colorectal cancer
d. chronic IBD

A patient with metastatic colorectal cancer is scheduled for both chemotherapy and
radiation therapy. Patient teaching regarding these therapies for this patient would
include an explanation that:
a. Chemotherapy can be used to cure colorectal cancer
b. Radiation is routinely used as adjuvant therapy following surgery
c. Both chemotherapy and radiation can be used as palliative treatments
d. The patient should expect few if any side effects from chemo-therapeutic agents

The nurse is planning care for a 68-year-old patient with an abdominal mass and
suspected bowel obstruction. Which of the following factors in
the patient's history increases the patient's risk for colorectal cancer?
a. Osteoarthritis
b. History of rectal polyps
c. History of lactose intolerance
d. Use of herbs as dietary supplements

The nurse is teaching a client about the modifiable risk factors that can reduce the
risk for colorectal cancer. The nurse places the highest priority on discussing which
risk factor with this client?
a. Age older than 30 years
b. High fat and low fiber diet
c. Distant relative with colorectal cancer
d. Personal history of ulcerative colitis or GI polyps

The nurse is teaching a client about the risk factors associated with colorectal
cancer. The nurse determines that further teaching is necessary if the client identifies
which of the following as an associated risk factor?
a. a history of inflammatory bowel disease
b. family history of colon cancer
c. a high fiber diet
d. a diet high in fats and carbohydrates

A client has been diagnosed with colon cancer of the rectum. While completing the
preoperative checklist the client asks the nurse "Where will my stoma be?" The
nurse's best response is:
a. right upper quadrant.
b. left upper quadrant
c. right lower quadrant
d. left lower quadrant

A client with cancer of the colon who is receiving chemotherapy tells a nurse that
some foods on the meal tray taste bitter. The nurse would try to limit which of the
following foods that is most likely to cause this taste for the client?
a. cantaloupe
b. potatoes
c. beef
d. custard

A nurse is reviewing the preoperative prescriptions for a client with colon tumor who
is scheduled for abdominal perineal resection and notes that the physician has
prescribed neomycin (Mycifradin) for the client. The nurse determines that this
medication has been prescribed primarily for which of the following purposes?
a. To prevent an immune dysfunction
b. Because the client has an infection
c. To decrease the bacteria in the bowel
d. Because the client is allergic to penicillin

During a routine health examination, a 30-year-old patient tells the nurse about a
family history of colon cancer. The nurse will plan to:
a. teach the patient about the need for a colonoscopy at age 50.
b. ask the patient to bring in a stool specimen to test for occult blood.
c. schedule a sigmoidoscopy to provide baseline data about the patient.
d. have the patient ask the doctor about specific tests for colon cancer.

The 85-year-old male client diagnosed with cancer of the colon asks the nurse, "Why
did I get this cancer?" Which statement is the nurse's best response?
a.Cancer of the colon is associated with a lack of fiber in the diet.
b.Cancer of the colon has a greater incidence among those younger than age 50
years.
c.Cancer of the colon has no known risk factors.
d.Cancer of the colon is rare among male clients.
The client with polyps has watched a video on primary prevention of colon cancer.
The nurse can evaluate teaching effectiveness when the client states "I should
follow:
a. low sodium low fat high fiber diet."
b. a low fat, low refined sugar and decrease red meat while eating more fiber."
c. a gluten free, low fat diet."
d. a low carbohydrate, low fat, low tyramine diet."

The night before surgery for colon cancer, the client refuses the bowel preparation
and angrily threatens to leave the hospital. Which of the following is the best
response?
a. "A tranquilizer will help soothe your nerves and make the preparation less painful".
b. "I'll call your minister to sit with you until you calm down and take your
medications".
c. "Tell me what is upsetting you and what I can do to help."
d. "I will call the physician to come and talk to you again about the surgery".

The nurse is caring for a patient receiving an initial dose of chemotherapy to treat a
rapidly growing metastatic colon cancer. The nurse is aware that this patient is at risk
for tumor lysis syndrome (TLS) and will monitor the patient closely for which of the
following abnormalities associated with this oncologic emergency?
a. hypokalemia
b. hypocalcemia
c. hypouricemia
d. hypophosphatemia

The nurse is performing an admission assessment on a client diagnosed with a right


colon tumor. The nurse asks the client about which characteristic symptom of this
type of tumor?
a. rectal bleeding
b. flat, ribbon-like stool
c. crampy, colicky abdominal pain
d. alternating constipation and diarrhea

A patient who has been told by the health care provider that the cells in a bowel
tumor are poorly differentiated asks the nurse what is meant by "poorly
differentiated." Which response should the nurse make?
a. "The cells in your tumor do not look very different from normal bowel cells."
b. "The tumor cells have DNA that is different from your normal bowel cells."
c. "Your tumor cells look more like immature fetal cells than normal bowel cells."
d. "The cells in your tumor have mutated from the normal bowel cells."
Teaching a client who has had recent bowel surgery how to facilitate the expulsion of
feces may include the process of increasing intra-abdominal pressure. Which of the
following best matches this process?
a. Crede's maneuver.
b. Valsalva's maneuver.
c. Heimlich's maneuver.
d.Epley's maneuver
The Valsalva's maneuver facilitates the expulsion of feces by closing the glottis and
increasing the intra-abdominal pressure.

Which information obtained by the nurse about a patient with colon cancer who is
scheduled for external radiation therapy to the abdomen indicates a need for patient
teaching?
a. The patient swims a mile 5 days a week.
b. The patient eats frequently during the day.
c. The patient showers with Dove soap daily.
d. The patient has a history of dental caries.

A patient with metastatic cancer of the colon experiences severe vomiting following
each administration of chemotherapy. An important nursing intervention for the
patient is to
a. teach about the importance of nutrition during treatment.
b. have the patient eat large meals when nausea is not present.
c. administer prescribed antiemetics 1 hour before the treatments.
d. offer dry crackers and carbonated fluids during chemotherapy.

When assessing a patient's needs for psychological support after the patient has
been diagnosed with stage I cancer of the colon, which question by the nurse will
provide the most information?
a. "Can you tell me what has been helpful to you in the past when coping with
stressful events?"
b. "How long ago were you diagnosed with this cancer?"
c. "Are you familiar with the stages of emotional adjustment to a diagnosis like
cancer of the colon?"
d. "How do you feel about having a possibly terminal illness?"

A nurse assesses clients at a community health center. Which client is at highest risk
for the development of colorectal cancer?
a. A 37-year-old who drinks eight cups of coffee daily
b. A 44-year-old with irritable bowel syndrome (IBS)
c. A 60-year-old lawyer who works 65 hours per week
d. A 72-year-old who eats fast food frequently
A nurse assessing a client with colorectal cancer auscultates high-pitched bowel
sounds and notes the presence of visible peristaltic waves. Which action should the
nurse take?
a. Ask if the client is experiencing pain in the right shoulder.
b. Perform a rectal examination and assess for polyps.
c. Contact the provider and recommend computed tomography.
d. Administer a laxative to increase bowel movement activity.

A nurse prepares a client for a colonoscopy scheduled for tomorrow. The client
states, "My doctor told me that the fecal occult blood test was negative for colon
cancer. I don't think I need the colonoscopy and would like to cancel it." How should
the nurse respond?
a. Your doctor should not have given you that information prior to the colonoscopy.
b. The colonoscopy is required due to the high percentage of false negatives with the
blood test.
c. A negative fecal occult blood test does not rule out the possibility of colon cancer.
d. I will contact your doctor so that you can discuss your concerns about the
procedure.

A nurse cares for a client newly diagnosed with colon cancer who has become
withdrawn from family members. Which action should the nurse take?
a. Contact the provider and recommend a psychiatric consult for the client.
b. Encourage the client to verbalize feelings about the diagnosis.
c. Provide education about new treatment options with successful outcomes.
d. Ask family and friends to visit the client and provide emotional support.

A nurse cares for a client with colon cancer who has a new colostomy. The client
states, "I think it would be helpful to talk with someone who has had a similar
experience." How should the nurse respond?
a. I have a good friend with a colostomy who would be willing to talk with you.
b. The enterostomal therapist will be able to answer all of your questions.
c. I will make a referral to the United Ostomy Associations of America.
d. You'll find that most people with colostomies don't want to talk about them.

A nurse assesses a client who is prescribed 5-fluorouracil (5-FU) chemotherapy


intravenously for the treatment of colon cancer. Which assessment finding should
alert the nurse to contact the health care provider?
a. White blood cell (WBC) count of 1500/mm3
b. Fatigue
c. Nausea and diarrhea
d. Mucositis and oral ulcers

A nurse teaches a client who is recovering from a colon resection. Which statement
should the nurse include in this client's plan of care?
a. You may experience nausea and vomiting for the first few weeks.
b. Carbonated beverages can help decrease acid reflux from anastomosis sites.
c. Take a stool softener to promote softer stools for ease of defecation.
d. You may return to your normal workout schedule, including weight lifting.

A nurse teaches a client who is at risk for colon cancer. Which dietary
recommendation should the nurse teach this client?
a. Eat low-fiber and low-residual foods.
b. White rice and bread are easier to digest.
c. Add vegetables such as broccoli and cauliflower to your new diet.
d. Foods high in animal fat help to protect the intestinal mucosa.

A nurse cares for a client who has a family history of colon cancer. The client states,
"My father & brother had colon cancer. What is the chance that I will get cancer?"
How should the nurse respond?
a. If you eat a low-fat and low-fiber diet, your chances decrease significantly.
b. You are safe. This is an autosomal dominant disorder that skips generations.
c. Preemptive surgery and chemotherapy will remove cancer cells and prevent
cancer.
d. You should have a colonoscopy more frequently to identify abnormal polyps early.

After teaching a client who is recovering from a colon resection, the nurse assesses
the client's understanding. Which statements by the client indicate a correct
understanding of the teaching? (SATA)
a. I must change the ostomy appliance daily and as needed.
b. I will use warm water and a soft washcloth to clean around the stoma.
c. I might start bicycling and swimming again once my incision has healed.
d. Cutting the flange will help it fit snugly around the stoma to avoid skin breakdown.
e. I will check the stoma regularly to make sure that it stays a deep red color.
f. I must avoid dairy products to reduce gas and odor in the pouch.

A client has late-stage colon cancer with metastasis to the spine and bones. Which
nursing intervention does the nurse add to the care plan to address a priority
problem?
a. Provide six small meals and snacks daily.
b. Offer the client prune juice twice a day.
c. Ensure that the client gets adequate rest.
d. Give the client pain medications around the clock.
BREAST CANCER
The nurse is caring for a patient diagnosed with breast cancer who just underwent
an axillary lymph node dissection. What intervention should the nurse use to
decrease the lymphedema?
A. Keep the affected arm flat at the patient's side.
B. Apply an elastic bandage on the affected arm.
C. Assess blood pressure only on unaffected arms.
D. Restrict exercise of the affected arm for 1 week

The nurse is volunteering at a community center to teach women about breast


cancer. What should the nurse include when discussing risk factors (select all that
apply)?
A. Nulliparity
B. Age 30 or over
C. Early menarche
D. Late menopause
E. Personal history of colon cancer

The nurse has been asked to participate in a healthy living workshop. While teaching
about women's health, which guidelines should the nurse provide to the audience?
A. "Mammograms are necessary if you have a family history of breast cancer."
B. "It's recommended that you get a mammogram each year after you turn 40."
C. "If you are not able to perform breast self-examination (BSE), you should go for
regular mammograms."
D. "You should ensure that your primary care provider performs a breast exam each
time you visit."

You might also like